Sunteți pe pagina 1din 147

Tiberiu Trif

Analiza matematica 2
Calcul diferent
,
ial n R
n
Cuprins
Notat
,
ii iii
1 Integrale improprii 1
1.1 Denit
,
ii s
,
i notat
,
ii . . . . . . . . . . . . . . . . . . . . . . . . . . 1
1.2 Integrala Riemann vs. integrala improprie . . . . . . . . . . . . 4
1.3 Calculul integralelor improprii . . . . . . . . . . . . . . . . . . . 7
1.4 Criterii de convergent
,
a pentru integrale improprii . . . . . . . . 10
1.5 Probleme . . . . . . . . . . . . . . . . . . . . . . . . . . . . . . 18
2 Topologie n R
n
27
2.1 Spat
,
iul euclidian R
n
. . . . . . . . . . . . . . . . . . . . . . . . 27
2.2 Structura topologica a spat
,
iului R
n
. . . . . . . . . . . . . . . . 31
2.3 S
,
iruri de puncte n R
n
. . . . . . . . . . . . . . . . . . . . . . . 36
2.4 Mult
,
imi compacte n R
n
. . . . . . . . . . . . . . . . . . . . . . 39
2.5 Probleme . . . . . . . . . . . . . . . . . . . . . . . . . . . . . . 43
2.6 Limite ale funct
,
iilor vectoriale de variabila
vectoriala . . . . . . . . . . . . . . . . . . . . . . . . . . . . . . 48
2.7 Continuitatea funct
,
iilor vectoriale de variabila vectoriala . . . . 51
2.8 Probleme . . . . . . . . . . . . . . . . . . . . . . . . . . . . . . 53
3 Calcul diferent
,
ial n R
n
57
3.1 Spat
,
iul normat al aplicat
,
iilor liniare . . . . . . . . . . . . . . . 57
3.2 Probleme . . . . . . . . . . . . . . . . . . . . . . . . . . . . . . 62
3.3 Derivata unei funct
,
ii vectoriale de variabila
reala . . . . . . . . . . . . . . . . . . . . . . . . . . . . . . . . . 63
3.4 Diferent
,
iabilitatea unei funct
,
ii vectoriale de
variabila vectoriala . . . . . . . . . . . . . . . . . . . . . . . . . 65
3.5 Derivata dupa o direct
,
ie a unei funct
,
ii vectoriale de variabila
vectoriala . . . . . . . . . . . . . . . . . . . . . . . . . . . . . . 69
i
ii
3.6 Derivate part
,
iale ale unei funct
,
ii vectoriale de variabila vectoriala 71
3.7 Probleme . . . . . . . . . . . . . . . . . . . . . . . . . . . . . . 76
3.8 Operat
,
ii cu funct
,
ii diferent
,
iabile . . . . . . . . . . . . . . . . . 85
3.9 Probleme . . . . . . . . . . . . . . . . . . . . . . . . . . . . . . 88
3.10 Diferent
,
iabilitatea funct
,
iei inverse . . . . . . . . . . . . . . . . . 93
3.11 Teoreme de medie pentru funct
,
ii de variabila vectoriala . . . . 96
3.12 Probleme . . . . . . . . . . . . . . . . . . . . . . . . . . . . . . 99
3.13 Funct
,
ii de clasa C
1
. . . . . . . . . . . . . . . . . . . . . . . . . 102
3.14 Teorema difeomorsmului local . . . . . . . . . . . . . . . . . . 104
3.15 Funct
,
ii implicite . . . . . . . . . . . . . . . . . . . . . . . . . . 108
3.16 Probleme . . . . . . . . . . . . . . . . . . . . . . . . . . . . . . 110
3.17 Extreme condit
,
ionate . . . . . . . . . . . . . . . . . . . . . . . 115
3.18 Derivate part
,
iale de ordinul doi . . . . . . . . . . . . . . . . . . 116
3.19 Diferent
,
iala a doua . . . . . . . . . . . . . . . . . . . . . . . . . 119
3.20 Probleme . . . . . . . . . . . . . . . . . . . . . . . . . . . . . . 122
3.21 Condit
,
ii necesare s
,
i suciente de extrem . . . . . . . . . . . . . 126
3.22 Probleme . . . . . . . . . . . . . . . . . . . . . . . . . . . . . . 130
3.23 Derivate part
,
iale s
,
i diferent
,
iale de ordin
superior . . . . . . . . . . . . . . . . . . . . . . . . . . . . . . . 134
3.24 Probleme . . . . . . . . . . . . . . . . . . . . . . . . . . . . . . 136
Bibliograe 137
Index 138
Notat
,
ii
N, Q, R mult
,
imile numerice clasice: a numerelor naturale (fara 0), a
numerelor rat
,
ionale s
,
i respectiv a numerelor reale;
R mult
,
imea extinsa a numerelor reale;

b
a
f(x)dx integrala improprie a funct
,
iei f pe pe intervalul [a, b), cu
< a < b ;

b0
a
f(x)dx integrala improprie a funct
,
iei f pe pe intervalul [a, b), cu
< a < b < ;


a
f(x)dx integrala improprie a funct
,
iei f pe pe intervalul [a, );

b
a+
f(x)dx integrala improprie a funct
,
iei f pe pe intervalul (a, b], cu
a < b < ;

b
a+0
f(x)dx integrala improprie a funct
,
iei f pe pe intervalul (a, b], cu
< a < b < ;

f(x)dx integrala improprie a funct


,
iei f pe pe intervalul (, b];

b
a+
f(x)dx integrala improprie a funct
,
iei f pe pe intervalul (a, b), cu
a < b ;


a+0
f(x)dx integrala improprie a funct
,
iei f pe pe intervalul (a, );
iii
iv Notat
,
ii

b0

f(x)dx integrala improprie a funct


,
iei f pe pe intervalul (, b);

f(x)dx integrala improprie a funct


,
iei f pe pe intervalul (, );
R
n
spat
,
iul euclidian n-dimensional;
0
n
originea lui R
n
;
e
1
, e
2
, . . . , e
n
elementele bazei canonice din R
n
;
x, y produsul scalar al vectorilor x, y R
n
;
x norma vectorului x R
n
;
d(x, y) distant
,
a euclidiana dintre x, y R
n
;
B(a, r) bila deschisa de centru a s
,
i raza r;


B(a, r) bila nchisa de centru a s
,
i raza r;
V(x) familia tuturor vecinatat
,
ilor punctului x R
n
;
int A mult
,
imea tuturor punctelor interioare ale lui A;
ext A mult
,
imea tuturor punctelor exterioare lui A;
cl A mult
,
imea tuturor punctelor aderente lui A;
bd A mult
,
imea tuturor punctelor frontiera pentru A;
A

mult
,
imea tuturor punctelor de acumulare pentru A;
[] matricea aplicat
,
iei liniare ;
norma aplicat
,
iei liniare ;
f

(x) derivata funct


,
iei de variabila reala f n punctul x;
f

(x; v) derivata funct


,
iei f n punctul x dupa direct
,
ia v;
df(x) diferent
,
iala funct
,
iei f n punctul x;
J(f)(x) matricea Jacobi a funct
,
iei f n punctul x;
f(x) gradientul funct
,
iei f n punctul x;
Notat
,
ii v
f

x
j
(x) =
f
x
j
(x) derivata part
,
iala n raport cu variabila x
j
a funct
,
iei
f n punctul x;
f

x
i
x
j
(x) =

2
f
x
j
x
i
(x) derivata part
,
iala de ordinul doi n raport cu
variabilele (x
i
, x
j
) a funct
,
iei f n punctul x;
d
2
f(x) diferent
,
iala a doua a funct
,
iei f n punctul x;
H(f)(x) matricea hessiana a funct
,
iei f n punctul x;
f
(k)
x
i
1
x
i
k
(x) =

k
f
x
i
k
x
i
1
(x) derivata part
,
iala de ordinul k n raport
cu variabilele (x
i
1
, . . . , x
i
k
) a funct
,
iei f n punctul x;
d
k
f(x) diferent
,
iala de ordinul k a funct
,
iei f n punctul x.
vi Notat
,
ii
Capitolul 1
Integrale improprii
1.1 Denit
,
ii s
,
i notat
,
ii
Conceptul de integrala Riemann a fost introdus pentru funct
,
ii denite pe un
interval compact [a, b]. Scopul teoriei integrarii poate extins considerand
s
,
i funct
,
ii denite pe intervale necompacte. Astfel de intervale e nu sunt
marginite, e nu sunt nchise. Ele pot clasicate astfel:
Tipul I: [a, b), [a, );
Tipul II: (a, b], (, b];
Tipul III: (a, b), (a, ), (, b), (, ).
1.1.1 Denit
,
ie (funct
,
ii local integrabile Riemann). Fie I R un interval
nedegenerat. O funct
,
ie f : I R se numes
,
te local integrabila Riemann (pe
I), daca ea este integrabila Riemann pe orice interval compact [u, v] I.
1.1.2 Denit
,
ie (integrale improprii pe intervale necompacte de tipul I). Fie
< a < b s
,
i e f : [a, b) R o funct ie local integrabila Riemann.
Daca exista limita lim
vb

v
a
f(x)dx R, atunci ea se numes
,
te integrala impro-
prie a funct
,
iei f pe [a, b) s
,
i va notata cu

b
a
f(x)dx. As
,
adar
(1)

b
a
f(x)dx = lim
vb

v
a
f(x)dx.
1
2 1 Integrale improprii
In exercit
,
ii se vor folosi urmatoarele notat
,
ii:

b
a
f(x)dx =
_

b0
a
f(x)dx daca b <


a
f(x)dx daca b = .
1.1.3 Denit
,
ie (integrale improprii pe intervale necompacte de tipul II). Fie
a < b < s
,
i e f : (a, b] R o funct ie local integrabila Riemann.
Daca exista limita lim
ua

b
u
f(x)dx R, atunci ea se numes
,
te integrala impro-
prie a funct
,
iei f pe (a, b] s
,
i va notata cu

b
a+
f(x)dx. As
,
adar
(2)

b
a+
f(x)dx = lim
ua

b
u
f(x)dx.
In exercit
,
ii se vor folosi urmatoarele notat
,
ii:

b
a+
f(x)dx =
_

b
a+0
f(x)dx daca a >

f(x)dx daca a = .
Daca limita din membrul drept n (1) este nita (respectiv ), atunci
se spune ca integrala improprie

b
a
f(x)dx este convergenta (respectiv diver-
genta). De asemenea, daca limita din membrul drept n (2) este nita (respec-
tiv ), atunci se spune ca integrala improprie

b
a+
f(x)dx este convergenta
(respectiv divergenta).
1.1.4 Denit
,
ie (integrale improprii pe intervale necompacte de tipul III).
Fie a < b s
,
i e f : (a, b) R o funct
,
ie local integrabila Riemann.
Se arata us
,
or ca daca exista un punct c (a, b) astfel ca integralele improprii

c
a+
f(x)dx s
,
i

b
c
f(x)dx sa existe n R, iar suma
(3)

c
a+
f(x)dx +

b
c
f(x)dx
1.1 Denit
,
ii si notat
,
ii 3
sa e denita n R, atunci pentru orice alt punct c

(a, b), integralele impro-


prii

a+
f(x)dx s
,
i

b
c

f(x)dx exista n R, iar suma

a+
f(x)dx +

b
c

f(x)dx
este denita n R. Mai mult, n acest caz are loc egalitatea

c
a+
f(x)dx +

b
c
f(x)dx =

a+
f(x)dx +

b
c

f(x)dx.
Aceasta observat
,
ie ne permite sa dam urmatoarea denit
,
ie: daca exista un
punct c (a, b) astfel ncat integralele improprii

c
a+
f(x)dx si

b
c
f(x)dx
sa existe n R, iar suma (3) sa e denita n R, atunci valoarea sumei (3) se
numes
,
te integrala improprie a funct
,
iei f pe (a, b) s
,
i va notata cu

b
a+
f(x)dx.
Daca suma (3) este nita (respectiv ), atunci se spune ca integrala impro-
prie

b
a+
f(x)dx este convergenta (respectiv divergenta). In exercit
,
ii se vor
folosi urmatoarele notat
,
ii:

b
a+
f(x)dx =
_

b0
a+0
f(x)dx daca a > s
,
i b <


a+0
f(x)dx daca a > s
,
i b =

b0

f(x)dx daca a = s
,
i b <

f(x)dx daca a = s
,
i b = .
1.1.5 Observat
,
ie. Fie a < b < s
,
i e f : (a, b] R o funct
,
ie local
integrabila Riemann. Avem

b
a+
f(x)dx = lim
ua

b
u
f(x)dx = lim
ua

b
u
f(t)dt = lim
ua

u
b
f(t)dt
= lim
va

v
b
f(x)dx =

(a)
b
f(x)dx,
4 1 Integrale improprii
deci orice integrala improprie de tipul II poate redusa la o integrala improprie
de tipul I. Avand n vedere aceasta observat
,
ie, precum s
,
i modul n care au
fost introduse integralele improprii de tipul III, rezulta ca este sucient sa
dezvoltam n continuare doar teoria integralelor improprii de tipul I, adica

b
a
f(x)dx cu < a < b .
1.2 Integrala Riemann vs. integrala improprie
1.2.1 Teorema. Fie a, b R cu a < b. Daca f : [a, b] R este o funct
,
ie
integrabila Riemann, atunci integrala improprie

b0
a
f(x)dx este convergenta
s
,
i are loc egalitatea

b0
a
f(x)dx =

b
a
f(x)dx.
Demonstrat
,
ie. Intrucat f este integrabila Riemann pe [a, b], ea este local in-
tegrabila Riemann. Mai mult, funct
,
ia F : [a, b] R, denita prin
F(v) :=

v
a
f(x)dx,
este continua. Drept urmare, exista limita lim
vb
F(v) s
,
i este egala cu F(b). Cu
alte cuvinte, integrala improprie

b0
a
f(x)dx este convergenta s
,
i

b
a
f(x)dx = lim
vb
F(v) = F(b) =

b
a
f(x)dx.
1.2.2 Teorema. Fie a, b R cu a < b s
,
i e f : [a, b) R o funct
,
ie marginita
local integrabila Riemann. Daca integrala improprie

b0
a
f(x)dx este conver-
genta, atunci orice prelungire f

: [a, b] R a lui f este integrabila Riemann


pe [a, b] s
,
i are loc egalitatea

b
a
f

(x)dx =

b0
a
f(x)dx.
1.2 Integrala Riemann vs. integrala improprie 5
Demonstrat
,
ie. Fie f

: [a, b] R o prelungire arbitrara a funct


,
iei f (adica
f

[a,b)
= f). Atunci f

este marginita, deoarece f este marginita. Notam

:= inf
x[a,b]
f

(x) s
,
i

:= sup
x[a,b]
f

(x).
Pentru a dovedi ca f

este integrabila Riemann vom folosi criteriul de inte-


grabilitate al lui Darboux. Fie > 0 oarecare s
,
i e
d := min
_
b a
2
,

1 + 3(

)
_
.
Intrucat f este integrabila Riemann pe [a, b d], din partea de necesitate a
criteriului lui Darboux rezulta existent
,
a unui

> 0 as
,
a ncat pentru orice
diviziune
0
a lui [a, b d] cu
0
<

sa avem
(1) S(f,
0
) s(f,
0
) <

3
.
Notam := min
_

,

1 + 3(

)
_
.
Fie apoi := (x
0
, x
1
, . . . , x
k
) o diviziune oarecare a lui [a, b] cu <
s
,
i e

j
:= inf
x[x
j1
,x
j
]
f

(x),

j
:= sup
x[x
j1
,x
j
]
f

(x), j = 1, . . . , k.
Sut posibile urmatoarele doua situat
,
ii.
Cazul I . b d {x
0
, x
1
, . . . , x
k
}.
Atunci exista un unic i {1, . . . , k 1} n as
,
a fel ncat x
i
= b d. Notand

0
:= (x
0
, x
1
, . . . , x
i
), avem
0
Div [a, b d] s
,
i
0
<

. Avem de
asemenea
S(f

, ) s(f

, ) =
i

j=1
(

j
)(x
j
x
j1
) +
k

j=i+1
(

j
)(x
j
x
j1
)
= S(f,
0
) s(f,
0
) +
k

j=i+1
(

j
)(x
j
x
j1
).
T
,
inand seama ca

oricare ar j {i +1, . . . , k}, n baza lui


(1) deducem ca
S(f

, ) s(f

, ) <

3
+ (

)
k

j=i+1
(x
j
x
j1
)
=

3
+ (

)d <

3
+

3
< .
6 1 Integrale improprii
Deci S(f

, ) s(f

, ) < .
Cazul II . b d {x
0
, x
1
, . . . , x
k
}.
Atunci exista un unic i {1, . . . , k} n as
,
a fel ncat x
i1
< b d < x
i
. Notand

0
:= (x
0
, . . . , x
i1
, b d), avem
0
Div [a, b d] s
,
i
0
<

. Avem
de asemenea
S(f

, ) s(f

, ) =
i1

j=1
(

j
)(x
j
x
j1
) + (

i
)(x
i
x
i1
)
+
k

j=i+1
(

j
)(x
j
x
j1
).
Dar
(

i
)(x
i
x
i1
) (

) (

)

1 + 3(

)
<

3
,
i1

j=1
(

j
)(x
j
x
j1
) S(f,
0
) s(f,
0
) <

3
,
k

j=i+1
(

j
)(x
j
x
j1
) (

)(x
k
x
i
) (

) d <

3
.
Deci s
,
i n acest caz are loc inegalitatea S(f

, ) s(f

, ) < .
Am dovedit as
,
adar ca pentru orice Div [a, b] cu < avem
S(f

, ) s(f

, ) < . Aplicand partea de sucient


,
a a criteriului lui Dar-
boux, deducem ca f

este integrabila Riemann. Pe de alta parte, avem s


,
i

b
a
f

(x)dx =

b0
a
f

(x)dx =

b0
a
f(x)dx,
conform teoremei 1.2.1.
1.2.3 Consecint
,
a. Fie a, b R cu a < b, e f : [a, b) R o funct ie con-
tinua care are limita nita n b s
,
i e f

: [a, b] R o prelungire a lui f.


Atunci integrala improprie

b0
a
f(x)dx este convergenta, f

este integrabila
Riemann pe [a, b] s
,
i are loc egalitatea

b0
a
f(x)dx =

b
a
f

(x)dx.
1.3 Calculul integralelor improprii 7
1.3 Calculul integralelor improprii
1.3.1 Teorema (liniaritatea integralei improprii). Fie < a < b
s
,
i e f, g : [a, b) R funct
,
ii local integrabile Riemann. Daca integralele
improprii

b
a
f(x)dx s
,
i

b
a
g(x)dx sunt ambele convergente, atunci pentru
orice , R integrala improprie

b
a
_
f(x) +g(x)
_
dx este de asemenea
convergenta s
,
i are loc egalitatea

b
a
_
f(x) +g(x)
_
dx =

b
a
f(x)dx +

b
a
g(x)dx.
Demonstrat
,
ie. Pentru orice v (a, b) avem

v
a
_
f(x) +g(x)
_
dx =

v
a
f(x)dx +

v
a
g(x)dx.
Cum
lim
vb

v
a
f(x)dx =

b
a
f(x)dx s
,
i lim
vb

v
a
g(x)dx =

b
a
g(x)dx,
rezulta ca exista
lim
vb

v
a
_
f(x) +g(x)
_
dx =

b
a
f(x)dx +

b
a
g(x)dx,
ceea ce demonstreaza armat
,
ia teoremei.
1.3.2 Teorema (formula lui LeibnizNewton). Fie < a < b , e
f : [a, b) R o funct
,
ie continua s
,
i e F : [a, b) R o primitiva a lui f. Daca
exista limita lim
vb
F(v) R, atunci are loc egalitatea

b
a
f(x)dx = lim
vb
F(v) F(a)
not.
= F

b
a
.
Demonstrat
,
ie. Pentru orice v (a, b) avem

v
a
f(x)dx = F(v) F(a),
formula lui LeibnizNewton pentru integrala Riemann. Egalitatea din enunt
,
rezulta facand v b.
8 1 Integrale improprii
1.3.3 Teorema (integrarea prin part
,
i n integrala improprie). Fie < a <
b s
,
i e f, g : [a, b) R funct
,
ii care ndeplinesc urmatoarele condit
,
ii:
(i) f s
,
i g sunt derivabile pe [a, b);
(ii) f

s
,
i g

sunt local integrabile Riemann pe [a, b);


(iii) limita lim
vb
f(v)g(v) exista s
,
i este nita.
Daca una dintre integralele improprii

b
a
f

(x)g(x)dx s
,
i

b
a
f(x)g

(x)dx
este convergenta, atunci s
,
i cealalta este convergenta s
,
i are loc egalitatea

b
a
f

(x)g(x)dx = fg

b
a

b
a
f(x)g

(x)dx,
unde fg

b
a
:= lim
vb
f(v)g(v) f(a)g(a).
Demonstrat
,
ie. Presupunem, pentru xarea ideilor, ca

b
a
f

(x)g(x)dx este
convergenta. Pentru orice v (a, b) avem
(1)

v
a
f

(x)g(x)dx = f(v)g(v) f(a)g(a)

v
a
f(x)g

(x)dx.
Intrucat limitele lim
vb

v
a
f

(x)g(x)dx s
,
i lim
vb
f(v)g(v) exista s
,
i sunt nite, rezul-
ta ca s
,
i lim
vb

v
a
f(x)g

(x)dx exista si este nita. Mai mult, facand v b n


(1), obt
,
inem egalitatea din concluzia teoremei.
1.3.4 Teorema (schimbarea de variabila n integrala improprie). Fie
a < b , < , e f : (a, b) R o funct
,
ie continua s
,
i e
: (, ) (a, b) o funct
,
ie bijectiva, derivabila s
,
i cu derivata continua. Daca
una dintre integralele improprii

b
a+
f(x)dx s
,
i


+
(f )(t) |

(t)| dt este
convergenta, atunci s
,
i cealalta este convergenta s
,
i are loc egalitatea
(2)

b
a+
f(x)dx =


+
(f )(t) |

(t)| dt.
1.3 Calculul integralelor improprii 9
Demonstrat
,
ie. Fiind injectiva s
,
i continua, funct
,
ia este strict monotona. Pre-
supunem, pentru xarea ideilor, ca este strict descrescatoare pe (, ). In-
trucat este surjectiva, trebuie sa avem
_

_
lim
s
(s) = b
lim
t
(t) = a
s
,
i
_

_
lim
ua

1
(u) =
lim
vb

1
(v) = .
Sa admitem ca

b
a+
f(x)dx este convergenta. Atunci exista un c (a, b)
as
,
a ncat

c
a+
f(x)dx s
,
i

b
c
f(x)dx sa e ambele convergente. Avem

b
a+
f(x)dx =

c
a+
f(x)dx +

b
c
f(x)dx
= lim
ua

c
u
f(x)dx + lim
vb

v
c
f(x)dx.
Fie u (a, c) s
,
i v (c, b) arbitrar alese. Cu schimbarea de variabila x = (t)
avem

c
u
f(x)dx =


1
(c)

1
(u)
f((t))

(t)dt
=


1
(u)

1
(c)
f((t))
_

(t)
_
dt
=


1
(u)

1
(c)
(f )(t) |

(t)|dt,
deoarece
1
(c) <
1
(u). Cum
1
este bijectiva, continua, strict des-
crescatoare s
,
i
1
(u) cand u a, avem
lim
v

1
(c)
(f )(t) |

(t)|dt = lim
ua


1
(u)

1
(c)
(f )(t) |

(t)|dt
= lim
ua

c
u
f(x)dx.
Drept urmare, integrala improprie

1
(c)
(f )(t) |

(t)|dt este convergenta


s
,
i

1
(c)
(f )(t) |

(t)|dt =

c
a+
f(x)dx.
10 1 Integrale improprii
Analog se arata ca s
,
i integrala improprie


1
(c)
+
(f )(t) |

(t)|dt este con-


vergenta s
,
i


1
(c)
+
(f )(t) |

(t)|dt =

b
c
f(x)dx.
In concluzie,


+
(f )(t) |

(t)|dt este convergenta s


,
i


+
(f )(t) |

(t)|dt
=


1
(c)
+
(f )(t) |

(t)|dt +

1
(c)
(f )(t) |

(t)|dt
=

b
c
f(x)dx +

c
a+
f(x)dx
=

b
a+
f(x)dx.
Sa admitem acum ca


+
(f )(t) |

(t)|dt este convergenta. Rat


,
io-
nand ca mai sus, se arata ca s
,
i

b
a+
f(x)dx este convergenta s
,
i ca egalitatea
(2) din enunt
,
are loc.
1.4 Criterii de convergent
,
a pentru integrale impro-
prii
1.4.1 Teorema (criteriul lui A. L. Cauchy). Fie < a < b s
,
i e
f : [a, b) R o funct
,
ie local integrabila Riemann. Atunci

b
a
f(x)dx este
convergenta daca s
,
i numai daca
> 0 c (a, b) a.. v, v

[c, b) :

v
f(x)dx

< .
Demonstrat
,
ie. Necesitatea. Presupunem ca

b
a
f(x)dx este convergenta. No-
tam I :=

b
a
f(x)dx. Fie > 0. Deoarece I = lim
vb

v
a
f(x)dx, rezulta ca
1.4 Criterii de convergent
,
a pentru integrale improprii 11
exista un c (a, b) as
,
a ncat
v [c, b) :

v
a
f(x)dx I

<

2
.
Atunci pentru orice v, v

[c, b) avem

v
f(x)dx

a
f(x)dx I +I

v
a
f(x)dx

a
f(x)dx I

v
a
f(x)dx

<

2
+

2
= .
Sucient
,
a. Pentru a dovedi ca

b
a
f(x)dx este convergenta, este sucient
sa aratam ca oricare ar s
,
irul (v
n
), de puncte din (a, b), cu proprietatea
lim
n
v
n
= b, s
,
irul
_
v
n
a
f(x)dx
_
este convergent, adica este fundamental. Fie,
n acest scop, > 0 arbitrar. Exista atunci un c (a, b) astfel ca pentru orice
v, v

[c, b) sa avem

v
f(x)dx

< . Intrucat (v
n
) b, exista un n
0
N cu
proprietatea ca v
n
[c, b) oricare ar n n
0
. Atunci pentru orice n, m n
0
avem

v
n
a
f(x)dx

v
m
a
f(x)dx

v
m
v
n
f(x)dx

< .
Prin urmare, s
,
irul
_
v
n
a
f(x)dx
_
este fundamental.
1.4.2 Teorema (criteriul comparat
,
iei). Fie < a < b si e funct
,
iile
local integrabile Riemann f, g : [a, b) [0, ). Atunci urmatoarele armat
,
ii
sunt adevarate:
1

Daca exista un c (a, b) as


,
a nc at pentru orice x [c, b) sa avem
f(x) g(x), atunci

b
a
g(x)dx convergenta

b
a
f(x)dx convergenta;

b
a
f(x)dx divergenta

b
a
g(x)dx divergenta.
12 1 Integrale improprii
2

Daca exista c (a, b) s


,
i exista numerele reale 0 < < as
,
a ncat
pentru orice x [c, b) sa avem g(x) > 0 s
,
i
f(x)
g(x)
, atunci integralele
improprii

b
a
f(x)dx s
,
i

b
a
g(x)dx au aceeas
,
i natura.
3

(criteriul comparat
,
iei sub forma de limita). Daca exista c (a, b) as
,
a
ncat g(x) > 0 pentru orice x [c, b) s
,
i exista lim
xb
f(x)
g(x)
(0, ), atunci
integralele improprii

b
a
f(x)dx si

b
a
g(x)dx au aceeas
,
i natura.
Demonstrat
,
ie. 1

Observam ca funct
,
iile F, G : [c, b) R, denite prin
F(v) :=

v
c
f(x)dx s
,
i respectiv G(v) :=

v
c
g(x)dx,
sunt crescatoare. Rezulta ca limitele
lim
vb
F(v) = lim
vb

v
c
f(x)dx s
,
i lim
vb
G(v) = lim
vb

v
c
g(x)dx
exista n [0, ].
Evident,

b
a
f(x)dx este convergenta (respectiv divergenta) daca s
,
i numai
daca

b
c
f(x)dx este convergenta (respectiv divergenta). Prin urmare,

b
a
f(x)dx convergenta lim
vb
F(v) <
(resp. divergenta) (resp. lim
vb
F(v) = ).
Analog, avem

b
a
g(x)dx convergenta lim
vb
G(v) <
(resp. divergenta) (resp. lim
vb
G(v) = ).
Deoarece f(x) g(x) oricare ar x [c, b), rezulta ca

v
c
f(x)dx

v
c
g(x)dx pentru orice v [c, b).
1.4 Criterii de convergent
,
a pentru integrale improprii 13
Drept urmare, avem
lim
vb

v
c
g(x)dx < lim
vb

v
c
f(x)dx < ;
lim
vb

v
c
f(x)dx = lim
vb

v
c
g(x)dx = ,
adica tocmai cele doua implicat
,
ii din enunt
,
.
2

Sa admitem mai ntai ca

b
a
f(x)dx este convergenta. Deoarece avem
g(x) f(x) oricare ar x [c, b), n baza lui 1

deducem ca s
,
i

b
a
g(x)dx
este convergenta, deci

b
a
g(x)dx este convergenta, ntrucat > 0.
Sa presupunem acum ca

b
a
g(x)dx este convergenta. Atunci s
,
i integrala
improprie

b
a
g(x)dx este convergenta. Cum f(x) g(x) oricare ar
x [c, b), n baza lui 1

deducem ca s
,
i

b
a
f(x)dx este convergenta.
As
,
adar,

b
a
f(x)dx este convergenta daca s
,
i numai daca

b
a
g(x)dx este
convergenta.
3

Daca := lim
xb
f(x)
g(x)
(0, ), atunci exista un c

(c, b) n as
,
a fel ncat

2

f(x)
g(x)
2 oricare ar x [c

, b).
Conform armat
,
iei 2

, integralele improprii

b
a
f(x)dx s
,
i

b
a
g(x)dx au
aceeas
,
i natura.
1.4.3 Consecint
,
a. Fie a, b R cu a < b, e f : [a, b) [0, ) o funct ie
continua s
,
i e p R as
,
a ncat sa existe := lim
xb
(b x)
p
f(x) R. Atunci
p < 1 s
,
i <

b0
a
f(x)dx < ;
p 1 s
,
i > 0

b0
a
f(x)dx = .
14 1 Integrale improprii
1.4.4 Consecint
,
a. Fie a, b R cu a < b, e f : (a, b] [0, ) o funct ie
continua s
,
i e p R as
,
a ncat sa existe := lim
xa
(x a)
p
f(x) R. Atunci
p < 1 s
,
i <

b
a+0
f(x)dx < ;
p 1 s
,
i > 0

b
a+0
f(x)dx = .
1.4.5 Consecint
,
a. Fie a R, e f : [a, ) [0, ) o funct
,
ie continua s
,
i
e p R as
,
a ncat sa existe := lim
x
x
p
f(x) R. Atunci
p > 1 s
,
i <


a
f(x)dx < ;
p 1 s
,
i > 0


a
f(x)dx = .
1.4.6 Denit
,
ie (integrale improprii absolut convergente). Fie < a <
b s
,
i e f : [a, b) R o funct
,
ie local integrabila Riemann. Conform
unui rezultat binecunoscut din teoria integralei Riemann, funct
,
ia |f| este de
asemenea local integrabila Riemann. Mai mult, limita lim
vb

v
a
|f(x)|dx exista
n [0, ]. Daca lim
vb

v
a
|f(x)|dx < , atunci se spune ca integrala improprie

b
a
f(x)dx este absolut convergenta.
1.4.7 Teorema. Fie < a < b s
,
i e f : [a, b) R o funct
,
ie
local integrabila Riemann. Daca integrala improprie

b
a
f(x)dx este absolut
converegenta, atunci ea este convergenta s
,
i are loc inegalitatea

b
a
f(x)dx

b
a
|f(x)|dx.
Demonstrat
,
ie. Fie > 0 arbitrar. Deoarece

b
a
|f(x)|dx este convergenta,
din partea de necesitate a criteriului lui Cauchy rezulta ca exista un c (a, b)
astfel ca pentru orice v, v

[c, b) cu v < v

sa avem

v
|f(x)|dx < . Dar
1.4 Criterii de convergent
,
a pentru integrale improprii 15

v
f(x)dx

v
|f(x)|dx. Drept urmare, avem

v
f(x)dx

< oricare ar v, v

[c, b) cu v < v

.
Aplicand acum partea de sucient
,
a a criteriului lui Cauchy, deducem ca inte-
grala improprie

b
a
f(x)dx este convergenta. Pe de alta parte, avem

v
a
f(x)dx

v
a
|f(x)|dx oricare ar v [a, b).
Facand v b, obt
,
inem inegalitatea din enunt
,
.
1.4.8 Denit
,
ie (integrale improprii semiconvergente). Reciproca teoremei
1.4.7 nu este, n general, adevarata:

b
a
f(x)dx convergenta

b
a
f(x)dx absolut convergenta.
O integrala improprie care este convergenta, dar nu este absolut convergenta,
se numes
,
te semiconvergenta.
1.4.9 Teorema (criteriul majorantei). Fie < a < b s
,
i e funct
,
iile
local integrabile Riemann f, g : [a, b) R. Daca exista c (a, b) astfel ca
|f(x)| g(x) oricare ar x [c, b)
s
,
i

b
a
g(x)dx este convergenta, atunci

b
a
f(x)dx este absolut convergenta,
deci convergenta.
Demonstrat
,
ie. Rezulta din teoremele 1.4.2 s
,
i 1.4.7.
1.4.10 Teorema (criteriul lui N. H. Abel). Fie < a < b s
,
i e
funct
,
iile f, g : [a, b) R, care ndeplinesc urmatoarele condit
,
ii:
(i) f este monotona s
,
i lim
xb
f(x) = 0;
(ii) g este continu a s
,
i sup
a<v<b

v
a
g(x)dx

< .
16 1 Integrale improprii
Atunci integrala improprie

b
a
f(x)g(x)dx este convergenta.
In demonstrat
,
ia criteriului lui Abel vom avea nevoie de urmatorul rezultat,
o varianta a celei de-a doua teoreme de medie a calculului integral, cunoscuta s
,
i
sub numele de formula de medie a lui Bonnet (dupa numele matematicianului
francez Ossian Bonnet care a descoperit-o n anul 1849).
1.4.11 Lema. Fie a, b R, a < b, e f : [a, b] [0, ) o funct
,
ie des-
crescatoare s
,
i e g : [a, b] R o funct
,
ie continua. Atunci exista un punct
c [a, b] astfel ncat

b
a
f(x)g(x)dx = f(a)

c
a
g(x)dx.
Demonstrat
,
ie. Fie G : [a, b] R funct
,
ia denita prin G(x) :=

x
a
g(t)dt.
Atunci G este continua pe [a, b]. Notam
m := min
x[a,b]
G(x) s
,
i respectiv M := max
x[a,b]
G(x).
Fie := (x
0
, x
1
, . . . , x
k
) o diviziune oarecare a lui [a, b]. Din prima teo-
rema de medie a calculului integral rezulta ca pentru ecare j {1, . . . , k}
exista un punct c
j
[x
j1
, x
j
] as
,
a ncat

x
j
x
j1
g(x)dx = g(c
j
)(x
j
x
j1
).
Notand := (c
1
, . . . , c
k
), avem P() s
,
i
(fg, , ) =
k

j=1
f(c
j
)g(c
j
)(x
j
x
j1
) =
k

j=1
f(c
j
)

x
j
x
j1
g(x)dx
=
k

j=1
f(c
j
)
_
G(x
j
) G(x
j1
)

= f(c
1
)
_
G(x
1
) G(x
0
)

+f(c
2
)
_
G(x
2
) G(x
1
)

+
+f(c
n1
_
G(x
n1
) G(x
n2
)

+f(c
n
)
_
G(x
n
) G(x
n1
)

.
T
,
inand seama ca G(x
0
) = G(a) = 0, obt
,
inem
(fg, , ) = G(x
0
)
_
f(a) f(c
1
)

+G(x
1
)
_
f(c
1
) f(c
2
)

+
+G(x
n1
)
_
f(c
n1
) f(c
n
)

+G(x
n
)f(c
n
).
1.4 Criterii de convergent
,
a pentru integrale improprii 17
Cum f este descrescatoare s
,
i nenegativa, deducem de aici ca
(fg, , ) M
_
f(a) f(c
1
)

+M
_
f(c
1
) f(c
2
)

+
+M
_
f(c
n1
) f(c
n
)

+Mf(c
n
),
adica (fg, , ) Mf(a). Analog se demonstreaza ca are loc s
,
i inegalitatea
(fg, , ) mf(a).
Am dovedit as
,
adar ca pentru orice Div [a, b] exista un P() cu
proprietatea ca
mf(a) (fg, , ) Mf(a).
Fie (
n
)) un s
,
ir de diviziuni ale lui [a, b] astfel ca lim
n

n
= 0 s
,
i pentru
ecare n N e
n
P(
n
) n as
,
a fel ncat
mf(a) (fg,
n
,
n
) Mf(a).
Facand n n acest lant
,
de inegalitat
,
i obt
,
inem
mf(a)

b
a
f(x)g(x)dx Mf(a).
Intrucat G este continua, deci are proprietatea lui Darboux, rezulta existent
,
a
unui punct c [a, b] pentru care

b
a
f(x)g(x)dx = f(a)G(c) = f(a)

c
a
g(x)dx.
Cu aceasta demonstrat
,
ia lemei este ncheiata.
Demonstrat
,
ia criteriului lui Abel. Presupunem ca f este descrescatoare (deci
nenegativa). In caz contrar, nlocuim funct
,
ia f cu f. Notam
M := sup
a<v<b

v
a
g(x)dx

.
Fie > 0 arbitrar. Deoarece f(x) 0 cand x b, exista un c (a, b) n as
,
a
fel ncat
0 f(v) <

2M + 1
pentru orice v [c, b).
Fie apoi v, v

[c, b), v < v

arbitrar alese. Din lema 1.4.11 rezulta existent


,
a
unui punct c [v, v

] cu proprietatea ca

v
f(x)g(x)dx = f(v)

c
v
g(x)dx,
18 1 Integrale improprii
deci

v
f(x)g(x)dx

= f(v)

c
a
g(x)dx

v
a
g(x)dx

2Mf(v) < .
In baza criteriului de convergent
,
a al lui Cauchy, deducem ca integrala impro-
prie

b
a
f(x)g(x)dx este convergenta.
1.5 Probleme
1. Sa se studieze daca integralele improprii de mai jos sunt convergente, iar
n caz armativ sa se determine valoarea lor.
1)

b0
a
1
(b x)
p
dx, a, b, p R, a < b;
2)

10
0

1 +x
1 x
dx;
3)

10
0
x
n

1 +x
1 x
dx, n N;
4)

10
0
ln(1 x)
(1 +x)
2
dx;
5)


a
1
x
p
dx, a > 0, p R;
6)


1
arctg x
x
2
dx;
7)


0
arctg x
(1 +x
2
)
3/2
dx;
8)


0
arctg x
x
2
+ 2bx + 1
dx, |b| < 1, (M. Ivan);
9)


1
dx
x

x
2n
+x
n
+ 1
, n N;
10)


0
dx
x
4
+ 1
;
11)


0
dx
(1 +x
2
)(1 +x

)
, > 0;
12)


0
e
ax
sin bxdx,


0
e
ax
cos bxdx, a, b R, a > 0;
1.5 Probleme 19
13)


0
e
ax
sin
n
xdx, a > 0, n N;
14)


0
e
x
2
dx, (EulerPoisson);
15)

b
a+0
1
(x a)
p
dx, a, b, p R, a < b;
16)

4
1+0
dx
x

2x
2
x 1
;
17)

2
1+0
dx
xln x
;
18)

1
0+0
(ln x)
n
dx, n N;
19)

1
0+0
ln
_
1 +x

1 x
_
dx;
20)

1
0+0
2
[
1
x
]
dx;
21)

1
0+0
[lnx]dx;
22)

1
0+0
x
_
1
x
_
dx;
23)

1
0+0
_
1
x
_
dx;
24)

1
0+0
_
1
x
_
2
dx;
A. F. S. Wong, Math. Mag. [2010, 150]
25)

1
0+0
x
_
1
x
_ _
1
x
_
dx;
26)

2
0+0
ln(sin x)dx,

0
0+0
ln(sin x)dx, (Euler);
27)

0
0+0
xln(sin x)dx;
28)

1
0+0
arcsin x
x
dx;
20 1 Integrale improprii
29)

b0
a+0
dx

(x a)(b x)
, a, b R, a < b;
30)

b0
a+0
x

(x a)(b x)
dx, a, b R, a < b;
31)

10
1+0
1
(ax +b)

1 x
2
dx, a, b R, |a| < |b|;
R:

b
2
a
2
32)


0
dx
4 + 3 cos x
;
33)

10
0+0
arcsin

x(1 x)
dx;
34)

10
0+0
ln x

1 x
dx;
35)

10
1+0
x
3

1 x
2
ln
1 +x
1 x
dx;
36)

10
0+0
ln x

1 x
2
dx;
37)


1+0
dx
(x
2
+ 1)

x
2
1
;
38)


0+0
ln x
x
2
+a
2
dx, a > 0;
39)


0+0
ln x
(x
2
+ 1)
2
dx;
40)


0+0
xln x
(x
2
+ 1)
3
dx;
41)


0+0
x
3
lnx
(x
4
+ 1)
3
dx;
42)


0+0
sin x
x
dx;
43)

x
2
x
4
+x
2
+ 1
dx.
2. (integralele lui Froullani) Fie a, b > 0 s
,
i f : (0, ) R o funct
,
ie con-
tinua. Sa se demonstreze ca:
1.5 Probleme 21
a) Daca exista s
,
i sunt nite limitele
lim
x0
f(x) =: f(0+) s
,
i lim
x
f(x) =: f(),
atunci are loc egalitatea


0+0
f(ax) f(bx)
x
dx = [f(0+) f()] ln
b
a
.
b) Daca exista s
,
i este nita limita lim
x0
f(x) =: f(0+) s
,
i exista c > 0 as
,
a
ncat integrala improprie


c
f(x)
x
dx este convergenta, atunci are loc
egalitatea


0+0
f(ax) f(bx)
x
dx = f(0+) ln
b
a
.
c) Daca exista s
,
i este nita limita lim
x
f(x) =: f() s
,
i exista c > 0 as
,
a
ncat integrala improprie

c
0+0
f(x)
x
dx este convergenta, atunci are loc
egalitatea


0+0
f(ax) f(bx)
x
dx = f() ln
b
a
.
3. Fiind date numerele reale a, b > 0, sa se calculeze urmatoarele integrale
improprii:
1)


0+0
e
ax
e
bx
x
dx;
2)


0+0
arctg (ax) arctg (bx)
x
dx;
3)


0+0
cos(ax) cos(bx)
x
dx;
4)


0+0
1
x
2
ln
(1 +ax)
b
(1 +bx)
a
dx.
4. Sa se studieze convergent
,
a urmatoarelor integrale improprii:
1)

10
0
dx
4

1 x
4
;
22 1 Integrale improprii
2)

10
0
dx

(1 x
2
)(1 k
2
x
2
)
, |k| < 1, (integralele eliptice de spe-
t
,
a ntai);
3)


1
dx
x

(1 +x
2
)

, , R;
4)


1
x

arctg x
1 +x

dx, , R;
5)


0
dx
1 +x
4
cos
2
x
;
6)


0
x
1 +x
2
cos
2
x
dx;
7)


0
sin(x
2
)dx,


0
cos(x
2
)dx, (integralele lui Fresnel);
8)


b
_

x +a

x b
_
dx, a, b > 0;
Concursul William Lowell Putnam 1995
9)

sin x
ln x
dx;
10)

1
0+0
dx
3

x(e
x
e
x
)
;
11)

4

4
+0
_
cos x sin x
cos x + sin x
_
p
dx, p > 0;
12)

1
0+0
_
x sin x
e
x
1
_

dx, R;
13)

10
0+0
x
a1
(1 x)
b1
dx, a, b R, (integralele lui Euler de spet
,
a
ntai);
14)


0+0
x
a1
e
x
dx, a R, (integralele lui Euler de spet
,
a a doua);
15)


0+0
| sin x|
x
dx;
16)


0+0
sin(xln x)dx;
17)


0+0
1
x
2
_
1
2

x
e
x
e
x
_
dx;
1.5 Probleme 23
18)


1+0
dx
x
_

ln x + ln
2
x
_.
5. Fie f : [0, ) [0, ) o funct
,
ie continua cu proprietatea ca


0
f(x)dx < .
a) Sa se demonstreze ca daca f este uniform continua, atunci f este
marginita.
b) Sa se demonstreze ca reciproca armat
,
iei a) nu este adevarata.
c) Sa se demonstreze ca daca se renunt
,
a la ipoteza de continuitate uni-
forma a lui f, atunci concluzia de la a) nu ramane, n general, adevarata.
Concursul Traian Lalescu 2011, faza nat
,
ionala
6. Fie f : [0, ) [0, ) o funct
,
ie descrescatoare cu proprietatea ca


0
f(x)dx < .
Sa se demonstreze ca lim
x
xf(x) = 0.
Berkeley 1983
7. Fiind dat p R, sa se demonstreze ca:
a) Daca f : (0, 1] R este o funct
,
ie monotona si integrala improprie

1
0+0
x
p
f(x)dx este convergenta, atunci lim
x0
x
p+1
f(x) = 0.
b) Daca f : [1, ) R este o funct
,
ie monotona s
,
i integrala improprie


1
x
p
f(x)dx este convergenta, atunci lim
x
x
p+1
f(x) = 0.
8. Fie f : (0, ) (0, ) funct
,
ia denita prin
f(x) = e
x
2
2


x
e

t
2
2
dt.
Sa se demonstreze ca:
a) pentru orice x > 0 are loc inegalitatea f(x) <
1
x
.
b) funct
,
ia f este strict descrescatoare.
Berkeley 1985
24 1 Integrale improprii
9. Fie f : (0, ) (0, ) o funct
,
ie strict crescatoare de clasa C
1
. Sa se
demonstreze ca daca


1
dx
f(x) +f

(x)
< , atunci s
,
i


1
dx
f(x)
< .
Olimpiada student
,
easca, U.R.S.S.
10. Fie numerele reale p > 1 s
,
i a > 0, iar f : [0, ) [a, ) o funct
,
ie
de clasa C
2
. Sa se demonstreze ca daca integrala


0
|f

(x)|dx este
convergenta, atunci s
,
i integrala


0
[f

(x)]
2
[f(x)]
p
dx este convergenta.
Olimpiada student
,
easca, U.R.S.S.
11. Sa se demonstreze ca daca f : (0, ) (0, ) este o funct
,
ie de clasa
C
1
, atunci


0+0

1 + (f

(x))
2
f(x)
dx = .
Olimpiada student
,
easca, U.R.S.S.
12. Fie f : [0, ) R o funct
,
ie descrescatoare cu proprietatea ca


0
f(x)dx = ,
iar g : [0, ) R o funct
,
ie local integrabila Riemann care pastreaza
semn constant pe [0, ). Sa se demonstreze ca


0
|f(x) cos x +g(x) sin x|dx =


0
|f(x) sin x +g(x) cos x|dx = .
W. F. Trench, Amer. Math. Monthly [1988, E3243]
13. Fie f : [0, 1) R o funct
,
ie monotona. Sa se demonstreze ca limitele
lim
x1

x
0
f(t)dt s
,
i lim
n
1
n
n1

k=0
f
_
k
n
_
exista s
,
i sunt egale.
M. Baluna, Olimpiada nat
,
ionala de matematica 1996
1.5 Probleme 25
14. Sa se demonstreze ca:
a) pentru ecare n N exista un unic polinom P
n
cu coecient
,
i reali,
de gradul n 1, n as
,
a fel ncat
x R : sin nx = sin xP
n
(cos x).
b) are loc egalitatea

1
0
P
2
n
(t)dt = 1 +
1
3
+
1
5
+ +
1
2n 1
.
I. Chit
,
escu, Olimpiada nat
,
ionala de matematica 1991
26 1 Integrale improprii
Capitolul 2
Topologie n R
n
2.1 Spat
,
iul euclidian R
n
2.1.1 Denit
,
ie (spat
,
iul liniar R
n
). Fiind dat n N, consideram mult
,
imea
R
n
:= { (x
1
, . . . , x
n
) | j {1, . . . , n} : x
j
R},
precum s
,
i operat
,
iile
+ : R
n
R
n
R
n
(x, y) R
n
R
n
x +y R
n
: R R
n
R
n
(, x) R R
n
x R
n
,
numite adunare s
,
i respectiv nmult
,
ire cu scalari , denite n felul urmator:
daca x := (x
1
, . . . , x
n
) R
n
, y := (y
1
, . . . , y
n
) R
n
s
,
i R, atunci punem
(1) x +y := (x
1
+y
1
, . . . , x
n
+y
n
)
s
,
i respectiv
(2) x := (x
1
, . . . , x
n
).
Se verica imediat ca aceste operat
,
ii se bucura de urmatoarele proprietat
,
i:
1

x, y, z R
n
: x + (y +z) = (x +y) +z;
2

x, y R
n
: x +y = y +x;
3

Exista un element 0
n
R
n
(numit originea lui R
n
) as
,
ancat x+0
n
= x
oricare ar x R
n
;
27
28 2 Topologie n R
n
4

Pentru orice x R
n
exista un element x R
n
(numit simetricul sau
opusul lui x) as
,
a ncat x + (x) = 0
n
;
5

x R
n
: 1 x = x;
6

, R x R
n
: ( x) = () x;
7

, R x R
n
: ( +) x = x + x;
8

R x, y R
n
: (x +y) = x + y.
Originea lui R
n
este unica s
,
i 0
n
= (0, . . . , 0). De asemenea, simetricul
oricarui element x R
n
este unic. Daca x = (x
1
, . . . , x
n
), atunci x =
(x
1
, . . . , x
n
).
O mult
,
ime nevida X, nzestrata cu doua operat ii
+ : X X X (x, y) X X x +y X
: R X X (, x) R X x X,
se numes
,
te spat
,
iu liniar peste corpul R al numerelor reale (sau spat
,
iu liniar
real ) daca sunt ndeplinite urmatoarele condit
,
ii:
(SL1) x, y, z X : x + (y +z) = (x +y) +z;
(SL2) x, y X : x +y = y +x;
(SL3) Exista un element 0
X
X (numit originea lui X) astfel ncat
x + 0
X
= x oricare ar x X;
(SL4) Pentru orice x X exista un element x X (numit simetricul
sau opusul lui x) as
,
a ncat x + (x) = 0
X
;
(SL5) x X : 1 x = x;
(SL6) , R x X : ( x) = () x;
(SL7) , R x X : ( +) x = x + x;
(SL8) R x, y X : (x +y) = x + y.
Din proprietat
,
ile 1

rezulta ca mult
,
imea R
n
, nzestrata cu adunarea s
,
i
nmult irea cu scalari denite prin (1) s
,
i (2), este un spat
,
iu liniar real.
In continuare, elementele lui R
n
vor numite vectori sau puncte, iar ele-
mentele lui R vor numite scalari . Produsul x, dintre scalarul R s
,
i
vectorul x R
n
, se va nota simplu cu x (adica punctul va omis).
2.1 Spat
,
iul euclidian R
n
29
2.1.2 Denit
,
ie (baza canonica a lui R
n
). Consideram vectorii
e
1
:= (1, 0, 0, . . . , 0) R
n
,
e
2
:= (0, 1, 0, . . . , 0) R
n
,
.
.
.
e
n
:= (0, 0, 0, . . . , 1) R
n
.
Mult
,
imea {e
1
, e
2
, . . . , e
n
} este o baza algebrica a spat
,
iului liniar real R
n
, nu-
mita baza canonica sau baza standard a lui R
n
. Orice vector x = (x
1
, . . . , x
n
)
din R
n
se reprezinta cu ajutorul vectorilor din baza canonica sub forma
x = x
1
e
1
+ +x
n
e
n
.
2.1.3 Denit
,
ie (produsul scalar n R
n
). Fie x = (x
1
, . . . , x
n
), y = (y
1
, . . . , y
n
)
doi vectori din R
n
. Produsul scalar dintre x s
,
i y este numarul real denit prin
(3) x, y := x
1
y
1
+ +x
n
y
n
.
Se constata imediat ca produsul scalar are urmatoarele proprietat
,
i:
1

x, y, z R
n
: x +y, z = x, z +y, z;
2

R x, y R
n
: x, y = x, y;
3

x, y R
n
: x, y = y, x;
4

x R
n
\ {0
n
} : x, x > 0.
Din (3) rezulta ca x, 0
n
= 0
n
, x = 0 s
,
i ca x, x 0 oricare ar x R
n
.
Fiind dat un spat
,
iu liniar real X, o funct
,
ie , : X X R se numes
,
te
produs scalar pe X, daca ndeplines
,
te urmatoarele condit
,
ii:
(PS1) x, y, z X : x +y, z = x, z +y, z;
(PS2) R x, y X : x, y = x, y;
(PS3) x, y X : x, y = y, x;
(PS4) x X \ {0
X
} : x, x > 0.
Perechea ordonata (X, , ) se numes
,
te spat
,
iu cu produs scalar sau spat
,
iu
prehilbertian real.
Din proprietat
,
ile 1

rezulta ca (R
n
, , ), unde , este produsul
scalar denit prin egalitatea (3), este un spat
,
iu prehilbertian real, numit spat
,
iul
euclidian R
n
.
30 2 Topologie n R
n
2.1.4 Teorema (inegalitatea lui CauchyBuniakovskiSchwarz). Pentru orice
vectori x, y R
n
are loc inegalitatea
|x, y|

x, x

y, y.
Demonstrat
,
ie. Se va face la seminar.
2.1.5 Denit
,
ie (norma euclidianan R
n
). Funct
,
ia : R
n
[0, ), denita
prin
(4) x :=

x, x =

x
2
1
+ +x
2
n
oricare ar x = (x
1
, . . . , x
n
) R
n
,
se numes
,
te norma euclidiana n R
n
. Este usor de vericat ca norma euclidiana
are urmatoarele proprietat
,
i:
1

x = 0 x = 0
n
;
2

R x R
n
: x = ||x;
3

x, y R
n
: x +y x +y.
Inegalitatea de la 3

se numes
,
te inegalitatea triunghiului s
,
i ea este o consecint
,
a
a inegalitat ii lui CauchyBuniakovskiSchwarz.
Fiind dat un spat
,
iu liniar real X, o funct
,
ie : X [0, ) se numes
,
te
norma pe X, daca ndeplines
,
te urmatoarele condit
,
ii:
(N1) x = 0 x = 0
X
;
(N2) R x X : x = ||x;
(N3) x, y X : x +y x +y.
Perechea ordonata (X, ) se numes
,
te spat
,
iu normat real .
Din proprietat
,
ile 1

rezulta ca (R
n
, ), unde este norma eucli-
diana denita prin (4), este un spat
,
iu normat real.
2.1.6 Denit
,
ie (distant
,
a euclidiana n R
n
). Funct
,
ia d : R
n
R
n
[0, ),
denita pentru orice puncte x = (x
1
, . . . , x
n
) R
n
s
,
i y = (y
1
, . . . , y
n
) R
n
prin
(5) d(x, y) := x y =

(x
1
y
1
)
2
+ + (x
n
y
n
)
2
,
se numes
,
te distant
,
a sau metrica euclidiana n R
n
. Proprietat
,
ile normei eucli-
diene garanteaza ca
2.2 Structura topologica a spat
,
iului R
n
31
1

d(x, y) = 0 x = y;
2

x, y R
n
: d(x, y) = d(y, x);
3

x, y, z R
n
: d(x, z) d(x, y) +d(y, z).
Fiind data o mult
,
ime nevida X, o funct
,
ie d : X X [0, ) se numes
,
te
metrica sau distant
,
a pe X, daca ndeplines
,
te urmatoarele condit
,
ii:
(M1) d(x, y) = 0 x = y;
(M2) x, y X : d(x, y) = d(y, x);
(M3) x, y, z X : d(x, z) d(x, y) +d(y, z).
Perechea ordonata (X, d) se numes
,
te spat
,
iu metric.
Din proprietat
,
ile 1

rezulta ca (R
n
, d), unde d este distant
,
a euclidiana
denita prin (5), este un spat
,
iu metric.
2.2 Structura topologica a spat
,
iului R
n
2.2.1 Denit
,
ie (bile). Fiind date a R
n
s
,
i r > 0, notam
B(a, r) := {x R
n
| d(x, a) < r} = {x R
n
| x a < r},

B(a, r) := {x R
n
| d(x, a) r} = {x R
n
| x a r}.
Aceste mult
,
imi se numesc bila deschisa s
,
i respectiv bila nchisa cu centrul n
a s
,
i de raza r. Evident, avem a B(a, r)

B(a, r).
2.2.2 Denit
,
ie (vecinatat
,
i). Fie x un punct arbitrar din R
n
. O mult
,
ime
V R
n
se numes
,
te vecinatate a lui x daca exista r > 0 astfel ca B(x, r) V .
Familia tuturor vecinatat
,
ilor lui x va notata cu V(x).
2.2.3 Denit
,
ie. Fie A o submult
,
ime a lui R
n
s
,
i e x R
n
. Se spune ca x
este
punct interior al lui A daca A V(x), adica daca exista r > 0 astfel ca
B(x, r) A;
punct exterior lui A daca R
n
\ A V(x), adica daca exista r > 0 astfel
ca B(x, r) R
n
\ A;
punct aderent lui A daca V A = oricare ar V V(x);
32 2 Topologie n R
n
punct frontiera pentru A daca V A = s
,
i V (R
n
\ A) = oricare
ar V V(x);
punct de acumulare pentru A daca (V A) \ {x} = oricare ar
V V(x);
punct izolat al lui A daca exista V V(x) astfel ca V A = {x}.
Mult
,
imea tuturor punctelor interioare ale lui A se numes
,
te interiorul lui
A s
,
i va notata cu int A.
Mult
,
imea tuturor punctelor exterioare lui A se numes
,
te exteriorul lui A s
,
i
va notata cu ext A.
Mult
,
imea tuturor punctelor aderente lui A se numes
,
te nchiderea sau
aderent
,
a lui A s
,
i va notata cu cl A.
Mult
,
imea tuturor punctelor frontiera pentru A se numes
,
te frontiera lui A
s
,
i va notata cu bd A.
Mult
,
imea tuturor punctelor de acumulare pentru A se numes
,
te derivata
mult
,
imii A s
,
i va notata cu A

.
2.2.4 Teorema. Pentru orice mult
,
ime A R
n
au loc urmatoarele relat
,
ii:
1

int A A cl A;
2

ext A = int
_
R
n
\ A
_
;
3

cl A = R
n
\ int
_
R
n
\ A
_
;
4

bd A = (cl A) cl
_
R
n
\ A
_
;
5

(int A) (bd A) = cl A;
6

(int A) (bd A) (ext A) = R


n
;
7

(int A) (bd A) = ;
8

(int A) (ext A) = ;
9

(ext A) (bd A) = ;
10

cl A = A A

.
Demonstrat
,
ie. Demonstram doar relat
,
ia 10

. Aratam mai ntai ca


(1) cl A A A

.
2.2 Structura topologica a spat
,
iului R
n
33
Fie n acest scop x cl A arbitrar. Daca x A, atunci x A A

. Daca
x A, sa dovedim ca x A

. Fie V o vecinatate oarecare a lui x. Cum


x cl A, avem V A = . Intrucat x A, deducem ca (V A) \ {x} = .
Drept urmare avem x A

, deci x A A

. As
,
adar, incluziunea (1) are
loc. t
,
inand seama de relat
,
ia 1

, precum s
,
i de incluziunea evidenta A

cl A,
deducem ca
(2) A A

cl A.
Din (1) s
,
i (2) rezulta ca relat
,
ia 10

are loc.
2.2.5 Denit
,
ie (mult
,
imi deschise s
,
i mult
,
imi nchise). O mult
,
ime A R
n
se
numes
,
te:
deschisa daca A V(x) oricare ar x A, adica daca pentru orice
x A exista un r > 0 as
,
a ncat B(x, r) A;
nchisa daca mult
,
imea R
n
\ A este deschisa.
2.2.6 Exemplu. Daca a R
n
s
,
i r > 0, atunci bila deschisa B(a, r) este o
mult
,
ime deschisa, iar bila nchisa

B(a, r) este o mult
,
ime nchisa. In adevar,
pentru orice x B(a, r) avem x a < r. Notand r

:= r x a > 0,
se arata us
,
or ca B(x, r

) B(a, r), deci B(a, r) este mult


,
ime deschisa. De
asemenea, pentru orice punct x R
n
\

B(a, r) avem x a > r. Notand
r

:= x a r > 0, se arata us
,
or ca B(x, r

) R
n
\

B(a, r). Prin urmare,
mult
,
imea R
n
\

B(a, r) este deschisa, deci

B(a, r) este o mult
,
ime nchisa.
2.2.7 Propozit
,
ie. Daca A este o submult
,
ime a lui R
n
, atunci int A este o
mult
,
ime deschisa, iar cl A este o mult
,
ime nchisa.
Demonstrat
,
ie. Fie x int A. Atunci exista un r > 0 astfel ncat B(x, r) A.
Dovedim ca B(x, r) int A. Fie n acest scop y B(x, r). Atunci avem
y x < r. Notand r

:= r y x > 0, are loc incluziunea


B(y, r

) B(x, r) A,
deci y int A. Prin urmare, exista pentru ecare x int A un r > 0 astfel ca
B(x, r) int A. Aceasta nseamna ca int A este mult
,
ime deschisa.
Conform relat
,
iei 3

din teorema 2.2.4 s


,
i a celor stabilite mai sus, mult
,
imea
R
n
\ cl A = int
_
R
n
\ A
_
este deschisa, deci mult imea cl A este nchisa.
2.2.8 Teorema (caracterizarea mult
,
imilor deschise). Fiind data o mult
,
ime
A R
n
, urmatoarele armat
,
ii sunt echivalente:
34 2 Topologie n R
n
1

A este deschisa.
2

Are loc egalitatea int A = A.


3

Are loc egalitatea A bd A = .


Demonstrat
,
ie. 1

Conform teoremei 2.2.4, avem int A A. Pe de alta


parte, deoarece A este deschisa, pentru ecare x A exista un r > 0 astfel
ncat B(x, r) A. Deci ecare punct x A este interior lui A. In consecint
,
a,
avem int A = A.
2

Daca int A = A, atunci n baza relat


,
iei 7

din teorema 2.2.4 avem


A bd A = (int A) (bd A) = .
3

Presupunem ca A bd A = , dar A nu este deschisa. Exista


atunci un punct x
0
A cu proprietatea ca B(x
0
, r) A oricare ar r > 0.
Rezulta de aici ca B(x
0
, r)
_
R
n
\A
_
= oricare ar r > 0. Pe de alta parte,
mai avem s
,
i x
0
B(x
0
, r) A oricare ar r > 0. In consecint
,
a, trebuie sa
avem x
0
bd A, deci x
0
A bd A, ceea ce contrazice presupunerea facuta.
Contradict
,
ia obt
,
inuta arata ca A este deschisa.
2.2.9 Teorema (caracterizarea mult
,
imilor nchise). Fiind data o mult
,
ime
A R
n
, urmatoarele armat
,
ii sunt echivalente:
1

A este nchisa.
2

Are loc egalitatea cl A = A.


3

Are loc incluziunea bd A A.


4

Are loc incluziunea A

A.
Demonstrat
,
ie. 1

Daca A este nchisa, atunci R


n
\ A este deschisa, deci
int
_
R
n
\ A
_
= R
n
\ A, conform teoremei 2.2.8. Folosind acum armat
,
ia 3

din
teorema 2.2.4 deducem ca
cl A = R
n
\ int
_
R
n
\ A
_
= R
n
\
_
R
n
\ A
_
= A.
2

Daca A = cl A, atunci mult


,
imea A este nchisa, conform propozi-
t
,
iei 2.2.7.
2

Din 2

s
,
i armat
,
ia 5

a teoremei 2.2.4 rezulta ca


A = cl A = (int A) (bd A),
2.2 Structura topologica a spat
,
iului R
n
35
deci bd A A.
3

Din 3

s
,
i armat
,
iile 1

s
,
i 5

ale teoremei 2.2.4 rezulta ca


cl A = (int A) (bd A) A cl A,
deci cl A = A.
2

Din 2

s
,
i armat
,
ia 10

a teoremei 2.2.4 rezulta ca A = cl A =


A A

, deci A

A.
4

Din 4

s
,
i armat
,
iile 1

s
,
i 10

ale teoremei 2.2.4 rezulta ca


cl A = A A

A cl A,
deci cl A = A.
2.2.10 Teorema. Familia T , a tuturor submult
,
imilor deschise ale lui R
n
, se
bucura de urmatoarele proprietat
,
i:
1

T s
,
i R
n
T .
2

Oricare ar familia (G
i
)
iI
, de mult
,
imi din T , avem
iI
G
i
T .
3

Pentru orice mult


,
imi G
1
, G
2
T , avem G
1
G
2
T .
Demonstrat
,
ie. Imediata.
2.2.11 Observat
,
ii. a) Fiind data o mult
,
ime arbitrara X, se numes
,
te topologie
pe X orice familie T P(X), de submult
,
imi ale lui X, care ndeplines
,
te
urmatoarele condit
,
ii:
(T1) T s
,
i X T ;
(T2) Oricare ar familia (G
i
)
iI
, de mult
,
imi din T , avem
iI
G
i
T ;
(T3) Pentru orice mult
,
imi G
1
, G
2
T , avem G
1
G
2
T .
Perechea ordonata (X, T ) se numes
,
te spat
,
iu topologic.
Din teorema 2.2.10 rezulta ca familia T , a tuturor submult
,
imilor deschise
ale lui R
n
, este o topologie pe R
n
, numita topologia euclidiana.
b) Familia T

, a tuturor submult
,
imilor nchise ale lui R
n
, se bucura de
urmatoarele proprietat
,
i:
1

s
,
i R
n
T

;
36 2 Topologie n R
n
2

Oricare ar familia (F
i
)
iI
, de mult
,
imi din T

, avem
iI
F
i
T

;
3

Pentru orice mult


,
imi F
1
, F
2
T

, avem F
1
F
2
T

.
c) Intersect
,
ia unei familii oarecare de mult
,
imi din T nu apart
,
ine, n general,
lui T . De exemplu, mult imea G
k
:=
_

1
k
,
1
k
_
este deschisa n R pentru ecare
k N, dar

k=1
G
k
= {0} este nchisa n R.
De asemenea, reuniunea unei familii oarecare de mult
,
imi din T

nu apart
,
i-
ne, n general, lui T

. De exemplu, mult
,
imea F
k
:=
_
1 +
1
2k
, 1
1
2k

este
nchisa n R pentru ecare k N, dar

k=1
F
k
= (1, 1) este deschisa n R.
2.3 S
,
iruri de puncte n R
n
2.3.1 Denit
,
ie (s
,
iruri convergente n R
n
). Orice funct
,
ie f : N R
n
se
numes
,
te s
,
ir de puncte din R
n
. Daca f(k) = x
k
R
n
pentru k N, atunci
s
,
irul va notat (x
k
)
kN
sau, simplu, (x
k
). Fiecare termen x
k
al s
,
irului ind
un punct din R
n
, este de forma x
k
= (x
k1
, x
k2
, . . . , x
kn
).
Fie (x
k
) un s
,
ir de puncte din R
n
s
,
i e x R
n
. Se spune ca (x
k
) converge
catre x (sau ca x este o limita a s
,
irului (x
k
)) daca
> 0 k
0
N a.. k k
0
: x
k
x
0
< .
Se constata imediat ca un s
,
ir de puncte din R
n
are cel mult o limita. In
cazul n care aceasta exista, s
,
irul se numes
,
te convergent. Faptul ca s
,
irul (x
k
)
converge catre x va notat prin (x
k
) x sau lim
k
x
k
= x.
2.3.2 Teorema. Fie (x
k
) un s
,
ir de puncte din R
n
s
,
i e x R
n
. Atunci
(x
k
) x lim
k
x
k
x = 0.
Demonstrat
,
ie. Evidenta.
2.3.3 Teorema. Fie (x
k
) s
,
i (y
k
) s
,
iruri convergente de puncte din R
n
, e
x := lim
k
x
k
, y := lim
k
y
k
, e (
k
) un s
,
ir convergent de numere reale
s
,
i e := lim
k

k
. Atunci
lim
k
(x
k
+y
k
) = x +y s
,
i lim
k
(
k
x
k
) = x.
Demonstrat
,
ie. Imediata.
2.3 S
,
iruri de puncte n R
n
37
2.3.4 Teorema. Fie (x
k
) un s
,
ir de puncte din R
n
, x
k
= (x
k1
, . . . , x
kn
) (k N)
s
,
i e x = ( x
1
, . . . , x
n
) R
n
. Atunci
lim
k
x
k
= x j {1, . . . , n} : lim
k
x
kj
= x
j
.
Demonstrat
,
ie. Necesitatea. Presupunem ca lim
k
x
k
= x. Conform teore-
mei 2.3.2 avem atunci lim
k
x
k
x = 0. Deoarece
x
k
x =

(x
k1
x
1
)
2
+ (x
k2
x
2
)
2
+ + (x
kn
x
n
)
2
|x
kj
x
j
|
pentru ecare j {1, . . . , n}, rezulta ca lim
k
|x
kj
x
j
| = 0, adica
lim
k
x
kj
= x
j
pentru orice j {1, . . . , n}.
Sucient
,
a. Admitem acum ca lim
k
x
kj
= x
j
, adica
lim
k
|x
kj
x
j
| = 0 pentru orice j {1, . . . , n}.
Deoarece
x
k
x =

_
n

j=1
(x
kj
x
j
)
2

j=1
|x
kj
x
j
|,
deducem ca lim
k
x
k
x = 0, deci lim
k
x
k
= x, conform teoremei
2.3.2.
2.3.5 Denit
,
ie (s
,
iruri fundamentale n R
n
). Un s
,
ir (x
k
), de puncte din R
n
se numes
,
te fundamental (sau sir Cauchy) daca
> 0 k
0
N a.. k, k
0
: x
k
x

< .
2.3.6 Teorema. Fie (x
k
) un s
,
ir din R
n
, x
k
= (x
k1
, . . . , x
kn
) (k N). Atunci
urmatoarele armat
,
ii sunt echivalente:
1

S
,
irul (x
k
) este fundamental.
2

Pentru ecare j {1, . . . , n}, s


,
irul de numere reale (x
kj
) este funda-
mental.
Demonstrat
,
ie. 1

Presupunem ca (x
k
) este fundamental. Fie j
{1, . . . , n} xat. Pentru a dovedi ca s
,
irul (x
kj
) este fundamental, e > 0.
Exista atunci un k
0
N as
,
a ncat pentru orice k, k
0
sa avem x
k
x

< .
Deoarece
x
k
x

(x
k1
x
1
)
2
+ + (x
kn
x
n
)
2
|x
kj
x
j
|,
38 2 Topologie n R
n
deducem ca |x
kj
x
j
| < pentru orice k, k
0
. Drept urmare, s
,
irul de
numere reale (x
kj
) este fundamental.
2

Fie > 0 oarecare. Pentru ecare j {1, . . . , n} exista un k


j
N
astfel ca
|x
kj
x
j
| <

n
pentru orice k, k
j
.
Notand k
0
:= max{k
1
, . . . , k
n
}, avem
x
k
x

_
n

j=1
(x
kj
x
j
)
2

j=1
|x
kj
x
j
| < ,
oricare ar k, k
0
. In consecint
,
a, s
,
irul (x
k
) este fundamental.
2.3.7 Teorema (A. L. Cauchy). Un s
,
ir de puncte din R
n
este convergent
daca s
,
i numai daca el este fundamental.
Demonstrat
,
ie. Fie (x
k
) un s
,
ir din R
n
, x
k
= (x
k1
, . . . , x
kn
) (k N). Avem
(x
k
) convergent j {1, . . . , n} s
,
irul (x
kj
) este convergent
j {1, . . . , n} s
,
irul (x
kj
) este fundamental
(x
k
) fundamental,
cele trei echivalent
,
e ind justicate de teorema 2.3.4, teorema lui Cauchy
pentru s
,
iruri de numere reale s
,
i respectiv teorema 2.3.6.
2.3.8 Teorema (caracterizarea secvent
,
iala a punctelor aderente). Fie A o
submult
,
ime a lui R
n
. Un punct x R
n
este aderent lui A daca s
,
i numai daca
exista un s
,
ir (x
k
), de puncte din A, care converge catre x.
Demonstrat
,
ie. Necesitatea. Daca x cl A, atunci pentru orice k N avem
B
_
x,
1
k
_
A = . Pentru ecare k N alegem x
k
B
_
x,
1
k
_
A. Atunci (x
k
)
este un s
,
ir de puncte din A cu proprietatea x
k
x <
1
k
oricare ar k N.
In baza teoremei 2.3.2 rezulta ca (x
k
) x.
Sucient
,
a. Admitem ca exista un s
,
ir (x
k
), de puncte din A, care converge
catre x. Fie V o vecinatate arbitrara a lui x. Exista atunci r > 0 n as
,
a fel
ncat B(x, r) V . Cum (x
k
) x, exista un k
0
N astfel ca x
k
x < r
oricare ar k k
0
. Atunci pentru orice k k
0
avem x
k
B(x, r) V s
,
i
x
k
A, deci V A = . Intrucat V a fost arbitrara, deducem ca x cl A.
2.4 Mult
,
imi compacte n R
n
39
2.3.9 Consecint
,
a (caracterizarea secvent
,
iala a punctelor de acumulare). Fie
A o submult
,
ime a lui R
n
. Un punct x R
n
este punct de acumulare al lui A
daca s
,
i numai daca exista un s
,
ir (x
k
), de puncte din A \ {x}, care converge
catre x.
Demonstrat
,
ie. Rezulta din teorema 2.3.8, t
,
inand seama ca x A

daca s
,
i
numai daca x cl
_
A\ {x}
_
.
2.3.10 Consecint
,
a (caracterizarea secvent
,
iala a mult
,
imilor nchise). O mul-
t
,
ime A R
n
este nchisa daca s
,
i numai daca pentru orice s
,
ir convergent de
puncte din A, limita sa apart ine lui A.
Demonstrat
,
ie. Se aplica teorema 2.3.8 s
,
i teorema 2.2.9.
2.4 Mult
,
imi compacte n R
n
2.4.1 Denit
,
ie (mult
,
imi compacte). Fie A R
n
. Se numes
,
te acoperire
a mult
,
imii A orice familie (A
i
)
iI
, de submult
,
imi ale lui R
n
, pentru care
A

iI
A
i
. Acoperirea (A
i
)
iI
se numes
,
te deschisa daca toate mult
,
imile
A
i
(i I) sunt deschise.
O mult
,
ime A R
n
se numes
,
te compacta daca din orice acoperire deschisa
a sa se poate extrage o subacoperire nita, adica daca pentru orice acoperire
deschisa (A
i
)
iI
a lui A exista o submult
,
ime nita J a lui I cu proprietatea
ca A

iJ
A
i
.
Evident, orice mult
,
ime nita A R
n
este compacta.
2.4.2 Exemplu. O mult
,
ime A R
n
se numes
,
te hipercub nchis daca este de
forma A = [a
1
, b
1
] [a
2
, b
2
] [a
n
, b
n
], unde a
1
, b
1
, a
2
, b
2
, . . . , a
n
, b
n
R,
a
1
< b
1
, a
2
< b
2
, . . ., a
n
< b
n
s
,
i b
1
a
1
= b
2
a
2
= = b
n
a
n
.
Vom dovedi n cele ce urmeaza ca orice hipercub nchis din R
n
este o
mult
,
ime compacta. Facem demonstrat
,
ia doar n cazul particular n = 2
(demonstrat
,
ia pentru n arbitrar este, n esent
,
a, aceeas
,
i, ind nsa put
,
in mai
dicil de redactat). Fie as
,
adar A = [a
1
, b
1
] [a
2
, b
2
] s
,
i e = b
1
a
1
= b
2
a
2
.
Presupunem prin absurd ca mult
,
imea A nu este compacta, adica exista o
acoperire deschisa (G
i
)
iI
a lui A, din care nu se poate extrage nici o suba-
coperire nita. Cu ajutorul centrului sau, mpart
,
im patratul An 2
2
patrate:
_
a
1
,
a
1
+b
1
2
_

_
a
2
,
a
2
+b
2
2
_
,
_
a
1
+b
1
2
, b
1
_

_
a
2
,
a
2
+b
2
2
_
,
_
a
1
,
a
1
+b
1
2
_

_
a
2
+b
2
2
, b
2
_
,
_
a
1
+b
1
2
, b
1
_

_
a
2
+b
2
2
, b
2
_
.
40 2 Topologie n R
n
Cel put
,
in unul dintre aceste 2
2
patrate nu poate acoperit cu un numar nit
de mult
,
imi din familia (G
i
)
iI
. Fie A
1
= [a
11
, b
11
] [a
21
, b
21
] un asemenea
patrat, unde
a
1
a
11
< b
11
b
1
, a
2
a
21
< b
21
b
2
, b
11
a
11
= b
21
a
21
=

2
.
Cu patratul A
1
se procedeaza ca s
,
i cu A. Se descompune A
1
cu ajutorul
centrului sau n 2
2
patrate. Cel put
,
in unul dintre aceste patrate nu poate
acoperit cu un numar nit de mult
,
imi din familia (G
i
)
iI
. Fie A
2
= [a
12
, b
12
]
[a
22
, b
22
] un asemenea patrat, unde
a
11
a
12
< b
12
b
11
, a
21
a
21
< b
21
b
22
, b
12
a
12
= b
22
a
22
=

2
2
.
Continuand rat
,
ionamentul, obt
,
inem inductiv un s
,
ir A
k
= [a
1k
, b
1k
] [a
2k
, b
2k
]
de patrate cu urmatoarele proprietat
,
i:
a) A
k
A oricare ar k N;
b) niciunul dintre patratele A
k
(k N) nu poate acoperit cu un numar
nit de mult
,
imi din familia (G
i
)
iI
;
c) s
,
irurile (a
1k
) s
,
i (a
2k
) sunt crescatoare, s
,
irurile (b
1k
) s
,
i (b
2k
) sunt des-
crescatoare s
,
i
b
1k
a
1k
= b
2k
a
2k
=

2
k
oricare ar k N.
Exista as
,
adar x

1
, x

2
R cu proprietatea
{x

1
} =

k=1
[a
1k
, b
1k
] s
,
i {x

2
} =

k=1
[a
2k
, b
2k
].
Notand x

:= (x

1
, x

2
), avem atunci x

k=1
A
k
A. Cum (G
i
)
iI
este
o acoperire a lui A, exista un i
0
I astfel ca x

G
i
0
. Deoarece G
i
0
este
deschisa, exista r > 0 as
,
a ncat B(x

, r) G
i
0
. Alegem un k N n asa fel
ncat

2/2
k
< r. Pentru orice punct x = (x
1
, x
2
) A
k
avem
|x
1
x

1
| b
1k
a
1k
=

2
k
si |x
2
x

2
| b
2k
a
2k
=

2
k
.
Urmeaza de aici ca
x x

(x
1
x

1
)
2
+ (x
2
x

2
)
2

2
2
k
< r,
2.4 Mult
,
imi compacte n R
n
41
deci A
k
B(x

, r) G
i
0
. Prin urmare, patratul A
k
poate acoperit cu o
mult
,
ime din familia (G
i
)
iI
, n contradict
,
ie cu construct
,
ia sirului de patrate
(A
k
). Contradict
,
ia obt
,
inuta arata ca mult
,
imea A este compacta.
2.4.3 Denit
,
ie (mult
,
imi marginite). O mult
,
ime A R
n
se numes
,
te margi-
nita daca exista a R
n
s
,
i r > 0 astfel ca A

B(a, r). Evident, toate bilele
nchise s
,
i toate bilele deschise din R
n
sunt mult
,
imi marginite. De asemenea,
orice submult ime a unei mult
,
imi marginite din R
n
este marginita.
2.4.4 Teorema (caracterizarea mult
,
imilor compacte). Fiind data o mult
,
ime
A R
n
, urmatoarele armat
,
ii sunt echivalente:
1

A este compacta.
2

Orice submult
,
ime innita a lui A are cel put in un punct de acumulare
care apart
,
ine lui A.
3

A este secvent
,
ial compacta, adica orice s
,
ir de puncte din A are un subs
,
ir
convergent catre un punct din A.
4

A este marginita s
,
i nchisa.
Demonstrat
,
ie. 1

Admitem ca A este compacta. Presupunem prin ab-


surd ca exista o mult
,
ime innita A
0
A care nu are niciun punct de acumulare
apart
,
inand lui A. Aceasta nseamna ca pentru ecare x A exista un r
x
> 0
ncat B(x, r
x
) A
0
\ {x} = , deci
B(x, r
x
) A
0
{x} oricare ar x A.
Familia
_
B(x, r
x
)
_
xA
este o acoperire deschisa a lui A. Intrucat A este com-
pacta, exista x
1
, . . . , x
m
An as
,
a fel ncat A
m

j=1
B(x
j
, r
x
j
). Atunci avem
A
0
= A A
0

m

j=1
_
B(x
j
, r
x
j
) A
0
_
{x
1
, . . . , x
m
},
ceea ce este absurd, caci A
0
este innita.
2

Fie (x
k
) un s
,
ir arbitrar de puncte din A s
,
i e A
0
mult
,
imea
termenilor s
,
irului, A
0
:= {x
k
| k = 1, 2, . . .}. Daca mult
,
imea A
0
este nita,
atunci cel put
,
in unul dintre termenii sirului se repeta de o innitate de ori. Prin
urmare, n acest caz s
,
irul (x
k
) poseda un subs
,
ir convergent catre un punct din
A s
,
i anume un subs
,
ir constant. Presupunemn continuare ca A
0
este innita.
Conform ipotezei 2

, mult
,
imea A
0
poseda cel put in un punct de acumulare
x A. Atunci orice vecinatate a lui x cont
,
ine o innitate de termeni ai s
,
irului
(x
k
). Construim un subs
,
ir (x
k
j
) al s
,
irului (x
k
) n felul urmator:
42 2 Topologie n R
n
alegem k
1
N astfel ca x
k
1
B(x, 1);
de ndata ce k
j
N a fost construit deja, alegem k
j+1
> k
j
n as
,
a fel
ncat x
k
j+1
B
_
x,
1
j+1
_
.
Cum x
k
j
x < 1/j oricare ar j N, rezulta n baza teoremei 2.3.2 ca
(x
k
j
) x.
3

Presupunand ca A nu este marginita, avem A



B(0
n
, k) oricare
ar k N. Prin urmare, pentru ecare k N exista un punct x
k
A cu
x
k
> k. Conform ipotezei 3

, sirul (x
k
) are un subs
,
ir (x
k
j
), convergent catre
un punct x A. Avem atunci lim
j
x
k
j
x = 0, conform teoremei 2.3.2.
Pe de alta parte, avem
x
k
j
x x
k
j
x > k
j
x pentru orice j N.
Cum lim
j
k
j
= , deducem ca lim
j
x
k
j
x = , ceea ce este absurd.
Contradict
,
ia obt inuta arata ca A este marginita.
Pentru a dovedi ca A este nchisa, folosim consecint
,
a 2.3.10. Fie (x
k
) un s
,
ir
convergent arbitrar de puncte din A s
,
i e x R
n
limita sa. Conform ipotezei
3

, s
,
irul (x
k
) are un subs
,
ir convergent catre un punct din A. Limita acestui
subs
,
ir ind x, rezulta ca x A. Prin urmare, A este nchisa.
4

Admitem ca A este marginita s


,
i nchisa. Exista atunci un r > 0
n as
,
a fel ncat A [r, r]
n
. Pentru a dovedi ca A este compacta, e (G
i
)
iI
o
acoperire deschisa a lui A. Atunci (G
i
)
iI
{R
n
\A} va o acoperire deschisa
a hipercubului nchis [r, r]
n
. Acesta ind o mult
,
ime compacta (a se vedea
exemplul 2.4.2), exista o mult
,
ime nita J I astfel ca
[r, r]
n

_
R
n
\ A
_

iJ
G
i
_
.
Intrucat A [r, r]
n
, rezulta ca A

iJ
G
i
, adica (G
i
)
iJ
este o sub-
acoperire nita a acoperirii (G
i
)
iI
. In consecint
,
a, mult
,
imea A este com-
pacta.
2.4.5 Observat
,
ii. a) Echivalent
,
a 1

din teorema 2.4.4 este cunoscuta


n literatura de specialitate sub numele de ,, teorema de compactitate metrica
a lui Hausdor. Ea este valabila nu doar n R
n
ci n orice spat
,
iu metric.
b) Echivalent
,
a 1

din teorema 2.4.4 este cunoscuta n literatura


matematica sub numele de ,, teorema lui BorelLebesgue . Ea este valabila n
R
n
precum si n orice spat
,
iu normat nit dimensional, dar nu s
,
i ntr-un spat
,
iu
metric arbitrar.
2.5 Probleme 43
2.5 Probleme
1. Fie a = (3, 2, 4) R
3
s
,
i b = (8, 6, 3) R
3
. Sa se determine a + b,
a b, 3a +b, a, b, a, b, d(a, b).
2. Sa se demonstreze inegalitatea lui Cauchy-Buneakovski-Schwarz:
x, y R
n
: |x, y|

x, x

y, y.
3. Sa se demonstreze identitatea paralelogramului:
x, y R
n
: x +y
2
+x y
2
= 2x
2
+ 2y
2
.
4. Fie m N s
,
i e x
1
, . . . , x
m
R
n
astfel ca
x
i
= 1 oricare ar i {1, . . . , m}
s
,
i
x
i
x
j
= 1 oricare ar i, j {1, . . . , m}, i = j.
Sa se demonstreze ca mult
,
imea {x
1
, . . . , x
m
} este liniar independenta.
Concursul Traian Lalescu, UBB, etapa locala, 2013
5. Fie {i, j, k} baza canonica a spat
,
iului R
3
.
a) Sa se demonstreze ca daca a, b, c R
3
satisfac inegalitatea
a||
2
+b
2
+c
2
< 1,
atunci {i +a, j +b, k +c} este baza n R
3
.
b) Sa se demonstreze ca daca a, b, c R
3
satisfac inegalitatea
cos(a, i) + cos(b, j) + cos(c, k) >
5
2
,
atunci {a, b, c} este baza n R
3
.
Concursul Traian Lalescu, UPB, etapa locala, 2010
6. Fie H o matrice de tipul n n cu elemente din mult
,
imea {1, 1}, ale
carei linii sunt ortogonale doua cate doua. Presupunem ca exista n H
o submatrice de tipul a b cu toate elementele 1. Sa se demonstreze ca
ab n.
Concursul William Lowell Putnam 2005
44 2 Topologie n R
n
7. Fiind date numerele reale p, q > 1, cu proprietatea
1
p
+
1
q
= 1, sa se
demonstreze ca:
1

Are loc inegalitatea lui Young


a, b [0, [ : ab
a
p
p
+
b
q
q
.
2

Are loc inegalitatea lui Holder


a
1
, . . . , a
n
, b
1
, . . . , b
n
[0, [ :
n

k=1
a
k
b
k

_
n

k=1
a
p
k
_1
p
_
n

k=1
b
q
k
_1
q
.
8. Fiind dat numarul real p 1, sa se demonstreze ca are loc inegalitatea
lui Minkowski
_
n

k=1
(a
k
+b
k
)
p
_1
p

_
n

k=1
a
p
k
_1
p
+
_
n

k=1
b
p
k
_1
p
oricare ar a
1
, . . . , a
n
, b
1
, . . . , b
n
[0, ).
9. Fiind dat numarul real p 1, sa se arate ca funct
,
ia
p
: R
n
[0, ),
denita prin
x
p
:=
_
n

k=1
|x
k
|
p
_1
p
oricare ar x = (x
1
, . . . , x
n
) R
n
,
este o norma, numita p-norma pe R
n
.
10. Fie X un spat
,
iu liniar real s
,
i : X [0, ) o norma pe X. Se spune
ca norma provine dintr-un produs scalar daca exista un produs
scalar , pe X, cu proprietatea x =

x, x pentru orice x X. Sa
se demonstreze ca p-norma
p
pe R
n
, unde p 1 s
,
i n 2, provine
dintr-un produs scalar daca s
,
i numai daca p = 2.
11. Sa se demonstreze ca funct
,
ia

: R
n
[0, ), denita prin
x

:= max { |x
1
|, . . . , |x
n
| } oricare ar x = (x
1
, . . . , x
n
) R
n
,
este o norma, numita norma Cebs
,
ev pe R
n
. Sa se arate ca norma
Cebs
,
ev nu provine dintr-un produs scalar.
2.5 Probleme 45
12. Sa se demonstreze ca pentru orice x R
n
are loc egalitatea
lim
p
x
p
= x

.
13. Fie A R
nn
s
,
i e : R
n
R
n
aplicat
,
ia liniara avand matricea A n
baza canonica a lui R
n
. Sa se demonstreze ca daca
(x)

= x

oricare ar x R
n
,
atunci exista un numar natural m astfel ca A
m
= I
n
.
SEEMOUS 2007
14. Sa se demonstreze ca pentru orice a R
n
s
,
i orice r > 0 are loc egalitatea
cl B(a, r) =

B(a, r).
15. Sa se demonstreze ca pentru orice mult
,
ime A R
n
sunt adevarate
urmatoarele relat
,
ii:
1

int A A cl A;
2

ext A = int (R
n
\ A);
3

cl A = R
n
\ int (R
n
\ A);
4

bd A = (cl A) cl (R
n
\ A);
5

(int A) (bd A) = cl A;
6

(int A) (bd A) (ext A) = R


n
;
7

(int A) (bd A) = ;
8

(int A) (ext A) = ;
9

(ext A) (bd A) = ;
10

cl A = A A

.
16. Sa se demonstreze ca pentru orice mult
,
imi A, B R
n
au loc urmatoarele
incluziuni:
int (A\ B) (int A) \ (int B) s
,
i (cl A) \ (cl B) cl (A\ B).
Sa se dea exemple de mult
,
imi A, B R
n
pentru care incluziunile de mai
sus sunt stricte.
46 2 Topologie n R
n
17. Fiind date mult
,
imile A, B R
n
, sa se demonstreze ca:
a) Daca A B = R
n
, atunci
_
cl A
_

_
int B
_
= R
n
.
b) Daca A B = , atunci
_
cl A
_

_
int B
_
= .
18. Sa se demonstreze ca pentru orice mult
,
imi A
1
, A
2
R
n
are loc egalitatea
cl(A
1
A
2
) = (cl A
1
) (cl A
2
).
Este adevarat ca pentru orice familie (A
i
)
iI
de submult
,
imi ale lui R
n
are loc egalitatea
cl
_

iI
A
i
_
=

iI
cl A
i
?
19. Sa se demonstreze ca pentru orice mult
,
imi A
1
, A
2
R
n
are loc egalitatea
(A
1
A
2
)

= A

1
A

2
.
Este adevarat ca pentru orice familie (A
i
)
iI
de submult
,
imi ale lui R
n
are loc egalitatea
_

iI
A
i
_

iI
A

i
?
20. Fie (x
k
) s
,
i (y
k
) s
,
iruri convergente de puncte din R
n
s
,
i e x := lim
k
x
k
,
y := lim
k
y
k
.
a) Sa se demonstreze ca lim
k
d(x
k
, y
k
) = d(x, y).
b) Sa se deduca apoi ca lim
k
x
k
= x.
21. Sa se determine
lim
k
_
_
_
_

2 +

2 + +

2 +

3
. .
k radicali
,
k

j=1
b(j)
j(j + 1)
_
_
_
_
,
unde b(j) este numarul cifrelor 1 din reprezentarea binara a lui j (de
exemplu, b(6) = b(110
2
) = 2, b(8) = b(1000
2
) = 1).
2.5 Probleme 47
22. Fie f : R R o funct
,
ie aditiva, adica o funct
,
ie cu proprietatea
x, y R : f(x +y) = f(x) +f(y)
s
,
i e
gr(f) := { (x, f(x)) R
2
| x R}
gracul lui f. Sa se demonstreze ca:
a) Daca f este continua n cel put
,
in un punct, atunci f este continua pe
R.
b) Daca f este continua n cel put
,
in un punct (deci pe R), atunci f(x) =
cx oricare ar x R, unde c = f(1).
c) Daca f este discontinua pe R, atunci cl gr(f) = R
2
, adica gr(f) este
o submult
,
ime densa a lui R
2
.
23. Fie (x
k
) un s
,
ir convergent de puncte din R
n
s
,
i x := lim
k
x
k
. Sa se demon-
streze ca mult
,
imea A := {x} { x
k
| k = 1, 2, . . . } este compacta.
24. Fiind date mult
,
imile A, B R
n
, notam
A+B := { x R
n
| a A s
,
i b B : x = a +b }.
a) Sa se demonstreze ca daca una dintre mult
,
imile A s
,
i B este nchisa,
iar cealalta este compacta, atunci mult
,
imea A+B este nchisa.
b) Dat
,
i exemplu de mult
,
imi nchise A s
,
i B pentru care mult
,
imea A+B
nu este nchisa.
25. Fie A s
,
i B submult
,
imi nevide ale lui R
n
s
,
i e
d(A, B) := inf { d(x, y) | x A, y B}.
a) Sa se demonstreze ca daca A = {a} s
,
i B este nchisa, atunci exista
un b B as
,
a ncat d(A, B) = d(a, b).
b) Sa se demonstreze ca daca A este compacta s
,
i B este nchisa, atunci
exista a A s
,
i exista b B as
,
a ncat d(A, B) = d(a, b).
c) Aratat
,
i printr-un contraexemplu ca armat
,
ia de la b) nu ramane
adevaratan cazul cand A s
,
i B sunt ambelenchise dar niciuna compacta.
Berkeley 1978
48 2 Topologie n R
n
26. Fiind data mult
,
imea A R
n
s
,
i numarul real r > 0, notam
B := { x R
n
| a A : x a = r }.
a) Sa se reprezinte grac mult
,
imea B n cazul n care r = 1, iar A R
2
este cercul cu centrul n (0, 0) s
,
i de raza 2, respectiv segmentul care
unes
,
te punctele (0, 0) s
,
i (1, 1).
b) Sa se demonstreze ca daca A este nchisa, atunci s
,
i B este nchisa.
Berkeley 1989
27. (lema numarului Lebesgue) Fiind data o mult
,
ime marginita A R
n
,
numarul real, denit prin
sup { d(x, y) | x, y A},
se numes
,
te diametrul mult
,
imii A. Sa se demonstreze ca daca A este
compacta, iar (G
i
)
iI
este o acoperire deschisa a lui A, atunci exista un
numar real > 0 cu urmatoarea proprietate: pentru orice submult
,
ime
B a lui A, avand diametrul cel mult , exista un i I astfel ca B G
i
.
(Orice numar cu aceasta proprietate se numes
,
te numar Lebesgue al
acoperirii (G
i
)
iI
.)
Berkeley 1986, 1994, 1996
2.6 Limite ale funct
,
iilor vectoriale de variabila
vectoriala
2.6.1 Denit
,
ie. O funct
,
ie f : A R
m
, unde A este o submult
,
ime nevida a
lui R, se numes
,
te funct
,
ie vectoriala de variabila reala.
O funct
,
ie f : A R, unde A este o submult
,
ime nevida a lui R
n
, se numes
,
te
funct
,
ie reala de variabila vectoriala.
O funct
,
ie f : A R
m
, unde A este o submult
,
ime nevida a lui R
n
, se
numes
,
te funct
,
ie vectoriala de variabila vectoriala.
Fie A o submult
,
ime nevida a lui R
n
s
,
i f : A R
m
o funct
,
ie vectoriala. Fie
apoi f
1
, . . . , f
m
: A R funct
,
iile reale denite n felul urmator: daca x A
s
,
i f(x) = (y
1
, . . . , y
m
) R
m
, atunci f
i
(x) := y
i
oricare ar i {1, . . . , m}.
Funct
,
iile f
1
, . . . , f
m
se numesc componentele scalare ale lui f. Se vede us
,
or ca
f(x) = (f
1
(x), . . . , f
m
(x)) pentru orice x A.
In continuare vom folosi notat
,
ia f = (f
1
, . . . , f
m
) : A R
m
atunci cand dorim
sa punem n evident
,
a componentele scalare ale lui f.
2.6 Limite ale funct iilor vectoriale de variabila vectoriala 49
2.6.2 Denit
,
ie (limita unei funct
,
ii ntr-un punct). Fie A R
n
, a A

,
f : A R
m
o funct
,
ie s
,
i b R
m
. Se spune ca b este o limita a lui f n punctul
a (sau ca f are limita b n punctul a) daca
> 0 > 0 a.. x A\ {a} cu x a < : f(x) b < .
Se vede imediat ca f are cel mult o limita n a. Faptul ca b este limita lui f
n punctul a se va nota prin lim
xa
f(x) = b.
2.6.3 Teorema (caracterizarea secvent
,
iala a limitei). Fie A o submult
,
ime a
lui R
n
, a A

, f : A R
m
o funct
,
ie s
,
i b R
m
. Atunci lim
xa
f(x) = b daca s
,
i
numai daca pentru orice s
,
ir (x
k
) de puncte din A \ {a}, care converge catre
a, avem lim
k
f(x
k
) = b.
Demonstrat
,
ie. Necesitatea. Presupunem ca lim
xa
f(x) = b. Fie (x
k
) un
s
,
ir oarecare de puncte din A \ {a}, astfel ca (x
k
) a. Pentru a dovedi ca
(f(x
k
)) b, e > 0 arbitrar ales. Exista atunci un > 0 n as
,
a fel ncat
pentru orice x A\{a}, cu xa < , sa avemf(x)b < . Cum (x
k
) a,
exista un k
0
N astfel ncat pentru orice k k
0
sa avem x
k
a < . Drept
urmare, avem f(x
k
) b < oricare ar k k
0
. Deci (f(x
k
)) b.
Sucient
,
a. Presupunand ca b nu este limita lui f n punctul a, rezulta ca
exista un > 0 cu proprietatea ca pentru orice > 0 exista cel put
,
in un punct
x A \ {a} astfel ca x a < s
,
i f(x) b . In particular, pentru
= 1/k, rezulta ca pentru ecare k N exista un punct x
k
A \ {a} n as
,
a
fel ncat x
k
a < 1/k s
,
i f(x
k
) b . Atunci (x
k
) este un s
,
ir de puncte
din A \ {a} s
,
i cum lim
k
x
k
a = 0, avem (x
k
) a. Conform ipotezei
noastre, trebuie sa avem lim
k
f(x
k
) = b, deci lim
k
f(x
k
) b = 0. Dar
aceasta egalitate este n contradict
,
ie cu faptul ca f(x
k
) b pentru orice
k N. Contradict
,
ia obt
,
inuta arata ca lim
xa
f(x) = b.
2.6.4 Teorema. Fie A R
n
, a A

, b, c R
m
, R s
,
i e f, g : A R
m
funct
,
ii cu proprietatea lim
xa
f(x) = b, lim
xa
g(x) = c. Atunci
lim
xa
_
f(x) +g(x)
_
= b +c s
,
i lim
xa
f(x) = b.
Demonstrat
,
ie. Rezulta din teoremele 2.3.3 s
,
i 2.6.3.
2.6.5 Teorema. Fie A R
n
, a A

, e b R, c R
m
si e funct
,
iile
f : A R, g : A R
m
cu proprietatea lim
xa
f(x) = b s
,
i lim
xa
g(x) = c. Atunci
lim
xa
f(x)g(x) = bc.
50 2 Topologie n R
n
Demonstrat
,
ie. Rezulta din teoremele 2.3.3 s
,
i 2.6.3.
2.6.6 Observat
,
ie. In general,
lim
xa
g(x) = b
lim
yb
f(y) = c
_

_
lim
xa
(f g)(x) = c.
Contraexemplu: e f, g : R R funct
,
iile denite prin
g(x) :=
_
1 daca x = 0
2 daca x = 0
s
,
i f(y) :=
_
2 daca y = 1
3 daca y = 1.
Atunci
(f g)(x) = f(g(x)) =
_
3 daca x = 0
2 daca x = 0.
Avem lim
x0
g(x) = 1, lim
y1
f(y) = 2, dar lim
x0
(f g)(x) = 3 = 2.
2.6.7 Teorema. Fie A R
n
, B R
m
, a A

, b B

, c R
p
s
,
i e
g : A B s
,
i f : B R
p
funct
,
ii care ndeplinesc urmatoarele condit
,
ii:
(i) lim
xa
g(x) = b s
,
i lim
yb
f(y) = c;
(ii) exista r > 0 astfel ncat pentru orice x A \ {a} cu x a < r sa
avem g(x) = b.
Atunci lim
xa
(f g)(x) = c.
Demonstrat
,
ie. Fie (x
k
) un s
,
ir arbitrar de puncte din A\ {a}, cu proprietatea
ca (x
k
) a. Notam y
k
:= g(x
k
) (k N). Atunci teorema 2.6.3 garanteaza ca
(y
k
) b. Cum (x
k
) converge catre a, exista un k
0
N astfel ca x
k
a < r
oricare ar k k
0
. Prin urmare, avem y
k
= g(x
k
) = b oricare ar k k
0
.
As
,
adar, (y
k
)
kk
0
este un s
,
ir din B \ {b}, care converge catre b. Aplicand
din nou teorema 2.6.3, deducem ca lim
k
f(y
k
) = c, adica lim
k
(f g)(x
k
) = c.
Intrucat s
,
irul (x
k
) a fost arbitrar, din partea de sucient
,
a a teoremei 2.6.3
rezulta ca lim
xa
(f g)(x) = c.
2.6.8 Teorema. Fie A R
n
, a A

, f : A R
m
o funct
,
ie s
,
i b R
m
.
Atunci lim
xa
f(x) = b daca s
,
i numai daca lim
xa
f(x) b = 0.
Demonstrat
,
ie. Imediata (se aplica denit
,
ia 2.6.2 funct
,
iilor f s
,
i g : A R,
g(x) := f(x) b).
2.7 Continuitatea funct iilor vectoriale de variabila vectoriala 51
2.6.9 Teorema. Fie A R
n
, a A

, f = (f
1
, . . . , f
m
) : A R
m
o funct
,
ie s
,
i
e b := (b
1
, . . . , b
m
) R
m
. Atunci
lim
xa
f(x) = b i {1, . . . , m} : lim
xa
f
i
(x) = b
i
.
Demonstrat
,
ie. Se aplica teoremele 2.6.3 s
,
i 2.3.4.
2.7 Continuitatea funct
,
iilor vectoriale de variabila
vectoriala
2.7.1 Denit
,
ie. Fie A R
n
s
,
i e a A. O funct
,
ie f : A R
m
se numes
,
te
continua n punctul a daca
> 0 > 0 a.. x A cu x a < : f(x) f(a) < .
2.7.2 Observat
,
ie. O funct
,
ie f : A R
m
este continua n toate punctele
izolate ale lui A. In adevar, e a un punct izolat al lui A s
,
i e > 0. Exista
o vecinatate V a lui a astfel ca V A = {a}. Alegem > 0 n as
,
a fel ncat
B(a, ) V . Atunci B(a, )A = {a}, deci singurul punct x A care satisface
x a < este x = a. Evident, pentru x = a inegalitatea f(x) f(a) <
are loc. In consecint
,
a, f este continua n a.
2.7.3 Teorema (caracterizarea secvent
,
iala a continuitat ii). Fie A R
n
s
,
i
e a A. O funct
,
ie f : A R
m
este continua n punctul a daca s
,
i nu-
mai daca pentru orice s
,
ir (x
k
) de puncte din A, care converge catre a, avem
lim
k
f(x
k
) = f(a).
Demonstrat
,
ie. Este asemanatoare cu demonstrat
,
ia teoremei 2.6.3.
2.7.4 Teorema (continuitate vs. limita). Fie A R
n
s
,
i e a A A

. O
funct
,
ie f : A R
m
este continua n a daca s
,
i numai daca lim
xa
f(x) = f(a).
Demonstrat
,
ie. Necesitatea. Rezulta din teoremele 2.6.3 s
,
i 2.7.3.
Sucient
,
a. Fie > 0 arbitrar. Cum lim
xa
f(x) = f(a), exista un > 0
astfel ncat pentru orice x A\{a} cu xa < sa avem f(x) f(a) < .
Dar aceasta inegalitate este evident adevarata s
,
i pentru x = a. In consecint
,
a,
pentru orice x A cu xa < avem f(x) f(a) < . Aceasta nseamna
ca f este continua n a.
2.7.5 Teorema. Fie A R
n
, a A, f, g : A R
m
funct
,
ii continue n a s
,
i
e R. Atunci funct
,
iile f s
,
i f +g sunt continue n a.
52 2 Topologie n R
n
Demonstrat
,
ie. Se aplica teoremele 2.7.3 s
,
i 2.3.3.
2.7.6 Teorema. Fie A R
n
, a A, s
,
i e f : A R, g : A R
m
funct
,
ii
continue n a. Atunci s
,
i funct ia fg este continua n a.
Demonstrat
,
ie. Se aplica teoremele 2.7.3 s
,
i 2.3.3.
2.7.7 Teorema. Fie A R
n
, a A, B R
m
, g : A B o funct ie continua
n a s
,
i f : B R
p
o funct
,
ie continua n punctul g(a). Atunci funct
,
ia f g
este continua n a.
Demonstrat
,
ie. Fie (x
k
) un s
,
ir oarecare de puncte din A, care converge catre
a. Cum g este continua n a, n baza teoremei 2.7.3 avem (g(x
k
)) g(a) =: b.
Deoarece f este continua n b, tot n baza teoremei 2.7.3 deducem ca
lim
k
f(g(x
k
)) = f(b) lim
k
(f g)(x
k
)) = (f g)(a).
Intrucat s
,
irul (x
k
) a fost arbitrar, partea de sucient a a teoremei 2.7.3 asigura
ca f g este continua n a.
2.7.8 Teorema. Fie A R
n
, a A s
,
i f = (f
1
, . . . , f
m
) : A R
m
o funct
,
ie.
Atunci f este continu a n punctul a daca s
,
i numai daca f
i
este continua n a
pentru ecare i {1, . . . , m}.
Demonstrat
,
ie. Se aplica teoremele 2.7.3 s
,
i 2.3.4.
2.7.9 Teorema. Daca A R
n
este o mult
,
ime compacta, iar f : A R
m
este o funct
,
ie continua, atunci mult
,
imea f(A) este compacta.
Demonstrat
,
ie. Fie (y
k
) un s
,
ir arbitrar de puncte din f(A). Pentru ecare
k N exista un x
k
A astfel ca y
k
= f(x
k
). Intrucat A este compacta,
conform implicat
,
iei 1

din teorema 2.4.4, s


,
irul (x
k
) poseda un subsir (x
k
j
)
convergent catre un punct x A. Continuitatea lui f n a s
,
i teorema 2.7.3
implica lim
j
f(x
k
j
) = f(x), adica lim
j
y
k
j
= f(x). As
,
adar, sirul (y
k
) poseda
subs
,
irul (y
k
j
), convergent catre f(x) f(A). In baza implicat iei 3

din
teorema 2.4.4, deducem ca mult
,
imea f(A) este compacta.
2.7.10 Teorema (K. Weierstrass). Daca A R
n
este o mult
,
ime compacta
nevida, iar f : A R este o funct
,
ie continua pe A, atunci f este marginita
s
,
i s
,
i atinge marginile.
2.8 Probleme 53
Demonstrat
,
ie. Conform teoremei 2.7.9, f(A) este o submult ime compacta a
lui R. In baza implicat
,
iei 1

din teorema 2.4.4, rezulta ca:


f(A) este marginita, deci f este marginita;
f(A) este nchisa, deci f(A) s
,
i cont ine atat marginea inferioara cat s
,
i
marginea superioara. Cu alte cuvinte, f s
,
i atinge marginile.
2.8 Probleme
1. Dat
,
i exemplu de funct
,
ii f, g : R R, care ndeplinesc urmatoarele
condit
,
ii:
(i) lim
x0
g(x) = 1;
(ii) lim
y1
f(y) = 2;
(iii) lim
x0
(f g)(x) = 2.
2. Sa se calculeze urmatoarele limite:
1) lim
(x,y)(0,0)
_
x
2
+y
2
_
sin
1
xy
;
2) lim
(x,y)(0,2)
sin(xy)
x
;
3) lim
(x,y)(0,0)
x
2
y
2
x
2
+y
2
;
4) lim
(x,y)(0,0)
x
3
+y
3
x
2
+y
2
;
5) lim
(x,y)(0,0)
x
3
+y
3
xy
;
6) lim
(x,y)(0,0)
1 cos
_
x
3
+y
3
_
x
2
+y
2
;
7) lim
(x,y)(0,0)
x
2
+y
2

1 +x
2
+y
2
1
;
8) lim
(x,y)(0,0)

1 +x
2
y
2
1
x
2
+y
2
;
9) lim
(x,y)(0,0)
1 cos
_
x
2
+y
2
_
x
2
y
2
(x
2
+y
2
)
;
54 2 Topologie n R
n
10) lim
(x,y)(0,0)
e

1
x
2
+y
2
x
4
+y
4
;
11) lim
(x,y)(0,0)
_
1 +x
2
y
2
_

1
x
2
+y
2
;
12) lim
(x,y)(0,0)
xy sin xsin y
xy(x
2
+y
2
)
;
13) lim
(x
1
,...,x
n
)0
n
x
1
x
n
x
2
1
+ +x
2
n
, n N;
14) lim
(x
1
,...,x
n
)0
n
x
p
1
+ +x
p
n
x
1
x
n
, n, p N;
15) lim
(x
1
,...,x
n
)0
n
x
1
x
n
sin x
1
sin x
n
x
1
x
n
(x
2
1
+ +x
2
n
)
.
3. Fie A o submult
,
ime compacta a lui R
n
, fara puncte izolate, iar f : A R
o funct
,
ie avand limita nita n ecare punct al lui A. Sa se demonstreze
ca f este marginita.
4. Fie A = [0, 1) [0, 1) s
,
i f : A R funct
,
ia denita prin
f(x, y) =

1
2

m
n
2
x
m
y
n
,
unde sumarea se face pentru toate perechile de numere naturale (m, n)
care satisfac inegalitat
,
ile indicate. Sa se determine
lim
(x,y)(1,1)
(1 xy
2
)(1 x
2
y)f(x, y).
Concursul William Lowell Putnam 1999
5. Sa se demonstreze ca pentru orice numar natural n 2 exista o funct
,
ie
f : R
n
R cu proprietatea ca toate cele n! limite iterate
lim
x
(1)
0
lim
x
(2)
0
lim
x
(n)
0
f(x
1
, x
2
, . . . , x
n
) S
n
exista s
,
i sunt distincte doua cate doua.
Olimpiada student
,
easca, U.R.S.S.
6. Fie B o submult
,
ime nchisa a lui R
n
s
,
i f
1
, . . . , f
p
, g
1
, . . . , g
q
: B R
funct
,
ii continue pe B. Sa se demonstreze ca mult
,
imea
A = { x B | f
i
(x) = 0, i = 1, . . . , p, g
j
(x) 0, j = 1, . . . , q }
este nchisa.
2.8 Probleme 55
7. Fie f : R
n
R
m
o funct ie continua pe R
n
si e B o submult ime nchisa
a lui R
m
. Sa se demonstreze ca mult imea
f
1
(B) := {x R
n
| f(x) B}
este nchisa n R
n
.
8. Sa se demonstreze ca norma euclidiana este o funct
,
ie continua de la R
n
n [0, ).
9. Conform teoremei lui Weierstrass, daca A R
n
este o mult
,
ime com-
pacta, atunci orice funct
,
ie continua f : A R este marginita (s
,
i chiar
s
,
i atinge marginile). Demonstrat
,
i reciproca: daca A este o submult
,
ime a
lui R
n
cu proprietatea ca orice funct
,
ie continua f : A R este marginita,
atunci A este compacta.
Berkeley 1987
10. Fie A o submult
,
ime compacta nevida a lui R
n
s
,
i f : A A o funct
,
ie cu
proprietatea
x, y A cu x = y : f(x) f(y) < x y.
Sa se demonstreze ca f are un unic punct x n A.
11. Fie f : B(0
n
, 1) B(0
n
, 1) o funct
,
ie continua cu proprietatea
x B(0
n
, 1) \ {0
n
} : f(x) < x.
Fiind dat un punct oarecare x
0
B(0
n
, 1), se construies
,
te recursiv s
,
irul
(x
k
) punand x
k
:= f (x
k1
). Sa se demonstreze ca lim
k
x
k
= 0
n
.
Berkeley 1991
56 2 Topologie n R
n
Capitolul 3
Calcul diferent
,
ial n R
n
3.1 Spat
,
iul normat al aplicat
,
iilor liniare
3.1.1 Denit
,
ie (aplicat
,
ii liniare). O aplicat
,
ie : R
n
R
m
se numes
,
te
liniara daca
, R, x, y R
n
: (x +y) = (x) +(y).
Notam
L(R
n
, R
m
) := { : R
n
R
m
| este aplicat
,
ie liniara }.
Se constata imediat ca, daca L(R
n
, R
m
), atunci (0
n
) = 0
m
s
,
i
x R
n
: (x) = (x).
De asemenea, se arata us
,
or (prin induct
,
ie) ca pentru orice k N, orice scalari

1
, . . . ,
k
R s
,
i orice puncte x
1
, . . . , x
k
R
n
are loc egalitatea
(
1
x
1
+ +
k
x
k
) =
1
(x
1
) + +
k
(x
k
).
3.1.2 Teorema (forma generala a aplicat
,
iilor liniare de la R
n
n R
m
). O
aplicat
,
ie : R
n
R
m
este liniara daca s
,
i numai daca exista n puncte
v
1
, . . . , v
n
R
m
n as
,
a fel ncat
(x) = x
1
v
1
+ +x
n
v
n
oricare ar x = (x
1
, . . . , x
n
) R
n
.
Demonstrat
,
ie. Necesitatea. Notam v
1
:= (e
1
), . . . , v
n
:= (e
n
), {e
1
, . . . , e
n
}
ind baza canonica a spat
,
iului R
n
. Atunci v
1
, . . . , v
n
R
m
si pentru orice
x = (x
1
, . . . , x
n
) R
n
avem
(x) = (x
1
e
1
+ +x
n
e
n
) = x
1
(e
1
) + +x
n
(e
n
)
= x
1
v
1
+ +x
n
v
n
.
57
58 3 Calcul diferent
,
ial n R
n
Sucient
,
a. Fie , R s
,
i x = (x
1
, . . . , x
n
), y = (y
1
, . . . , y
n
) puncte din
R
n
. Atunci x +y = (x
1
+y
1
, . . . , x
n
+y
n
), deci
(x +y) = (x
1
+y
1
)v
1
+ + (x
n
+y
n
)v
n
= (x
1
v
1
+ +x
n
v
n
) +(y
1
v
1
+ +y
n
v
n
)
= (x) +(y).
Prin urmare, este liniara.
3.1.3 Consecint
,
a (forma generala a aplicat
,
iilor liniare de la R
n
n R). O
aplicat
,
ie : R
n
R este liniara daca s
,
i numai daca exista un punct v R
n
as
,
a ncat
(x) = x, v oricare ar x R
n
.
Demonstrat
,
ie. Necesitatea. Adnit
,
and ca este liniara, n baza part
,
ii de nece-
sitate a teoremei 3.1.2 rezulta ca exista v
1
, . . . , v
n
R astfel ncat
(x) = x
1
v
1
+ +x
n
v
n
oricare ar x = (x
1
, . . . , x
n
) R
n
.
Notand v := (v
1
, . . . , v
n
), avem v R
n
s
,
i (x) = x, v oricare ar x R
n
.
Sucient
,
a. Este evidenta.
3.1.4 Denit
,
ie (matricea unei aplicat
,
ii liniare). Fie = (
1
, . . . ,
m
)
L(R
n
, R
m
). Atunci (e
1
), . . . , (e
n
) sunt puncte ale lui R
m
. Fie
(e
1
) := (v
11
, v
12
, . . . , v
1m
), v
1i
=
i
(e
1
), i = 1, . . . , m,
(e
2
) := (v
21
, v
22
, . . . , v
2m
), v
2i
=
i
(e
2
), i = 1, . . . , m,
.
.
.
(e
n
) := (v
n1
, v
n2
, . . . , v
nm
), v
ni
=
i
(e
n
), i = 1, . . . , m.
Construim matricea
[] := (
i
(e
j
))
i=1,m,j=1,n
=
_
_
_
_
_
v
11
v
21
. . . v
n1
v
12
v
22
. . . v
n2
.
.
.
.
.
.
.
.
.
v
1m
v
2m
. . . v
nm
_
_
_
_
_
R
mn
.
Ea se numes
,
te matricea aplicat
,
iei liniare . Se observa ca pe prima coloana
n [] se aa vectorul (e
1
), pe coloana a doua se aa vectorul (e
2
), . . ., pe
3.1 Spat
,
iul normat al aplicat
,
iilor liniare 59
coloana n se aa vectorul (e
n
). Pentru orice punct x = (x
1
, . . . , x
n
) R
n
are loc egalitatea
(1)
_
_
_

1
(x)
.
.
.

m
(x)
_
_
_
= []
_
_
_
x
1
.
.
.
x
n
_
_
_
.
3.1.5 Observat
,
ie. Atunci cand intervine n egalitat
,
i matriciale, un punct
(x
1
, . . . , x
n
) R
n
va identicat cu matricea coloana
_
_
_
x
1
.
.
.
x
n
_
_
_
R
n1
. Ast-
fel, putem scrie
x, y =
_
x
1
x
2
. . . x
n
_
_
_
_
_
_
y
1
y
2
.
.
.
y
n
_
_
_
_
_
= x
T
y.
In particular, x
2
= x, x = x
T
x. De asemenea, cu convent
,
ia de mai sus,
egalitatea (1) poate rescrisa matricial sub forma
(x) = [] x.
3.1.6 Teorema. Daca a, b R, iar , L(R
n
, R
m
), atunci a + b
L(R
n
, R
m
) s
,
i are loc egalitatea [a +b] = a[] +b[].
Demonstrat
,
ie. Imediata.
Din teorema 3.1.6 rezulta ca mult
,
imea L(R
n
, R
m
), nzestrata cu operat
,
iile
de adunare a funct
,
iilor s
,
i respectiv de nmult
,
ire a unei funct
,
ii cu un scalar
real, este un spat
,
iu liniar real. In acest spat
,
iu liniar, originea este funct
,
ia
x R
n
0
m
R
m
, iar opusa unei aplicat
,
ii L(R
n
, R
m
) este aplicat
,
ia
x R
n
(x) R
m
. De asemenea, funct
,
ia
L(R
n
, R
m
) [] R
mn
este un izomorsm ntre spat
,
iile liniare reale L(R
n
, R
m
) s
,
i R
mn
.
3.1.7 Teorema. Daca L(R
n
, R
m
) s
,
i L(R
m
, R
p
) atunci
L(R
n
, R
p
) s
,
i are loc egalitatea [ ] = [] [].
Demonstrat
,
ie. Imediata.
60 3 Calcul diferent
,
ial n R
n
3.1.8 Teorema. Aplicat
,
ia = (
1
, . . . ,
m
) : R
n
R
m
este liniara daca s
,
i
numai daca toate aplicat
,
iile
1
, . . . ,
m
: R
n
R sunt liniare.
Demonstrat
,
ie. Imediata.
3.1.9 Teorema. Orice aplicat
,
ie liniara : R
n
R
m
este o funct ie Lipschitz.
Demonstrat
,
ie. Fie x = (x
1
, . . . , x
n
) un punct arbitrar din R
n
. Avem
(x) = (x
1
e
1
+ +x
n
e
n
) = x
1
(e
1
) + +x
n
(e
n
),
deci
(x) |x
1
| (e
1
) + +|x
n
| (e
n
)
x (e
1
) + +x (e
n
).
Notand := (e
1
) + + (e
n
), deducem ca (x) x oricare ar
x R
n
. Pentru orice x, x

R
n
avem atunci
(x) (x

) = (x x

) x x

,
deci este o funct
,
ie Lipschitz.
3.1.10 Denit
,
ie (norma unei aplicat
,
ii liniare). Din teorema 3.1.9 rezulta ca
orice aplicat ie L(R
n
, R
m
) este continua pe R
n
. Notam
S
n1
:= {(x
1
, . . . , x
n
) R
n
| x
2
1
+ +x
2
n
= 1},
sfera cu centrul n origine s
,
i de raza 1 din R
n
. Fiind marginita s
,
i nchisa,
S
n1
este o submult
,
ime compacta a lui R
n
, conform teoremei 2.4.4. Deoarece
funct
,
ia x R
n
(x) [0, ) este continua, n baza teoremei lui
Weierstrass putem introduce numarul real
:= max
xS
n1
(x).
Acesta se numes
,
te norma aplicat
,
iei liniare .
3.1.11 Teorema. Daca L(R
n
, R
m
) s
,
i L(R
m
, R
p
) atunci urmatoarele
armat
,
ii sunt adevarate:
1

(x) x oricare ar x R
n
.
2

.
3.1 Spat
,
iul normat al aplicat
,
iilor liniare 61
Demonstrat
,
ie. 1

Fie x R
n
. Daca x = 0
n
, atunci inegalitatea din enunt
,
are
loc cu egalitate. Presupunand x = 0
n
, notam y :=
1
x
x. Atunci y S
n1
,
deci
(y) =
_
_
_
_

_
1
x
x
__
_
_
_
=
1
x
(x).
In consecint
,
a, avem (x) x.
2

Conform teoremei 3.1.7, avem L(R


n
, R
p
). Fie x
0
S
n1
un
punct cu proprietatea ( )(x
0
) = . Folosind inegalitatea de la 1

pentru n locul lui s


,
i (x
0
) n locul lui x, avem
= ((x
0
)) (x
0
) .
3.1.12 Teorema. Funct
,
ia : L(R
n
, R
m
) [0, ) este o norma pe spat
,
iul
liniar real L(R
n
, R
m
).
Demonstrat
,
ie. Trebuie sa aratam ca funct
,
ia ndeplines
,
te urmatoarele
condit
,
ii:
(N1) = 0 = 0;
(N2) R L(R
n
, R
m
) : = ||;
(N3) , L(R
n
, R
m
) : + +.
(N1) Conform teoremei 3.1.11 avem
= 0 x R
n
: (x) = 0
m
= 0.
(N2) Fie x
0
S
n1
as
,
a ncat = (x
0
) Avem
x S
n1
: ()(x) = (x) = ||(x) ||,
()(x
0
) = ||(x
0
) = ||,
deci || = max
xS
n1
()(x) = .
(N3) Fie x
0
S
n1
astfel ca ( +)(x
0
) = +. Avem atunci
+ = (x
0
) +(x
0
) (x
0
) +(x
0
) +.
62 3 Calcul diferent
,
ial n R
n
3.1.13 Teorema. Daca L(R
n
, R
n
), atunci urmatoarele armat
,
ii sunt
echivalente:
1

este bijectiv a.
2

este injectiv a.
3

este surjectiv a.
4

det[] = 0.
Demonstrat
,
ie. Se s
,
tie de la cursul de Algebra liniara.
3.1.14 Teorema. Daca : R
n
R
n
este o aplicat
,
ie liniara bijectiva, atunci

1
L(R
n
, R
n
) s
,
i are loc egalitatea [
1
] = []
1
.
Demonstrat
,
ie. Imediata.
3.2 Probleme
1. Fie : R
2
R
2
aplicat
,
ia liniara avand matricea [] =
_
a
11
a
12
a
21
a
22
_
.
Sa se determine .
2. Fie a
1
, . . . , a
n
R s
,
i : R
n
R aplicat
,
ia liniara denita prin
(x
1
, . . . , x
n
) := a
1
x
1
+ +a
n
x
n
.
Sa se demonstreze ca =

a
2
1
+ +a
2
n
.
3. Fie a
1
, . . . , a
n
R s
,
i : R
n
R
n
aplicat
,
ia liniara avand matricea
[] =
_
_
_
_
_
a
1
0 0 0
0 a
2
0 0
.
.
.
0 0 0 a
n
_
_
_
_
_
.
Sa se determine .
4. Fie : R
n
R
m
o aplicat
,
ie liniara avand matricea [] = (a
ij
) R
mn
.
Sa se demonstreze ca are loc inegalitatea

m
i=1

n
j=1
a
2
ij
n

_
m

i=1
n

j=1
a
2
ij
.
3.3 Derivata unei funct ii vectoriale de variabila reala 63
5. Fie : R
n
R
m
o aplicat
,
ie liniara avand matricea [] R
mn
. Sa se
demonstreze ca =

, unde

este cea mai mare valoare proprie


a matricei simetrice []
T
[].
6. Sa se demonstreze ca pentru orice a, b, c R
n
are loc inegalitatea
_
ab, c
_
2
+
_
ba, c
_
2
a b c
2
_
a b +|a, b|
_
.
SEEMOUS 2011
3.3 Derivata unei funct
,
ii vectoriale de variabila
reala
3.3.1 Denit
,
ie. Fie A R, a AA

s
,
i f : A R
m
o funct
,
ie. Daca exista
un element R
m
as
,
a ncat
lim
xa
1
x a
_
f(x) f(a)

= ,
atunci se spune ca f este derivabila n punctul a, iar se numes
,
te derivata
funct
,
iei f n a s
,
i se noteaza cu f

(a) sau cu
df
dx
(a).
3.3.2 Teorema. Fie A R, a A A

s
,
i f = (f
1
, . . . , f
m
) : A R
m
o
funct
,
ie. Atunci urmatoarele armat
,
ii sunt adevarate:
1

Daca f este derivabila n punctul a, atunci f


1
, . . . , f
m
sunt derivabile
n a s
,
i are loc egalitatea
(1) f

(a) = (f

1
(a), . . . , f

m
(a)).
2

Daca f
1
, . . . , f
m
sunt derivabile n punctul a, atunci f este derivabila
n a s
,
i are loc egalitatea (1).
Demonstrat
,
ie. Avem f(x) = (f
1
(x), . . . , f
m
(x)) pentru orice x A, deci
1
x a
_
f(x) f(a)

=
_
f
1
(x) f
1
(a)
x a
, . . . ,
f
m
(x) f
m
(a)
x a
_
pentru orice x A\{a}. T
,
inand seama de aceasta egalitate, cele doua armat
,
ii
sunt consecint
,
e imediate ale teoremei 2.6.9.
64 3 Calcul diferent
,
ial n R
n
Proprietat
,
ile privind operat
,
ii cu funct
,
ii reale de variabila reala derivabile
se transpun imediat la cadrul funct
,
iilor vectoriale de variabila reala derivabile.
Teorema de medie nu ramane nsa adevarata sub forma unei egalitat
,
i n cazul
funct
,
iilor vectoriale de variabila reala.
Contraexemplu: e f : [0, 2] R
2
, f(x) := (cos x, sin x). Se constata
imediat ca nu exista niciun c (0, 2) astfel ca f(2) f(0) = 2f

(c).
3.3.3 Teorema (teorema de medie pentru funct
,
ii vectoriale de variabila
reala). Fie f : [a, b] R
m
o funct
,
ie continua pe [a, b] s
,
i derivabila pe (a, b).
Atunci exista un punct c (a, b) asa nc at
f(b) f(a) (b a)f

(c).
Demonstrat
,
ie. Daca f(a) = f(b), atunci c poate ales arbitrar n (a, b). Pre-
supunem n continuare ca f(a) = f(b). Consideram vectorul
v :=
1
f(b) f(a)
_
f(b) f(a)

R
m
s
,
i cu ajutorul sau denim funct
,
ia g : [a, b] R prin g(x) := v, f(x). Daca
v
1
, . . . , v
m
sunt coordonatele lui v, iar f
1
, . . . , f
m
sunt componentele scalare
ale lui f, atunci
g(x) = v
1
f
1
(x) + +v
m
f
m
(x) oricare ar x [a, b].
Deoarece f este continua pe [a, b], toate funct
,
iile f
1
, . . . , f
m
sunt continue pe
[a, b] conform teoremei 2.7.8, deci g este continua pe [a, b]. Pe de alta parte,
deoarece f este derivabila pe (a, b), toate funct
,
iile f
1
, . . . , f
m
sunt derivabile pe
(a, b) conform teoremei 3.3.2, deci g este derivabila pe (a, b). In plus, pentru
orice x (a, b) avem
g

(x) = v
1
f

1
(x) + +v
m
f

m
(x) = v, f

(x).
Aplicand lui g teorema de medie pentru funct
,
ii reale de variabila reala (teo-
rema lui Lagrange), deducem existent
,
a unui punct c (a, b) astfel ncat
g(b) g(a) = (b a)g

(c),
adica
v, f(b) f(a) = (b a)v, f

(c).
Conform inegalitat
,
ii lui CauchyBuniakovskiSchwarz, avem
v, f(b) f(a) (b a)v f

(c).
Dar v = 1, iar v, f(b) f(a) = f(b) f(a). In concluzie, punctul c
satisface inegalitatea f(b) f(a) (b a)f

(c).
3.4 Diferent
,
iabilitatea unei funct
,
ii vectoriale de variabila vectoriala 65
3.4 Diferent
,
iabilitatea unei funct
,
ii vectoriale de
variabila vectoriala
3.4.1 Lema. Fie A R
n
, a int A s
,
i e f : A R
m
o funct
,
ie. Daca

1
,
2
L(R
n
, R
m
) sunt aplicat
,
ii liniare cu proprietatea
(1) lim
xa
1
x a
_
f(x) f(a)
i
(x a)

= 0
m
, i = 1, 2,
atunci
1
=
2
.
Demonstrat
,
ie. Fixam un punct oarecare v R
n
s
,
i dovedim ca
1
(v) =
2
(v).
Aceasta egalitate ind evident adevarata pentru v = 0
n
, presupunemn contin-
uare ca v = 0
n
. Fie r > 0 astfel ca B(a, r) A. Intrucat a+tva = |t|v,
notand := r/v, avem a + tv B(a, r) pentru orice t (, ). Denind
h : (, ) B(a, r) prin h(t) := a +tv, avem
(2) lim
t0
h(t) = a s
,
i h(t) = a oricare ar t (, ) \ {0}.
Pe de alta parte, pentru orice x A\ {a} avem
1
x a
_

1
(x a)
2
(x a)

(3)
=
1
x a
_
f(x) f(a)
2
(x a)

1
x a
_
f(x) f(a)
1
(x a)

.
Denind g : A \ {a} R
m
prin g(x) :=
1
x a
_

1
(x a)
2
(x a)

, din
(1) s
,
i (3) rezulta ca
(4) lim
xa
g(x) = 0
m
.
Din (2) s
,
i (4), n baza teoremei 2.6.7, deducem ca lim
t0
(g h)(t) = 0
m
, deci
(5) lim
t0
(g h)(t) = 0
m
.
Dar, pentru orice t (0, ) avem
(g h)(t) =
1
a +tv a
_

1
(a +tv a)
2
(a +tv a)

(6)
=
1
tv
t
_

1
(v)
2
(v)

=
1
v
_

1
(v)
2
(v)

.
Din (5) s
,
i (6) rezulta acum ca
1
(v) =
2
(v).
66 3 Calcul diferent
,
ial n R
n
3.4.2 Denit
,
ie (diferent
,
iala unei funct
,
ii vectoriale de variabila vectoriala).
Fie A R
n
, a int A s
,
i f : A R
m
o funct
,
ie. Daca exista o aplicat
,
ie liniara
: R
n
R
m
n as
,
a fel ncat
(7) lim
xa
1
x a
_
f(x) f(a) (x a)

= 0
m
,
atunci se spune ca funct
,
ia f este diferent
,
iabila (n sens Frechet) n punctul
a. Daca f este diferent
,
iabila n punctul a, atunci lema 3.4.1 arata ca exista
o singura aplicat
,
ie liniara : R
n
R
m
care satisface (7). Aceasta unica
aplicat
,
ie liniara se numeste diferent
,
iala (Frechet) a lui f n punctul a s
,
i va
notata cu df(a).
As
,
adar, daca f este diferent
,
iabila n a, atunci
df(a) L(R
n
, R
m
) s
,
i x R
n
: df(a)(x) R
m
.
De asemenea, avem
(8) lim
xa
1
x a
_
f(x) f(a) df(a)(x a)

= 0
m
.
3.4.3 Propozit
,
ie. Fie A R
n
, a int A s
,
i f : A R
m
o funct ie. Atunci
urmatoarele armat
,
ii sunt adevarate:
1

Daca f este diferent


,
iabila n a, atunci exista o funct
,
ie : A R
m
, cu
urmatoarele proprietat
,
i:
(9) lim
xa
(x) = 0
m
s
,
i
(10) f(x) = f(a) +df(a)(x a) +x a(x) oricare ar x A.
2

Daca exista o aplicat


,
ie liniara : R
n
R
m
s
,
i o funct
,
ie : A R
m
as
,
a ncat sa aiba loc (9) s
,
i
(11) f(x) = f(a) +(x a) +x a(x) oricare ar x A,
atunci f este diferent
,
iabila n a s
,
i df(a) = .
Demonstrat
,
ie. 1

Daca f este diferent


,
iabila n punctul a, atunci are loc (8).
Fie : A R
m
funct
,
ia denita prin
(x) :=
1
x a
_
f(x) f(a) df(a)(x a)

daca x = a,
(a) := 0
m
.
3.4 Diferent
,
iabilitatea unei funct
,
ii vectoriale de variabila vectoriala 67
Evident, satisface (9) s
,
i (10).
2

Din (11) rezulta ca


(x) =
1
x a
_
f(x) f(a) (x a)

pentru orice x A\{a}. T


,
inand seama de (9), deducem ca (7) are loc. Drept
urmare, f este diferent
,
iabila n punctul a si df(a) = .
3.4.4 Teorema (continuitatea funct
,
iilor diferent iabile). Daca A R
n
, a
int A s
,
i f : A R
m
este o funct
,
ie diferent iabil a n punctul a, atunci f este
continua n a.
Demonstrat
,
ie. Conform primei armat
,
ii din propozit
,
ia 3.4.3, exista o funct
,
ie
: A R
m
care satisface (9) s
,
i (10). Din (10) rezulta ca pentru orice x A
avem
f(x) f(a) df(a) x a +x a (x).
In consecint
,
a, avem lim
xa
f(x) f(a) = 0, deci lim
xa
f(x) = f(a), conform teo-
remei 2.6.8. Cum int A A A

, n baza teoremei 2.7.4 deducem ca f este


continua n a.
3.4.5 Teorema (legatura dintre derivata s
,
i diferent iala n cazul funct
,
iilor
vectoriale de variabila reala). Fie A R, a int A s
,
i f : A R
m
o funct
,
ie.
Atunci urmatoarele armat
,
ii sunt adevarate:
1

Daca f este derivabila n punctul a, atunci f este diferent


,
iabila n a s
,
i
are loc egalitatea
(12) df(a)(x) = xf

(a) oricare ar x R.
2

Daca f este diferent


,
iabila n punctul a, atunci f este derivabila n a s
,
i
are loc egalitatea (12).
Demonstrat
,
ie. 1

Presupunem ca f este derivabila n a. Fie : R R


m
funct
,
ia denita prin (x) := xf

(a) (x R). Evident, avem L(R, R


m
).
Deoarece
lim
xa
_
_
_
_
1
|x a|
_
f(x) f(a) (x a)

_
_
_
_
= lim
xa
_
_
_
_
1
|x a|
_
f(x) f(a) (x a)f

(a)

_
_
_
_
= lim
xa
_
_
_
_
x a
|x a|
_
1
x a
_
f(x) f(a)
_
f

(a)
__
_
_
_
= lim
xa
_
_
_
_
1
x a
_
f(x) f(a)
_
f

(a)
_
_
_
_
= 0,
68 3 Calcul diferent
,
ial n R
n
conform teoremei 2.6.8 rezulta ca
lim
xa
1
|x a|
_
f(x) f(a) (x a)

= 0
m
,
deci f este diferent
,
iabila n a s
,
i df(a) = . Cu alte cuvinte, (12) are loc.
2

Admitem acum ca f este diferent


,
iabila n a. Cum df(a) L(R, R
m
),
n baza teoremei 3.1.2 exista un v R
m
astfel ca df(a)(x) = xv oricare ar
x R. Avem
lim
xa
1
|x a|
_
f(x) f(a) df(a)(x a)

= 0
m
,
deci
lim
xa
1
|x a|
_
_
f(x) f(a) (x a)v
_
_
= 0,
adica
lim
xa
_
_
_
_
1
x a
_
f(x) f(a)
_
v
_
_
_
_
= 0.
Aplicand teorema 2.6.8 rezulta ca
lim
xa
1
x a
_
f(x) f(a)
_
= v.
Prin urmare, f este derivabila n a s
,
i f

(a) = v. Aceasta egalitate probeaza


validitatea lui (12).
Teorema precedenta arata ca diferent
,
iabilitatea Frechet generalizeaza no-
t
,
iunea de derivabilitate, introdusa pentru funct
,
ii vectoriale de variabila reala.
3.4.6 Teorema. Fie A R
n
, a int A s
,
i f = (f
1
, . . . , f
m
) : A R
m
o
funct
,
ie. Atunci urmatoarele armat
,
ii sunt adevarate:
1

Daca f este diferent


,
iabila n punctul a, atunci f
1
, . . . , f
m
sunt diferen-
t
,
iabile n a s
,
i are loc egalitatea
(13) df(a) = (df
1
(a), . . . , df
m
(a)).
2

Daca f
1
, . . . , f
m
sunt diferent
,
iabile n punctul a, atunci f este diferen-
t
,
iabila n punctul a, s
,
i are loc egalitatea (13).
3.5 Derivata dupa o direct
,
ie 69
Demonstrat
,
ie. 1

Fie := df(a) L(R


n
, R
m
) s
,
i e
1
, . . . ,
m
componentele
scalare ale lui . Pentru orice x A\ {a} avem
1
x a
_
f(x) f(a) (x a)

(14)
=
_
f
1
(x) f
1
(a)
1
(x a)
x a
, . . . ,
f
m
(x) f
m
(a)
m
(x a)
x a
_
Intrucat are loc (7), din (14) deducem n baza teoremei 2.6.9 ca
(15) lim
xa
f
i
(x) f
i
(a)
i
(x a)
x a
= 0 oricare ar i {1, . . . , m}.
Drept urmare, toate funct
,
iile f
1
, . . . , f
m
sunt diferent iabile n a s
,
i df
i
(a) =
i
pentru ecare i {1, . . . , m}. Deci egalitatea (13) are loc.
2

Notand
i
:= df
i
(a), are loc (15). Notand apoi := (
1
, . . . ,
m
), avem
L(R
n
, R
m
). Pe de lata parte, din (14) s
,
i (15), n baza teoremei 2.6.9
rezulta ca (7) are loc, deci f este diferent
,
iabilan a s
,
i df(a) = . Aceasta
egalitate garanteaza validitatea lui (13).
3.5 Derivata dupa o direct
,
ie a unei funct
,
ii vectoriale
de variabila vectoriala
3.5.1 Denit
,
ie. Fie A R
n
, a int A, f : A R
m
o funct
,
ie si v R
n
.
Daca exista un element R
m
as
,
a ncat
lim
t0
1
t
_
f(a +tv) f(a)

= ,
atunci se spune ca f este derivabila n punctul a dupa direct
,
ia v, iar se
numes
,
te derivata funct iei f n punctul a dupa direct
,
ia v s
,
i va notata cu
f

(a; v).
3.5.2 Teorema. Fie A R
n
, a int A, f = (f
1
, . . . , f
m
) : A R
m
o funct
,
ie
s
,
i v R
n
. Atunci urmatoarele armat
,
ii sunt adevarate:
1

Daca f este derivabila n punctul a dupa direct


,
ia v, atunci f
1
, . . . , f
m
sunt derivabile n a dupa direct
,
ia v s
,
i are loc egalitatea
(1) f

(a; v) =
_
f

1
(a; v), . . . , f

m
(a; v)
_
.
2

Daca f
1
, . . . , f
m
sunt derivabile n punctul a dupa direct
,
ia v, atunci f
este derivabila n a dupa direct
,
ia v s
,
i are loc egalitatea (1).
70 3 Calcul diferent
,
ial n R
n
Demonstrat
,
ie. Deoarece a int A, exista un > 0 astfel ncat pentru orice
t [, ] sa avem a +tv A. Oricare ar t [, ] \ {0} avem
1
t
_
f(a +tv) f(a)

=
_
1
t
_
f
1
(a +tv) f
1
(a)

, . . . ,
1
t
_
f
m
(a +tv) f
m
(a)

_
.
T
,
inand seama de aceasta egalitate, cele doua armat ii sunt consecint
,
e imediate
ale teoremei 2.6.9.
3.5.3 Teorema. Fie A R
n
, a int A, iar f : A R
m
o funct ie diferent
,
ia-
bila n punctul a. Atunci f este derivabila n a dupa orice direct
,
ie v R
n
s
,
i
are loc egalitatea
v R
n
: f

(a; v) = df(a)(v).
Demonstrat
,
ie. Fie v R
n
arbitrar. Vom dovedi ca
(2) lim
t0
1
t
_
f(a +tv) f(a)

= df(a)(v).
Conform armat
,
iei 1

din propozit
,
ia 3.4.3, exista o funct
,
ie : A R
m
astfel
ca
lim
xa
(x) = 0
m
s
,
i
(3) f(x) = f(a) +df(a)(x a) +x a(x) oricare ar x A.
Fara a restrange generalitatea, putem presupune ca (a) = 0
m
, adica este
continua n a.
Deoarece a int A, exista un > 0 astfel ca a + tv A oricare ar
t [, ]. T
,
inand seama de (3), deducem ca pentru orice t [, ] avem
f(a +tv) = f(a) +tdf(a)(v) +|t| v(a +tv).
Rezulta de aici ca pentru orice t [, ] \ {0} avem
1
t
_
f(a +tv) f(a)

df(a)(v) =
|t|
t
v(a +tv),
deci
_
_
_
_
1
t
_
f(a +tv) f(a)

df(a)(v)
_
_
_
_
= v (a +tv) 0
cand t 0. Conform teoremei 2.6.8, rezulta ca (2) are loc, deci f este
derivabila n punctul a dupa direct
,
ia v s
,
i f

(a; v) = df(a)(v).
3.6 Derivate part
,
iale ale unei funct
,
ii vectoriale de variabila vectoriala 71
3.6 Derivate part
,
iale ale unei funct
,
ii vectoriale de
variabila vectoriala
3.6.1 Denit
,
ie. Fie {e
1
, . . . , e
n
} baza canonica a spat
,
iului R
n
. Fie apoi
A R
n
, a int A, f : A R
m
o funct ie s
,
i j {1, . . . , n}. Daca f este
derivabila n punctul a dupa direct
,
ia e
j
, atunci se spune ca f este derivabila
part
,
ial n raport cu variabila x
j
n a, iar elementul f

(a; e
j
) R
m
se numes
,
te
derivata part
,
iala a funct
,
iei f n raport cu variabila x
j
n a s
,
i se noteaza cu
f
x
j
(a) sau cu f

x
j
(a) sau cu D
j
f(a). Avem
f
x
j
(a) = f

(a; e
j
) = lim
t0
1
t
_
f(a +te
j
) f(a)

= lim
t0
1
t
_
f(a
1
, . . . , a
j1
, a
j
+t, a
j+1
, . . . , a
n
)
f(a
1
, . . . , a
j1
, a
j
, a
j+1
, . . . , a
n
)

= lim
x
j
a
j
1
x
j
a
j
_
f(a
1
, . . . , a
j1
, x
j
, a
j+1
, . . . , a
n
)
f(a
1
, . . . , a
j1
, a
j
, a
j+1
, . . . , a
n
)

.
Daca f este derivabila part
,
ial n punctul an raport cu ecare dintre variabilele
x
1
, . . . , x
n
, atunci vom spune simplu ca f este derivabila part
,
ial n punctul a.
3.6.2 Teorema. Fie A R
n
, a int A, f = (f
1
, . . . , f
m
) : A R
m
o funct
,
ie
s
,
i j {1, . . . , n}. Atunci urmatoarele armat
,
ii sunt adevarate:
1

Daca f este derivabila part


,
ial n raport cu variabila x
j
n punctul a,
atunci f
1
, . . . , f
m
sunt derivabile part
,
ial n raport cu x
j
n a s
,
i are loc egalitatea
(1)
f
x
j
(a) =
_
f
1
x
j
(a), . . . ,
f
m
x
j
(a)
_
.
2

Daca f
1
, . . . , f
m
sunt derivabile part
,
ial n raport cu variabila x
j
n punc-
tul a, atunci s
,
i f este derivabila part
,
ial n raport cu x
j
n punctul a s
,
i are loc
egalitatea (1).
Demonstrat
,
ie. Rezulta din Teorema 3.5.2 pentru v = e
j
.
3.6.3 Denit
,
ie (matricea Jacobi). Fie A o submult
,
ime a lui R
n
, e a int A
s
,
i e f = (f
1
, . . . , f
m
) : A R
m
o funct
,
ie derivabila part ial n punctul a.
72 3 Calcul diferent
,
ial n R
n
Atunci putem forma matricea
_
_
_
f
1
x
1
(a)
f
1
x
2
(a)
f
1
x
n
(a)
.
.
.
.
.
.
.
.
.
f
m
x
1
(a)
f
m
x
2
(a)
f
m
x
n
(a)
_
_
_
R
mn
.
Aceasta se numes
,
te matricea Jacobi a lui f n punctul a s
,
i se noteaza cu
J(f)(a).
In cazul particular m = 1, matricea Jacobi a unei funct
,
ii f : A R n a
este J(f)(a) =
_
f
x
1
(a)
f
x
n
(a)
_
R
1n
. Punctul
_
f
x
1
(a), . . . ,
f
x
n
(a)
_
R
n
se numes
,
te gradientul lui f n punctul a s
,
i se noteaza cu f(a). Cu convent
,
ia
din observat
,
ia 3.1.5 (punctele lui R
n
sunt identicate cu matrice coloana de
tipul n 1), putem scrie J(f)(a) = f(a)
T
.
3.6.4 Teorema (legatura dintre diferent
,
iala s
,
i derivatele part
,
iale). Fie A o
submult ime a lui R
n
, a int A s
,
i f : A R
m
o funct
,
ie diferent
,
iabila n
punctul a. Atunci urmatoarele armat
,
ii sunt adevarate:
1

f este derivabila part


,
ial n a s
,
i [df(a)] = J(f)(a).
2

Pentru orice h = (h
1
, . . . , h
n
) R
n
are loc egalitatea
df(a)(h) = h
1
f
x
1
(a) + +h
n
f
x
n
(a),
sau, scris sub forma matriceala, df(a)(h) = J(f)(a) h.
Demonstrat
,
ie. 1

Din teorema 3.5.3 rezulta ca f este derivabila n a dupa


ecare dintre direct iile e
1
, . . . , e
n
s
,
i
f

(a; e
j
) = df(a)(e
j
) oricare ar j {1, . . . , n}.
Cu alte cuvinte, f este derivabila part
,
ial n a si avem
(2)
f
x
j
(a) = df(a)(e
j
) oricare ar j {1, . . . , n}.
Dar
f
x
j
(a) este, conform teoremei 3.6.2, tocmai coloana j n matricea J(f)(a),
n timp ce df(a)(e
j
) reprezinta coloana j n matricea [df(a)]. T inand cont de
(2), deducem ca [df(a)] = J(f)(a).
3.6 Derivate part
,
iale ale unei funct
,
ii vectoriale de variabila vectoriala 73
2

Daca h = (h
1
, . . . , h
n
) R
n
, atunci h = h
1
e
1
+ +h
n
e
n
, deci
df(a)(h) = df(a)(h
1
e
1
+ +h
n
e
n
) = h
1
df(a)(e
1
) + +h
n
df(a)(e
n
)
= h
1
f
x
1
(a) + +h
n
f
x
n
(a).
3.6.5 Consecint
,
a. Daca A R
n
, a int A, iar f : A R este o funct
,
ie
diferent
,
iabila n punctul a, atunci
df(a)(h) = h, f(a) oricare ar h R
n
,
sau, scris sub forma matriceala, df(a)(h) = h
T
f(a).
3.6.6 Propozit
,
ie. Fie n 2, A R
n
, a int A s
,
i f : A R o funct
,
ie care
se bucura de urmatoarele proprietat
,
i:
(i) exista r > 0 astfel ncat B(a, r) A s
,
i f este derivabila part
,
ial n
ecare punct din B(a, r);
(ii) pentru ecare j {1, . . . , n}, funct
,
ia
x B(a, r)
f
x
j
(x) R
este continua n a.
Atunci f este diferent
,
iabila n punctul a.
Demonstrat
,
ie. Fie : R
n
R funct
,
ia denita prin
(h) :=
n

j=1
h
j
f
x
j
(a) pentru orice h = (h
1
, . . . , h
n
) R
n
.
Evident, L(R
n
, R). Aratam ca f este diferent
,
iabila n a s
,
i ca df(a) = ,
adica
lim
xa
1
x a
_
f(x) f(a) (x a)

= 0.
Se constata imediat ca a demonstra egalitatea de mai sus este echivalent cu
a dovedi validitatea urmatoarei armat
,
ii: oricare ar > 0 exista un > 0
astfel ncat pentru orice x A cu x a < sa avem
(3) |f(x) f(a) (x a)| x a.
74 3 Calcul diferent
,
ial n R
n
Fie > 0 arbitrar. Din condit
,
ia (ii) rezulta ca pentru ecare j {1, . . . , n}
exista un
j
> 0 asa ncat pentru orice x A cu x a <
j
sa avem

f
x
j
(x)
f
x
j
(a)

<

:=

n
.
Notam := min {r,
1
, . . . ,
n
} > 0. Atunci B(a, ) A s
,
i demonstrat
,
ia va
ncheiata de ndata ce vom arata ca (3) are loc pentru orice x B(a, ).
Fie as
,
adar x := (x
1
, . . . , x
n
) R
n
cu proprietatea ca x a < . Avem
f(x) f(a) = f(x
1
, x
2
, . . . , x
n
) f(a
1
, a
2
, . . . , a
n
)
= f(x
1
, a
2
, a
3
, . . . , a
n1
, a
n
) f(a
1
, a
2
, a
3
, . . . , a
n1
, a
n
)
+f(x
1
, x
2
, a
3
, . . . , a
n1
, a
n
) f(x
1
, a
2
, a
3
, . . . , a
n1
, a
n
)
+ +
+f(x
1
, x
2
, x
3
, . . . , x
n1
, x
n
) f(x
1
, x
2
, x
3
, . . . , x
n1
, a
n
),
deci
f(x) f(a)
=
n

j=1
_
f(x
1
, . . . , x
j1
, x
j
, a
j+1
, . . . , a
n
) f(x
1
, . . . , x
j1
, a
j
, a
j+1
, . . . , a
n
)
=
n

j=1
_
F
j
(x
j
) F
j
(a
j
)

,
unde F
j
(t) := f(x
1
, . . . , x
j1
, t, a
j+1
, . . . , a
n
). Aplicand teorema de medie a
lui Lagrange funct
,
iei reale de variabila reala F
j
, rezulta existent
,
a unui punct
c
j
, situat ntre a
j
s
,
i x
j
, astfel ca
F
j
(x
j
) F
j
(a
j
) = (x
j
a
j
)F

j
(c
j
).
Dar
F

j
(t) =
f
x
j
(x
1
, . . . , x
j1
, t, a
j+1
, . . . , a
n
).
Notand b
j
:= (x
1
, . . . , x
j1
, c
j
, a
j+1
, . . . , a
n
), avem
F
j
(x
j
) F
j
(a
j
) = (x
j
a
j
)
f
x
j
(b
j
),
deci
f(x) f(a) =
n

j=1
(x
j
a
j
)
f
x
j
(b
j
).
3.6 Derivate part
,
iale ale unei funct
,
ii vectoriale de variabila vectoriala 75
Drept urmare, avem
|f(x) f(a) (x a)| =

j=1
(x
j
a
j
)
f
x
j
(b
j
)
n

j=1
(x
j
a
j
)
f
x
j
(a)

j=1
|x
j
a
j
|

f
x
j
(b
j
)
f
x
j
(a)

.
Dar b
j
a = (x
1
a
1
, . . . , x
j1
a
j1
, c
j
a
j
, 0, . . . , 0
. .
nj
), deci
b
j
a x a <
j
.
Deducem de aici ca

f
x
j
(b
j
)
f
x
j
(a)

<

oricare ar j {1, . . . , n}.


In consecint
,
a, avem
|f(x) f(a) (x a)|

j=1
|x
j
a
j
| n

x a
= x a,
deci (3) are ntr-adevar loc.
3.6.7 Teorema. Fie n 2, A R
n
, a int A s
,
i f : A R
m
o funct
,
ie care
se bucura de urmatoarele proprietat i:
(i) exista r > 0 astfel ncat B(a, r) A s
,
i f este derivabila part
,
ial n
ecare punct din B(a, r);
(ii) pentru ecare j {1, . . . , n}, funct
,
ia
x B(a, r)
f
x
j
(x) R
m
este continua n a.
Atunci f este diferent
,
iabila n punctul a.
76 3 Calcul diferent
,
ial n R
n
Demonstrat
,
ie. Fie f
1
, . . . , f
m
componentele scalare ale lui f. Fixam i arbitrar
n mult
,
imea {1, . . . , m}. Din teorema 3.6.2 rezulta ca f
i
este derivabila part ial
n ecare punct din B(a, r). Din (ii), teorema 3.6.2 s
,
i teorema 2.7.8 rezulta ca
pentru ecare j {1, . . . , n} funct
,
ia
x B(a, r)
f
i
x
j
(x) R
este continuan a. Conform propozit
,
iei 3.6.6, deducem ca f
i
este diferent
,
iabila
n punctul a.
Intrucat toate funct
,
iile f
1
, . . . , f
m
sunt diferent iabile n a, teorema 3.4.6
garanteaza ca s
,
i f este diferent
,
iabila n a.
3.7 Probleme
1. Cu ajutorul funct
,
iei f : [0, 2] R
2
, f(x) := (sin x, cos x), sa se demon-
streze ca teorema de medie a lui Lagrange nu ramane adevarata sub
forma unei egalitat
,
i n cazul funct
,
iilor vectoriale de o variabila reala.
2. Fie f : (0, )
2
R
3
funct
,
ia denita prin
f(x, y) =
_
arctg
x
y
,
1
xy
, x
y
+y
x
_
.
Sa se determine derivatele part
,
iale de ordinul ntai ale lui f s
,
i df(1, 1).
3. Fie A = { (x, y, z) R
3
| xy +z > 0 } s
,
i f : A R
2
funct
,
ia denita prin
f(x, y, z) =
_
ln(xy +z), sin
_

4
(xy +yz +zx)
__
.
Sa se determine derivatele part
,
iale de ordinul ntai ale lui f s
,
i df(1, 2, 1).
4. Sa se demonstreze ca funct
,
ia f : R
2
R, denita prin
f(x, y) =
_
xsin yy sin x
x
2
+y
2
daca (x, y) = (0, 0)
0 daca (x, y) = (0, 0),
este de clasa C
1
.
5. Sa se demonstreze ca funct
,
ia f : R
2
R, denita prin
f(x, y) =

|x| ln
_
1 +x
2
sin
2
y
_
,
este de clasa C
1
.
3.7 Probleme 77
6. Fie f : R R o funct
,
ie continua. Sa se arate ca funct
,
ia F : R
2
R,
denita prin F(x, y) =

y
0
(x t)f(t) dt, este de clasa C
1
.
7. Fie A = { (x, y, z) R
3
| x
3
+ y
3
+ z
3
> 3xyz } s
,
i f : A R funct
,
ia
denita prin f(x, y, z) = ln(x
3
+y
3
+z
3
3xyz). Sa se demonstreze ca
pentru orice (x, y, z) A are loc egalitatea
f
x
(x, y, z) +
f
y
(x, y, z) +
f
z
(x, y, z) =
3
x +y +z
.
8. Sa se determine R astfel ncat funct
,
ia f : R (0, ) R, denita
prin f(x, y) = y

e
x
2
/(4y)
, sa satisfaca
(x, y) R (0, ) : x
2
f
y
(x, y) =

x
_
x
2
f
x
(x, y)
_
.
9. Fie r > 0, A = { (x, y) R
2
| x
2
+y
2
< r
2
} s
,
i f : A R funct
,
ia denita
prin
f(x, y) = 2 ln
r

8
r
2
x
2
y
2
.
Sa se demonstreze ca pentru orice (x, y) A are loc egalitatea

2
f
x
2
(x, y) +

2
f
y
2
(x, y) = e
f(x,y)
.
10. Fie n 2 un numar natural, > 0 s
,
i f : R
n
R funct
,
ia denita prin
f(x) = [( +n 2)]
/2
x

,
norma considerata pe R
n
ind cea euclidiana. Sa se demonstreze ca
pentru orice x R
n
\ {0
n
} are loc egalitatea

2
f
x
2
1
(x) + +

2
f
x
2
n
(x) = [f(x)]
2

.
11. Fie n 3, A = { x R
n
| x < 1 } s
,
i f : A R funct
,
ia denita prin
f(x) =
_

n(n 2)
1 x
2
_n2
2
,
78 3 Calcul diferent
,
ial n R
n
norma considerata pe R
n
ind cea euclidiana. Sa se demonstreze ca
pentru orice x A are loc egalitatea

2
f
x
2
1
(x) + +

2
f
x
2
n
(x) = [f(x)]
n+2
n2
.
12. Fie f : (0, )
2
R
3
funct
,
ia denita n problema 2. Sa se determine
derivata lui f n punctul (1, 1), dupa direct
,
ia versorului care face cu
semiaxa Ox un unghi de 60

.
13. Sa se determine derivata funct
,
iei f din problema 3, n punctul (1, 2, 1),
dupa direct
,
ia v = (v
1
, v
2
, v
3
) R
3
.
14. Fie f : R
2
R o funct
,
ie diferent
,
iabila. S
,
tiind ca derivatele lui f n
punctul (1, 2) dupa direct
,
iile (2, 2) s
,
i (2, 1) sunt egale cu 2 s
,
i respectiv
2, sa se determine gradientul lui f n punctul (1, 2). Sa se determine
derivata lui f n acest punct dupa direct
,
ia (4, 6).
15. Fie a, b > 0. Sa se determine derivata funct
,
iei f : R
2
R,
f(x, y) = 1
x
2
a
2

y
2
b
2
,
n punctul
_
a

2
,
b

2
_
, dupa direct
,
ia versorului normalei interioare n
acest punct la elipsa de ecuat
,
ie
x
2
a
2
+
y
2
b
2
= 1.
16. (Exemplu de funct
,
ie discontinua, derivabila dupa orice direct
,
ie) Sa se
demonstreze ca funct
,
ia f : R
2
R, denita prin
f(x, y) =
_
x
2
y
x
6
+y
2
daca (x, y) = (0, 0)
0 daca (x, y) = (0, 0),
este discontinua n punctul (0, 0), dar este derivabila dupa orice direct
,
ie
n acest punct.
17. Sa se demonstreze ca daca f : R
n
R
m
este o aplicat
,
ie liniara, atunci
f este diferent
,
iabila pe R
n
s
,
i df(a) = f oricare ar a R
n
.
18. Fie f : R
n
R
m
o funct
,
ie cu proprietatea
t > 0, x R
n
: f(tx) = tf(x).
Sa se demonstreze ca daca f este diferent
,
iabila n 0
n
, atunci ea este
liniara.
3.7 Probleme 79
19. Fie A R
n
deschisa nevida s
,
i f : A R
m
o funct
,
ie diferent
,
iabila cu
proprietatea ca exista M > 0 n as
,
a fel ncat
x, y A : f(x) f(y) Mx y.
Sa se demonstreze ca df(x) M oricare ar x A.
20. Fie f = (f
1
, . . . , f
n
) : R
n
R
n
o funct
,
ie diferent
,
iabila n originea 0
n
a
lui R
n
. Sa se demonstreze ca daca f(0
n
) = 0
n
s
,
i
n

i=1
n

j=1
_
f
i
x
j
(0
n
)
_
2
< 1,
atunci exista o bila B R
n
, cu centrul n 0
n
, astfel ca f(B) B.
Berkeley 2000
21. Fie A R
n
, a = (a
1
, . . . , a
n
) int A s
,
i f : A R
m
o funct
,
ie derivabila
part
,
ial n raport cu ecare dintre variabilele x
1
, . . . , x
n
n punctul a. Sa
se demonstreze ca f este diferent
,
iabila n a daca s
,
i numai daca
lim
xa
1
x a
_
_
f(x) f(a)
n

j=1
(x
j
a
j
)
f
x
j
(a)
_
_
= 0
m
.
Problema precedenta furnizeaza urmatorul algoritm pentru studiul diferen-
t
,
iabilitat
,
ii unei funct
,
ii reale f : A R
n
R ntr-un punct a int A:
I. Se studiaza daca f este derivabila part
,
ial n punctul a.
daca f nu este derivabila part
,
ial n a, atunci f nu este diferent
,
iabila n
punctul a;
daca f este derivabila part
,
ial n a, atunci se calculeaza
f
x
j
(a) pentru
j = 1, . . . , n s
,
i se trece la etapa urmatoare.
II. Se studiaza limita
= lim
xa
1
x a
_
_
f(x) f(a)
n

j=1
(x
j
a
j
)
f
x
j
(a)
_
_
= lim
(h
1
,...,h
n
)0
n
f(a
1
+h
1
, . . . , a
n
+h
n
) f(a)

n
j=1
h
j
f
x
j
(a)

h
2
1
+ +h
2
n
.
80 3 Calcul diferent
,
ial n R
n
daca = 0, atunci f este diferent
,
iabila n a s
,
i
h = (h
1
, . . . , h
n
) R
n
: df(a)(h) =
n

j=1
h
j
f
x
j
(a);
n caz contrar, f nu este diferent
,
iabila n a.
Pentru o funct
,
ie vectoriala f = (f
1
, . . . , f
m
) : A R
m
se studiaza, pe baza
algoritmului de mai sus, diferent
,
iabilitatea n a a ecarei funct
,
ii f
i
: A R
(i = 1, . . . , m).
daca toate funct
,
iile f
1
, . . . , f
m
sunt diferent
,
iabile n a, atunci f este
diferent
,
iabila n a s
,
i
df(a) = (df
1
(a), . . . , df
m
(a)).
n caz contrar, f nu este diferent
,
iabila n a.
22. (Exemplu de funct
,
ie continua, derivabila dupa orice direct
,
ie, dar care
nu este diferent
,
iabila) Sa se demonstreze ca funct
,
ia f : R
2
R, denita
prin
f(x, y) =
_
x
3
y
x
4
+y
2
daca (x, y) = (0, 0)
0 daca (x, y) = (0, 0),
este continua s
,
i derivabila dupa orice direct
,
ie n punctul (0, 0), dar nu
este diferent
,
iabila n acest punct.
23. Sa se studieze diferent
,
iabilitatea n (0, 0) a funct
,
iei f : R
2
R, denite
prin
f(x, y) =
_
(x
2
+y
2
) sin
1
x
2
+y
2
daca (x, y) = (0, 0)
0 daca (x, y) = (0, 0).
24. Sa se studieze diferent
,
iabilitatea funct
,
iei f : R
2
R, denite prin
f(x, y) =
3

x
3
+y
3
.
25. Sa se studieze diferent
,
iabilitatea n (0, 0) a funct
,
iei f : R
2
R, denite
prin
f(x, y) =
_
xy
x
2
y
2
x
2
+y
2
daca (x, y) = (0, 0)
0 daca (x, y) = (0, 0).
3.7 Probleme 81
26. Sa se studieze diferent
,
iabilitatea funct
,
iei f : R
2
R, denite prin
f(x, y) =
_
x
4/3
sin
y
x
daca x = 0
0 daca x = 0.
Berkeley 1986
27. Sa se studieze diferent
,
iabilitatea funct
,
iei f : R
2
R, denite prin
f(x, y) =
_
y
2
sin
x
y
daca y = 0
0 daca y = 0.
28. Sa se studieze diferent
,
iabilitatea funct
,
iei f : R
2
R, denite prin
f(x, y) =
_
x
3
y
3
x
2
+y
2
daca (x, y) = (0, 0)
0 daca (x, y) = (0, 0).
29. Sa se studieze diferent
,
iabilitatea funct
,
iei f : R
2
R, denite prin
f(x, y) =
_
(x
4
y
2
)
2
x
2
+y
2
daca x
4
> y
2
0 daca x
4
y
2
.
30. Fie f : R
2
R funct
,
ia denita n felul urmator:
f(x, y) = y x
2
daca y x
2
f(x, y) =
y
2
x
2
y daca 0 y < x
2
f(x, y) = f(x, y) daca y < 0.
Sa se demonstreze ca f este diferent
,
iabila pe R
2
, dar nu este de clasa C
1
pe R
2
.
31. Sa se studieze diferent
,
iabilitatea funct
,
iei f : R
2
R, denite prin
f(x, y) =
_
min {x, y} +
(x
2
y
2
)
2
x
2
+y
2
daca x = y
x daca x = y.
32. Fie p, q N, iar f : R
2
R funct
,
ia denita prin
f(x, y) =
_
x
p
y
q
x
2
xy+y
2
daca (x, y) = (0, 0)
0 daca (x, y) = (0, 0).
82 3 Calcul diferent
,
ial n R
n
Sa se determine pentru ce valori ale numerelor p s
,
i q funct
,
ia f este:
a) continua pe R
2
;
b) diferent
,
iabila pe R
2
;
c) de clasa C
1
;
d) derivabila n (0, 0) dupa orice direct
,
ie v R
2
.
33. Sa se studieze diferent
,
iabilitatea funct
,
iei f = (f
1
, f
2
) : R
2
R
2
, denite
prin
f
1
(x, y) = (x 1)e
y
,
f
2
(x, y) =
_
x
3
+y
3
|x|+|y|
daca (x, y) = (0, 0)
0 daca (x, y) = (0, 0),
n punctul (0, 0).
34. Sa se studieze diferent
,
iabilitatea funct
,
iei f = (f
1
, f
2
) : R
3
R
2
, denite
prin
f
1
(x
1
, x
2
, x
3
) =
_
|x
1
x
2
x
3
| cos
1
x
1
daca x
1
= 0
0 daca x
1
= 0,
f
2
(x
1
, x
2
, x
3
) = (x
1
+x
2
+x
3
)e
x
1
2x
2
3x
3
,
n punctul (0, 0, 0).
35. Sa se studieze diferent
,
iabilitatea funct
,
iei f = (f
1
, f
2
) : R
2
R
2
, denite
prin
f
1
(x, y) = e
x
2
y
, f
2
(x, y) =
_
x
2
sin
1
x
2
+y
2
daca x = 0
y
2
daca x = 0,
n punctul (0, 0).
36. Sa se studieze diferent
,
iabilitatea funct
,
iei f = (f
1
, f
2
) : R
2
R
2
, denite
prin
f
1
(x, y) = |xy|, f
2
(x, y) =
_
x
2
cos

2y
daca y = 0
0 daca y = 0,
n punctul (0, 0).
3.7 Probleme 83
37. Sa se studieze diferent
,
iabilitatea funct
,
iei f = (f
1
, f
2
) : R
2
R
2
, denite
prin
f
1
(x, y) =
xy
1 +|xy|
,
f
2
(x, y) =
_
x
2
(x+y)
|x|+|y|
daca (x, y) = (0, 0)
0 daca (x, y) = (0, 0),
n punctul (0, 0).
38. Sa se studieze diferent
,
iabilitatea funct
,
iei f : R
n
R, denite prin
f(x
1
, . . . , x
n
) =
_
x
1
x
n
x
2
1
++x
2
n
daca (x
1
, . . . , x
n
) = 0
n
0 daca (x
1
, . . . , x
n
) = 0
n
.
39. Fie R
nn
spat
,
iul liniar al matricelor reale de tipul n n, identicat n
modul uzual cu spat iu euclidian R
n
2
. (Norma unei matrice oarecare
X = (x
ij
)
1i,jn
din R
nn
este data de X
2
=
n

i,j=1
x
2
ij
.) Fie apoi
f : R
nn
R
nn
funct
,
ia denita prin f(X) = X
2
. Sa se determine
diferent
,
iala df a funct
,
iei f.
Berkeley 1978, 1999
40. Fie A R
2
deschisa nevida, iar f : A R o funct
,
ie derivabila part
,
ial pe
A. Sa se demonstreze ca daca derivatele part
,
iale ale lui f sunt marginite
pe un dreptunghi [a, b] [c, d] A, atunci f este uniform continua pe
[a, b] [c, d].
41. Fie A R
2
deschisa nevida, a < b s
,
i c < d numere reale astfel ca
[a, b] [c, d] A, iar f : A R o funct
,
ie care ndeplines
,
te urmatoarele
condit
,
ii:
(i) pentru orice y [c, d], funct
,
ia f(, y) este continua pe [a, b];
(ii) f este derivabila part
,
ial n raport cu variabila y pe A;
(iii) funct
,
ia
f
y
este marginita pe [a, b] [c, d].
Sa se demonstreze ca f este continua pe [a, b] [c, d].
84 3 Calcul diferent
,
ial n R
n
42. Fie f : R
2
R o funct
,
ie derivabila part
,
ial pe R
2
s
,
i cu proprietatea ca cel
put
,
in una dintre derivatele sale part
,
iale este continua. Sa se demonstreze
ca:
a) funct
,
ia g : R R, denita prin g(t) = f(t, t), este derivabila s
,
i
t R : g

(t) =
f
x
(t, t) +
f
y
(t, t).
b) daca f(0, 0) = 0 s
,
i
(x, y) R
2
:

f
x
(x, y)

2|x y| s
,
i

f
y
(x, y)

2|x y|,
atunci |f(5, 4)| 1.
Olimpiada student
,
easca, U.R.S.S.
43. Fie f : R
n
R
m
o funct
,
ie care satisface urmatoarele condit
,
ii:
(i) f este derivabila n 0
n
dupa orice direct
,
ie v R
n
;
(ii) funct
,
ia : R
n
R
m
, denita prin (v) = f

(0
n
; v) este liniara;
(iii) exista 0 as
,
a ncat
x, x

R
n
: f(x) f(x

) x x

.
Sa se demonstreze ca f este diferent
,
iabila n 0
n
.
Olimpiada student
,
easca, U.R.S.S.
44. Fie f : R
n
R o funct
,
ie derivabila part
,
ial pe R
n
.
a) Pentru n = 2, sa se demonstreze ca daca cel put
,
in una dintre derivatele
part
,
iale ale lui f este continua pe R
2
, atunci f este diferent
,
iabila pe R
2
.
b) Sa se generalizeze armat
,
ia de la a) pentru n 2 arbitrar.
Concursul student
,
esc Traian Lalescu, etapa locala 2001
45. Sa se demonstreze ca daca f : R R este o funct
,
ie de clasa C
2
, atunci
funct
,
ia F : R
2
R, denita prin
F(x, y) =
_
f(y)f(x)
yx
daca x = y
f

(x) daca x = y,
este de clasa C
1
.
3.8 Operat
,
ii cu funct
,
ii diferent
,
iabile 85
3.8 Operat
,
ii cu funct
,
ii diferent
,
iabile
3.8.1 Teorema. Fie A o submult
,
ime a lui R
n
, a int A, , R, iar
f, g : A R
m
funct
,
ii diferent
,
iabile n punctul a. Atunci funct
,
ia f +g este
diferent
,
iabila n a s
,
i au loc egalitat
,
ile
d(f +g)(a) = df(a) +dg(a),
J(f +g)(a) = J(f)(a) +J(g)(a).
Demonstrat
,
ie. Notam F := f + g s
,
i := df(a) + dg(a). Vom arata ca
F este diferent
,
iabila n a s
,
i ca dF(a) = . Deoarece f s
,
i g sunt diferent
,
iabile
n a, avem
lim
xa
1
x a
_
f(x) f(a) df(a)(x a)

= 0
m
s
,
i
lim
xa
1
x a
_
g(x) g(a) dg(a)(x a)

= 0
m
.
Intrucat pentru orice x A\ {a} avem
1
x a
_
F(x) F(a) (a)(x a)

=
1
x a
_
f(x) f(a) df(a)(x a)

+
1
x a
_
g(x) g(a) dg(a)(x a)

,
deducem ca
lim
xa
1
x a
_
F(x) F(a) (a)(x a)

= 0
m
.
Prin urmare, F este diferent
,
iabila n a s
,
i dF(a) = , deci prima egalitate din
enunt
,
are loc. Pe de alta parte, n baza teoremelor 3.6.4 s
,
i 3.1.6 avem
J(F)(a) =
_
dF(a)

=
_
df(a) +dg(a)

= [df(a)] +[dg(a)]
= J(f)(a) +J(g)(a).
3.8.2 Teorema. Fie A R
n
, a int A, B R
m
, g : A B o funct
,
ie
diferent
,
iabila n punctul a, cu proprietatea g(a) int B, iar f : B R
p
o
86 3 Calcul diferent
,
ial n R
n
funct
,
ie diferent
,
iabila n punctul g(a). Atunci funct
,
ia f g este diferent
,
iabila
n a s
,
i au loc egalitat
,
ile
d(f g)(a) = df(g(a)) dg(a),
J(f g)(a) = J(f)(g(a)) J(g)(a).
Demonstrat
,
ie. Notam F := f g, b := g(a), := dg(a) s
,
i := df(b). Aplicand
propozit
,
ia 3.4.3, rezulta existent
,
a funct
,
iilor : A R
m
s
,
i : B R
p
, cu
urmatoarele proprietat
,
i:
lim
xa
(x) = 0
m
, (1)
g(x) = g(a) +(x a) +x a (x) oricare ar x A, (2)
lim
ub
(u) = 0
p
, (3)
f(u) = f(b) +(u b) +u b (u) oricare ar u B. (4)
Fara a restrange generalitatea, putem presupune ca (a) = 0
m
s
,
i (b) = 0
p
,
adica s
,
i sunt continue n a s
,
i respectiv b. Conform relat
,
iei (4), pentru
orice x A avem
F(x) = f(g(x)) = f(b) +(g(x) b) +g(x) b (g(x))
= f(g(a)) +(g(x) g(a)) +g(x) g(a) (g(x)).
T
,
inand seama de (2), obt
,
inem
F(x) = F(a) +
_
(x a) +x a (x)
_
+g(x) g(a) (g(x))
= F(a) +
_

_
(x a) +x a((x)) +g(x) g(a) (g(x)).
Fie : A R
p
funct
,
ia denita prin
(x) := ((x)) +
g(x) g(a)
x a
(g(x)) daca x A\ {a}
(a) := 0
p
.
Avem atunci
(5) F(x) = F(a) +
_

_
(x a) +x a (x) oricare ar x A.
Vom mai dovedi ca
(6) lim
xa
(x) = 0
p
.
3.8 Operat
,
ii cu funct
,
ii diferent
,
iabile 87
Intr-adevar, pentru orice x A\ {a} avem
(x) ((x)) +
g(x) g(a)
x a
(g(x))
= ((x)) +
_
_
(x a) +x a (x)
_
_
x a
(g(x))
(x) +
x a +x a (x)
x a
(g(x)),
deci
(7) (x) (x) +
_
+(x)
_
(g(x)) oricare ar x A.
Deoarece g este diferent
,
iabila n punctul a, ea este continua n a. Cum este
continua n b := g(a), rezulta ca g este continua n a, deci
lim
xa
(g(x)) = (g(a)) = (b) = 0
p
.
Aceasta egalitate, mpreuna cu (1), implica
lim
xa
_
(x) +
_
+(x)
_
(g(x))
_
= 0.
T
,
inand seama de (7), deducem ca lim
xa
(x) = 0, deci (6) are loc. In baza
propozit
,
iei 3.4.3, din (5) s
,
i (6) urmeaza ca F este diferent
,
iabila n a s
,
i
dF(a) = = df(g(a)) dg(a).
In plus, n baza teoremelor 3.6.4 s
,
i 3.1.7 avem
J(F)(a) =
_
dF(a)

=
_
df(g(a)) dg(a)

= [df(g(a))] [dg(a)]
= J(f)(g(a)) J(g)(a).
3.8.3 Observat
,
ie. Fie A o submult
,
ime a lui R
n
, a int A, B o submult
,
ime
a lui R
m
, g = g(x
1
, . . . , x
n
) : A B o funct ie diferent
,
iabila n punctul
a, cu proprietatea g(a) int B, iar f = f(u
1
, . . . , u
m
) : B R o funct
,
ie
diferent
,
iabila n g(a). Atunci avem J(f g)(a) = J(f)(g(a)) J(g)(a), adica
_
(f g)
x
1
(a)
(f g)
x
n
(a)
_
=
_
f
u
1
(g(a))
f
u
m
(g(a))
_

_
_
_
g
1
x
1
(a)
g
1
x
n
(a)
.
.
.
.
.
.
g
m
x
1
(a)
g
m
x
n
(a)
_
_
_
.
88 3 Calcul diferent
,
ial n R
n
Aceasta egalitate implica
(f g)
x
j
(a) =
f
u
1
(g(a))
g
1
x
j
(a) +
f
u
2
(g(a))
g
2
x
j
(a)
+ +
f
u
m
(g(a))
g
m
x
j
(a)
pentru ecare j {1, . . . , n}. Scris scurt
(f g)
x
j
=
m

i=1
f
u
i

g
i
x
j
, j = 1, . . . , n.
Aceasta scriere nu este riguroasa deoarece derivatele part iale
(f g)
x
j
s
,
i
g
i
x
j
se evalueaza n punctul a, pe cand derivatele part
,
iale
f
u
i
se evalueaza n
punctul g(a).
3.9 Probleme
1. Sa se demonstreze ca funct
,
ia f : R
2
R, denita prin
f(x, y) =
_
x
2
+y
2
+ exp
_
1
x
2
+y
2
1
_
daca x
2
+y
2
< 1
1 + ln
_
x
2
+y
2
_
daca x
2
+y
2
1,
este diferent
,
iabila s
,
i sa se determine diferent iala sa.
2. Fie > 0 s
,
i F : R (0, ) R funct
,
ia denita prin
F(x, y) =
x
2

y
0
e
t
2
dt.
Sa se arate ca
(x, y) R (0, ) :

2
F
x
2
(x, y) =
F
y
(x, y).
3. Fie A R
n
, a int A, iar f : A R s
,
i g : A R
m
funct
,
ii diferent
,
iabile
n punctul a. Sa se demonstreze ca funct
,
ia F : A R
m
, denita prin
F(x) = f(x)g(x), este diferent
,
iabila n a s
,
i, pentru orice h R
n
, are loc
egalitatea
dF(a)(h) = df(a)(h) g(a) +f(a) dg(a)(h).
3.9 Probleme 89
4. Fie A = { x R
n
| x < 1 }. Sa se determine diferent
,
iala funct
,
iei
F : A R
n
, denite prin
F(x) =
1

1 x
2
x,
ntr-un punct oarecare x A.
5. Fie A R
n
, a int A s
,
i f, g : A R
m
funct
,
ii diferent
,
iabile n a. Sa se
demonstreze ca funct
,
ia F : A R, denita prin F(x) = f(x), g(x) ,
este diferent
,
iabila n punctul a s
,
i, pentru orice h R
n
, are loc egalitatea
dF(a)(h) = df(a)(h), g(a) + f(a), dg(a)(h) .
6. Fie f = f(u, v) : R
2
R o funct
,
ie diferent
,
iabila pe R
2
, iar F : R
3
R
funct
,
ia denita prin
F(x, y, z) = f(e
x
ch (y +z), e
x
sh (y +z)).
Sa se determine, n funct
,
ie de derivatele part
,
iale ale lui f, derivatele
part
,
iale de ordinul ntai ale lui F.
7. Fie f = f(u, v, w) : R
3
R o funct
,
ie diferent
,
iabila pe R
3
, iar F : R
2

R funct
,
ia denita prin
F(x, y) = f(3x + 2y, x
2
+y
2
, 2x
3
y
3
).
Sa se determine, n funct
,
ie de derivatele part
,
iale ale lui f, derivatele
part
,
iale de ordinul ntai ale lui F.
8. Fie f = f(u, v) : R
2
R o funct
,
ie diferent
,
iabila pe R
2
, iar F : R
2
R
funct
,
ia denita prin F(x, y) = sin(y + f(y
2
, x)). Sa se determine, n
funct
,
ie de derivatele part
,
iale ale lui f, derivatele part
,
iale de ordinul
ntai ale lui F.
9. Fie f : R
3
R
2
o funct
,
ie diferent
,
iabila pe R
3
, iar F : R
2
R
2
funct
,
ia
denita prin F(x, y) = f (cos x + sin y, sin x + cos y, e
xy
).
a) Sa se demonstreze ca daca f este de clasa C
1
pe R
3
, atunci F este de
clasa C
1
pe R
2
.
b) S
,
tiind ca
J(f)(1, 1, 1) =
_
1 3 4
2 1 3
_
,
sa se determine dF
_

2
,

2
_
.
90 3 Calcul diferent
,
ial n R
n
10. Fie f : R
2
R
3
o funct
,
ie diferent
,
iabila pe R
2
, iar F : R
3
R
3
funct
,
ia
denita prin
F(x, y, z) = f(sin x 2 sin y + 3 sin z, cos x 2 cos y + 3 cos z).
a) Sa se demonstreze ca daca f este de clasa C
1
pe R
2
, atunci F este de
clasa C
1
pe R
3
.
b) Sa se determine J(F)
_

2
,

2
,

2
_
s
,
i dF
_

2
,

2
,

2
_
(1, 0, 1), daca
J(f)(2, 0) =
_
_
1 2
3 0
4 1
_
_
.
11. Fie f = f(u, v) : R
2
R o funct
,
ie diferent
,
iabila pe R
2
, cu proprietatea
(u, v) R
2
:
f
u
(u, v) =
f
v
(u, v),
iar F : R
2
R funct
,
ia denita prin F(x, y) = f
_
x +y
2
,
x y
2
_
. Sa se
demonstreze ca
(x, y) R
2
:
F
y
(x, y) = 0.
12. Fie f = f(u, v) : R
2
R o funct
,
ie diferent
,
iabila pe R
2
, cu proprietatea
(u, v) R
2
: v
f
u
(u, v) = u
f
v
(u, v),
e mult
,
imea A = { (x, y) R
2
| x
2
< y } s
,
i F : A R funct
,
ia denita
prin F(x, y) = f(x,

y x
2
). Sa se demonstreze ca
(x, y) A :
F
x
(x, y) = 0.
13. Fie f = f(u, v) : R
2
R o funct
,
ie diferent
,
iabila pe R
2
, cu proprietatea
(u, v) R
2
: (u +v)
f
u
(u, v) = (u v)
f
v
(u, v),
iar F : R
2
R funct
,
ia denita prin F(x, y) = f(e
x
cos y, e
x
sin y). Sa se
demonstreze ca
(x, y) R
2
:
F
x
(x, y) =
F
y
(x, y).
3.9 Probleme 91
14. Fie f = f(x, y) : R
2
R s
,
i F = F(u, v) : R
2
R funct
,
ii diferent
,
iabile
pe R
2
, cu proprietatea F
_
x,
y
x
_
= f(x, y) pentru orice (x, y) R
2
cu
x = 0. Sa se demonstreze ca daca
(x, y) R
2
: x
f
x
(x, y) +y
f
y
(x, y) = f(x, y),
atunci
x
F
u
_
x,
y
x
_
= F
_
x,
y
x
_
pentru orice (x, y) R
2
cu x = 0.
15. Fie f = f(x, y) : R
2
(0, ) s
,
i F = F(u, v) : R
2
R funct
,
ii
diferent
,
iabile pe R
2
, cu proprietatea
F
_
x
2
+y
2
,
1
x
+
1
y
_
= ln f(x, y) (x +y)
pentru orice (x, y) R
2
cu x = 0 s
,
i y = 0. Sa se demonstreze ca daca
(x, y) R
2
: y
f
x
(x, y) x
f
y
(x, y) = (y x)f(x, y),
atunci
F
v
_
x
2
+y
2
,
1
x
+
1
y
_
= 0
pentru orice (x, y) R
2
cu x = 0, y = 0 s
,
i x = y.
16. Fie a > 0, A = { (x, y) R
2
| 0 < x
2
+y
2
< a
2
} s
,
i f = f(x, y) : A R
o funct
,
ie diferent
,
iabila.
a) Sa se determine, n funct
,
ie de derivatele part
,
iale ale lui f, derivatele
part
,
iale de ordinul ntai ale funct
,
iei F : (0, a) R R, denite prin
F(, ) = f( cos , sin ).
b) Folosind eventual rezultatul de la a), sa se demonstreze ca nu exista
nici o funct
,
ie diferent
,
iabila f : A R, care satisface
(x, y) A : y
f
x
(x, y) x
f
y
(x, y) = 1.
17. Sa se demonstreze ca nu exista nici o funct
,
ie f : R
2
R, continua pe
R
2
, diferent
,
iabila pe R
2
\ {(0, 0)} s
,
i care sa satisfaca
(x, y) R
2
\ {(0, 0)} : y
f
x
(x, y) +x
f
y
(x, y) = 1.
92 3 Calcul diferent
,
ial n R
n
18. Folosind eventual coordonatele polare, sa se determine funct
,
iile diferen-
t
,
iabile f : (0, )
2
R, care satisfac
(x, y) (0, )
2
: x
f
x
(x, y) +y
f
y
(x, y) =
x

x
2
+y
2
.
Fie p un numar real. O funct
,
ie f : R
n
R se numes
,
te p-omogena daca
t > 0, (x
1
, . . . , x
n
) R
n
: f(tx
1
, . . . , tx
n
) = t
p
f(x
1
, . . . , x
n
).
19. (L. Euler) Sa se demonstreze ca daca f : R
n
R este o funct
,
ie diferen-
t
,
iabila, atunci urmatoarele armat
,
ii sunt echivalente:
1

f este p-omogena.
2

Pentru orice (x
1
, . . . , x
n
) R
n
are loc egalitatea
x
1
f
x
1
(x
1
, . . . , x
n
) + +x
n
f
x
n
(x
1
, . . . , x
n
) = p f(x
1
, . . . , x
n
).
20. Fie f : R
2
R o funct
,
ie de clasa C
1
cu proprietatea f(0, 0) = 0. Sa se
demonstreze ca pentru orice (x, y) R
2
are loc egalitatea
f(x, y) = x

1
0
f
x
(tx, ty)dt +y

1
0
f
y
(tx, ty)dt.
Olimpiada student
,
easca, U.R.S.S.
21. (Exemplu de funct
,
ii derivabile part
,
ial, a caror compusa nu este derivabila
part
,
ial) Fie g : R
2
R
2
funct
,
ia denita prin g(x, y) = (x
2
+y
2
, x
2
+y
2
),
iar f : R
2
R funct ia denita prin
f(x, y) =
_
xy
x
2
+y
2
daca (x, y) = (0, 0)
0 daca (x, y) = (0, 0).
Sa se demonstreze ca g este derivabila part
,
ial n a = (0, 0), f este deriv-
abila part
,
ial n b = g(a) = (0, 0), dar f g nu este derivabila part
,
ial n
punctul a.
3.10 Diferent
,
iabilitatea funct
,
iei inverse 93
3.10 Diferent
,
iabilitatea funct
,
iei inverse
3.10.1 Observat
,
ie. Fie A s
,
i B submult
,
imi ale lui R
n
s
,
i f : A B o funct
,
ie
bijectiva. Daca a int A este un punct n care f este diferent
,
iabila, iar
f(a) int B, atunci nu rezulta, n general, ca f
1
este diferent
,
iabila n f(a).
De exemplu, funct
,
ia f : R R, f(x) := x
3
este bijectiva s
,
i derivabila
(deci diferent
,
iabila) pe R, dar inversa ei f
1
: R R, f
1
(y) :=
3

y nu este
derivabila (deci nici diferent iabila) n punctul 0 = f(0).
3.10.2 Teorema. Fie A s
,
i B submult
,
imi ale lui R
n
, a int A, b int B, iar
f : A B o funct
,
ie bijectiva care ndeplines
,
te urmatoarele condit
,
ii:
(i) f este diferent
,
iabila n punctul a;
(ii) f(a) = b;
(iii) f
1
este diferent
,
iabila n punctul b.
Atunci aplicat
,
ia df(a) este bijectiva, matricea J(f)(a) este inversabila s
,
i au
loc urmatoarele egalitat
,
i:
df(a)
1
= df
1
(b),
J(f)(a)
1
= J(f
1
)(b).
Demonstrat
,
ie. Avem f f
1
= 1
B
s
,
i f
1
f = 1
A
. Aplicand teorema 3.8.2,
rezulta ca
d1
B
(b) = df(a) df
1
(b) s
,
i d1
A
(a) = df
1
(b) df(a).
Intrucat d1
A
(a) = d1
B
(b) = 1
R
n, deducem ca df(a) este bijectiva s
,
i inversa ei
este df(a)
1
= df
1
(b). Pe de alta parte, din egalitatea df(a) df
1
(b) = 1
R
n,
rezulta
_
df(a) df
1
(b)

= I
n
. In baza teoremei 3.1.7, avem
_
df(a)

_
df
1
(b)

= I
n
,
adica J(f)(a) J(f
1
)(b) = I
n
. Drept urmare, matricea J(f)(a) este in-
versabila s
,
i inversa ei este matricea J(f
1
)(b).
3.10.3 Observat
,
ie. Din teorema 3.10.2 rezulta ca o condit
,
ie necesara ca
o funct
,
ie bijectiva f : A B, diferent iabila n punctul a, sa aiba inversa
diferent iabilan punctul b := f(a), este ca diferent
,
iala lui f n a sa e bijectiva.
Aceasta condit
,
ie, mpreuna cu continuitatea lui f
1
n b este s
,
i sucienta, dupa
cum arata teorema urmatoare.
94 3 Calcul diferent
,
ial n R
n
3.10.4 Teorema. Fie A s
,
i B submult
,
imi ale lui R
n
, a int A, b int B, iar
f : A B o funct
,
ie bijectiv a, diferent
,
iabila n a s
,
i cu proprietatea f(a) = b.
Atunci urmatoarele armat
,
ii sunt echivalente:
1

f
1
este diferent
,
iabila n b.
2

df(a) este bijectiv a s


,
i f
1
este continua n b.
Demonstrat
,
ie. 1

Rezulta din teorema 3.10.2 s


,
i teorema 3.4.4.
2

Notam := df(a). Conform propozit


,
iei 3.4.3, exista o funct
,
ie
: A R
n
care ndeplines
,
te urmatoarele condit ii:
f(x) = f(a) +(x a) +x a (x) oricare ar x A, (1)
lim
xa
(x) = 0
n
.
Fara a restrange generalitatea, putem presupune ca (a) = 0
n
, adica este
continua n a. Punand x := f
1
(y) n (1), deducem ca pentru orice y B
avem
f(f
1
(y)) = b +
_
f
1
(y) f
1
(b)
_
+f
1
(y) f
1
(b) (f
1
(y)),
de unde

_
f
1
(y) f
1
(b)
_
= y b f
1
(y) f
1
(b) (f
1
(y)).
Aplicand funct
,
ia
1
ambilor membri, rezulta ca
(2) f
1
(y) = f
1
(b) +
1
(y b) f
1
(y) f
1
(b)
1
_

_
f
1
(y)
_
_
oricare ar y B. Fie : B R
n
funct
,
ia denita prin
(y) :=
f
1
(y) f
1
(b)
y b

1
_

_
f
1
(y)
_
_
daca y B \ {b}
(b) := 0
n
.
Avem atunci
(3) f
1
(y) = f
1
(b) +
1
(y b) +y b (y) oricare ar y B.
Vom mai dovedi ca
(4) lim
yb
(y) = 0
n
.
3.10 Diferent
,
iabilitatea funct
,
iei inverse 95
Deoarece este continua n punctul a = f
1
(b) si f
1
este continua n b,
rezulta ca f
1
este continua n punctul b. Cum
_
f
1
_
(b) = (a) = 0
n
,
exista un r > 0 as
,
a ncat B(b, r) B s
,
i
_
_

_
f
1
(y)
__
_
<
1
2
1

oricare ar y B(b, r).


T
,
inand seama de (3), deducem ca pentru orice y B(b, r) avem
f
1
(y) f
1
(b) =
_
_
_
1
(y b) f
1
(y) f
1
(b)
1
_

_
f
1
(y)
_
__
_
_

1
(y b) +f
1
(y) f
1
(b)
_
_
_
1
_

_
f
1
(y)
_
__
_
_

1
y b +f
1
(y) f
1
(b)
1

_
_

_
f
1
(y)
__
_

1
y b +
1
2
f
1
(y) f
1
(b),
deci
1
2
f
1
(y) f
1
(b)
1
y b oricare ar y B(b, r).
Prin urmare, oricare ar y B(b, r) \ {b} avem
(y) =
f
1
(y) f
1
(b)
y b
_
_
_
1
_

_
f
1
(y)
_
__
_
_
2
1

1

_
_

_
f
1
(y)
__
_
,
adica
(y) 2
_
_

1
_
_
2
_
_

_
f
1
(y)
__
_
pentru orice y B(b, r).
Deoarece lim
yb

_
f
1
(y)
_
=
_
f
1
(b)
_
= (a) = 0
n
, rezulta ca egalitatea (4)
are loc. Din (3) s
,
i (4) urmeaza ca f
1
este diferent
,
iabila n punctul b s
,
i ca
df
1
(b) =
1
= df(a)
1
.
3.10.5 Observat
,
ie. Teorema precedenta are aplicabilitate practica redusa.
Intr-adevar, a demonstra diferent
,
iabilitatea funct
,
iei f
1
n punctul b pe baza
implicat
,
iei 2

din teorema 3.10.4 presupune a dovedi ca f


1
este continua
n b. In general nsa, funct
,
ia f
1
nu este cunoscuta.
96 3 Calcul diferent
,
ial n R
n
3.11 Teoreme de medie pentru funct
,
ii de variabila
vectoriala
3.11.1 Denit
,
ie (extreme). Fie A R
n
o mult
,
ime nevida s
,
i f : A R o
funct
,
ie. Un punct a A se numes
,
te punct de minim local (respectiv punct de
maxim local ) al lui f daca exista V V(a) astfel ncat pentru orice x V A
sa avem
(1) f(a) f(x) (respectiv f(a) f(x)).
Daca inegalitatea (1) are loc pentru orice x A, atunci a se numes
,
te punct
de minim global (respectiv punct de maxim global ).
Punctele de minim local s
,
i punctele de maxim local se numesc puncte de
extrem local ale lui f. Punctele de minim global s
,
i punctele de maxim global
se numesc puncte de extrem global ale lui f.
3.11.2 Teorema (P. Fermat). Fie A R
n
o mult
,
ime nevida, f : A R o
funct
,
ie s
,
i a un punct care ndeplines
,
te urmatoarele condit
,
ii:
(i) a int A;
(ii) a este punct de extrem local al lui f;
(iii) f este derivabila part
,
ial n punctul a.
Atunci f(a) = 0
n
, adica
f
x
j
(a) = 0 oricare ar j {1, . . . , n}.
Demonstrat
,
ie. Fie a := (a
1
, . . . , a
n
). Deoarece a int A, exista un > 0 astfel
ca [a
1
, a
1
+ ] [a
n
, a
n
+ ] A. Fixam un j {1, . . . , n} s
,
i
consideram funct
,
ia g : [a
j
, a
j
+] R, denita prin
g(t) := f(a
1
, . . . , a
j1
, t, a
j+1
, . . . , a
n
).
Deoarece a este punct de extrem local al lui f, urmeaza ca a
j
este punct de
extrem local al lui g (avand aceeas
,
i natura ca s
,
i a). Evident, g este derivabila
n a
j
s
,
i g

(a
j
) =
f
x
j
(a). Conform teoremei lui Fermat pentru funct
,
ii reale de
variabila reala, avem g

(a
j
) = 0, deci
f
x
j
(a) = 0.
3.11.3 Observat
,
ii. 1

Daca ipoteza (iii) din teorema 3.11.2 se nlocuies


,
te cu
(iii) f este diferent
,
iabila n punctul a,
3.11 Teoreme de medie pentru funct
,
ii de variabila vectoriala 97
atunci concluzia teoremei devine df(a) = 0.
2

Daca f este diferent


,
iabila n punctul a, iar df(a) = 0, atunci a se
numes
,
te punct critic (sau punct stat
,
ionar) al lui f. Din teorema lui Fermat
rezulta ca punctele de extrem local din int A ale unei funct
,
ii diferent
,
iabile
f : A R se aa printre punctele sale critice. In general nsa, nu orice punct
critic este punct de extrem local. Cu alte cuvinte, condit
,
ia f(a) = 0
n
este
necesara dar nu s
,
i sucienta ca a sa e punct de extrem local al lui f.
Fie, spre exemplu, funct
,
ia f : R
2
R, denita prin f(x, y) = x
2
y
2
.
Avem f(0, 0) = (0, 0), deci a := (0, 0) este punct critic pentru f. Se vede
imediat nsa ca a nu este punct de extrem local al lui f. Suprafat
,
a de ecuat
,
ie
z = f(x, y) = x
2
y
2
are drept imagine un paraboloid hiperbolic, care n
vecinatatea originii ,,arata ca o s
,
a (a se vedea gura 3.11.1). Din acest motiv,
punctele critice ale unei funct
,
ii care nu sunt puncte de extrem local sunt numite
puncte s
,
a.
Figura 3.11.1: Gracul suprafet
,
ei de ecuat
,
ie z = x
2
y
2
.
3

Fie A o submult
,
ime compacta cu interiorul nevid a spat
,
iului R
n
, iar
f : A R o funct
,
ie continua pe A s
,
i diferent
,
iabila pe int A. Atunci, conform
teoremei lui Weierstrass, f este marginita s
,
i s
,
i atinge marginile, deci putem
considera numerele reale m := min f(A) s
,
i M := max f(A). Intrucat s
,
i
mult
,
imea bd A este compacta, tot n baza teoremei lui Weierstrass, putem
considera s
,
i numerele reale m
1
:= min f(bd A) s
,
i M
1
:= max f(bd A). Fie
apoi C mult
,
imea punctelor critice ale lui f din int A. Presupunand ca C este
nita, notam m
2
:= min f(C) s
,
i M
2
:= max f(C). Teorema 3.11.2 garanteaza
98 3 Calcul diferent
,
ial n R
n
atunci ca
m = min {m
1
, m
2
} s
,
i M = max {M
1
, M
2
}.
3.11.4 Denit
,
ie (mult
,
imi convexe). O mult
,
ime A R
n
se numes
,
te convexa
daca pentru orice puncte x, x

A s
,
i orice t [0, 1] avem (1 t)x + tx

A
(a se vedea gura 3.11.2).
A
x
x
(a) mult
,
ime convex a
A
x
x
(b) mult
,
ime neconvex a
Figura 3.11.2: Exemplu de mult
,
ime convexa s
,
i respectiv neconvexa
3.11.5 Lema. Fie A R
n
o mult
,
ime convexa, deschisa, nevida, f : A R
m
o funct
,
ie diferent
,
iabila pe A, e a s
,
i b puncte din A s
,
i e F : [0, 1] R
m
funct
,
ia denita prin F(t) := f((1t)a+tb). Atunci F este derivabila pe [0, 1]
s
,
i
(2) F

(t) = df((1 t)a +tb)(b a) oricare ar t [0, 1].


Demonstrat
,
ie. Fie t
0
[0, 1] xat arbitrar s
,
i e a := (1t
0
)a+t
0
b, v := ba.
Deoarece A este convexa si deschisa, a este punct interior lui A. Avem
lim
tt
0
1
t t
0
_
F(t) F(t
0
)

= lim
s0
1
s
_
F(t
0
+s) F(t
0
)

= lim
s0
1
s
_
f((1 t
0
s)a + (t
0
+s)b) f((1 t
0
)a +t
0
b)

= lim
s0
1
s
_
f( a +sv) f( a)

= f

( a; v) = df( a)(v).
Prin urmare, F este derivabila n t
0
s
,
i F

(t
0
) = df((1 t
0
)a +t
0
b)(b a).
3.12 Probleme 99
3.11.6 Teorema (teorema de medie pentru funct
,
ii reale de variabila vecto-
riala). Fie A R
n
o mult
,
ime deschisa, convexa, nevida si f : A R o funct
,
ie
diferent
,
iabila pe A. Atunci oricare ar punctele a, b A, exista un (0, 1)
as
,
a ncat notand c := (1 )a +b sa avem
f(b) f(a) = df(c)(b a).
Demonstrat
,
ie. Fie a, b A xate s
,
i e F : [0, 1] R funct
,
ia denita prin
F(t) := f((1 t)a + tb). Conform lemei 3.11.5, funct
,
ia F este derivabila
pe [0, 1]. Aplicand lui F teorema de medie (a lui Lagrange) pentru funct
,
ii
reale de variabila reala, rezulta existent
,
a unui punct (0, 1) astfel ncat
F(1) F(0) = F

(). Notand c := (1 )a + b, din relat


,
ia (2) rezulta ca
f(b) f(a) = df(c)(b a).
3.11.7 Teorema (teorema de medie pentru funct
,
ii vectoriale de variabila
vectoriala). Fie A R
n
deschisa convexa nevida s
,
i f : A R
m
o funct
,
ie
diferent
,
iabila pe A. Atunci oricare ar punctele a, b A, exista un (0, 1)
as
,
a ncat notand c := (1 )a +b sa avem
f(b) f(a) df(c) b a.
Demonstrat
,
ie. Fie a, b A xate s
,
i e F : [0, 1] R
m
funct
,
ia denita prin
F(t) := f((1 t)a + tb). Conform lemei 3.11.5, funct
,
ia F este derivabila pe
[0, 1]. Aplicand lui F teorema de medie pentru funct
,
ii vectoriale de variabila
reala (teorema 3.3.3), rezulta existent
,
a unui punct (0, 1) astfel ncat sa
avem F(1) F(0) F

(). Notand c := (1 )a + b, din relat


,
ia (2)
rezulta ca
f(b) f(a) df(c)(b a) df(c) b a.
3.12 Probleme
1. (Teorema lui Rolle pentru funct
,
ii reale de variabila vectoriala) Fie A o
submult
,
ime compacta cu interiorul nevid a spat
,
iului R
n
s
,
i f : A R o
funct
,
ie care ndeplines
,
te urmatoarele condit
,
ii:
(i) f este continua pe A;
(ii) f este diferent
,
iabila pe int A;
(iii) f este constanta pe bd A.
Sa se demonstreze ca exista un punct c int A astfel ca df(c) = 0.
100 3 Calcul diferent
,
ial n R
n
2. Cu ajutorul funct
,
iei f : [0, ] R
2
, f(x) = (sin x, sin 2x), sa se demon-
streze ca teorema lui Rolle nu ramane adevarata pentru funct
,
ii vectori-
ale.
3. Fie x, y, z [0, ) as
,
a ncat x +y +z = 1. Sa se demonstreze ca
0 xy +yz +zx 2xyz
7
27
.
OIM, Praga 1984
4. Fie x, y, z [0, ) as
,
a ncat x +y +z = 1. Sa se demonstreze ca
4(xy +yz +zx) 9xyz + 1.
5. Fie x, y, z [0, ) as
,
a ncat x +y +z = 1. Sa se demonstreze ca
1
4
x
3
+y
3
+z
3
+ 6xyz 1.
Olimpiada student
,
easca, U.R.S.S.
6. Fie A = { (x, y, z) R
3
| x 0, y 0, z 0, x
2
+ y
2
+ z
2
= 1 } s
,
i
f : A R funct
,
ia denita prin f(x, y, z) = (1 x)(1 y)(1 z). Sa se
determine max f(A).
7. Sa se demonstreze ca pentru orice x, y 0 are loc inegalitatea
x
2
+y
2
4
e
x+y2
.
Berkeley 1993
8. Sa se demonstreze ca funct
,
ia f : (0, )
2
R, denita prin
f(x, y) =
xy
(x + 1)(y + 1)(x +y)
,
poseda un maxim global s
,
i sa se determine acest maxim.
9. Fie ABC s
,
i A
1
B
1
C
1
doua triunghiuri avand unghiurile de masuri , ,
s
,
i respectiv
1
,
1
,
1
. Sa se demonstreze ca are loc inegalitatea
cos
1
sin
+
cos
1
sin
+
cos
1
sin
ctg + ctg + ctg .
Kvant, Nr. 11987
3.12 Probleme 101
10. Fie A R
2
o mult
,
ime deschisa cu proprietatea ca [0, 1] [0, 1] A s
,
i e
f : A R o funct
,
ie diferent
,
iabila pe A, care ndeplines
,
te urmatoarele
condit
,
ii:
(i) f(0, 0) +f(1, 0) +f(1, 1) +f(0, 1) = 0;
(ii) (x, y) [0, 1] [0, 1] :

f
x
(x, y)

f
y
(x, y)

1.
Sa se demonstreze ca (x, y) [0, 1] [0, 1] : |f(x, y)|
3
4
.
Olimpiada student
,
easca, U.R.S.S.
11. Fie f : R
n
R
n
o funct
,
ie diferent
,
iabila care ndeplines
,
te urmatoarele
condit
,
ii:
(i) x R
n
: rang J(f)(x) = n;
(ii) pentru orice mult
,
ime compacta A R
n
, mult imea f
1
(A) este com-
pacta.
Sa se demonstreze ca f(R
n
) = R
n
.
Berkeley 1992
12. Fie n 2, A R
n
o mult
,
ime deschisa, a A, iar f : A R o funct
,
ie
continua pe A s
,
i diferent
,
iabila n a, cu df(a) = 0. Sa se demonstreze ca
pentru orice vecinatate V a lui a exista un x V \ { a } n as
,
a fel ncat
f(x) = f(a).
Concursul student
,
esc Traian Lalescu, etapa nala 1997
13. Fie f : R
n
R
m
o funct
,
ie, 0 s
,
i p > 1. Sa se demonstreze ca daca
x, y R
n
: f(x) f(y) x y
p
,
atunci f este constanta.
14. Fie A R
n
o mult
,
ime conexa deschisa nevida s
,
i f : A R
m
o funct
,
ie
diferent
,
iabila care se bucura de urmatoarele proprietat
,
i:
(i) exista un punct a A as
,
a ncat f(a) = 0
m
;
(ii) exista un 0 astfel ca x A : df(x) f(x).
Sa se demonstreze ca f = 0
m
.
102 3 Calcul diferent
,
ial n R
n
15. Fie f : R
n
R o funct
,
ie diferent
,
iabila, iar un numar real cu propri-
etatea
> 0 > 0 as
,
a ncat x R
n
cu x > : |f(x) | < .
Sa se demonstreze ca exista un punct c R
n
astfel ca df(c) = 0.
16. Fie f : R
n
R o funct
,
ie diferent
,
iabila proprie (adica pentru orice
mult
,
ime compacta A R, mult
,
imea f
1
(A) este compacta n R
n
). Sa
se demonstreze ca daca f are un singur punct critic care este punct de
extrem local, atunci acesta este punct de extrem global.
17. Fie A R
n
convexa deschisa nevida s
,
i f
k
: A R
m
(k N) un
s
,
ir de funct
,
ii diferent iabile care converge punctual pe A catre funct
,
ia
f : A R
m
. Sa se arate ca daca
sup { df
k
(x) | k N, x A} < ,
atunci f este continua pe A.
3.13 Funct
,
ii de clasa C
1
3.13.1 Denit
,
ie. Fie A R
n
o mult
,
ime deschisa nevida, iar f : A R
m
o
funct
,
ie care este diferent
,
iabila n ecare punct al lui A. Consideram funct
,
ia
df : A L(R
n
, R
m
), care asociaza ecarui punct x A, diferent iala df(x)
L(R
n
, R
m
), a lui f n punctul x. Se spune ca funct
,
ia df este continua ntr-un
punct a A daca pentru orice > 0 exista un > 0 astfel ca pentru orice
x A cu x a < sa avem df(x) df(a) < .
3.13.2 Teorema. Fie A R
n
o mult
,
ime deschisa, a A s
,
i f : A R
m
o funct
,
ie diferent
,
iabila pe A. Atunci funct
,
ia df : A L(R
n
, R
m
) este con-
tinua n punctul a daca s
,
i numai daca pentru orice j {1, . . . , n} funct
,
ia
f
x
j
: A R
m
este continua n a.
Demonstrat
,
ie. Necesitatea. Fixam un j {1, . . . , n}. Pentru a dovedi ca
funct
,
ia
f
x
j
este continua n a, e > 0 oarecare. Deoarece funct
,
ia df este
continua n a, exista > 0 astfel ca pentru orice x A cu xa < sa avem
df(x) df(a) < . Rezulta de aici ca pentru orice x A cu x a < s
,
i
orice h R
n
cu h = 1 avem
df(x)(h) df(a)(h) =
_
_
_
df(x) df(a)
_
(h)
_
_
df(x) df(a) < .
3.13 Funct
,
ii de clasa C
1
103
Alegand h = e
j
, deducem ca
df(x)(e
j
) df(a)(e
j
) =
_
_
_
_
f
x
j
(x)
f
x
j
(a)
_
_
_
_
<
oricare ar x A cu x a < , deci funct
,
ia
f
x
j
este continua n a.
Sucient
,
a. Fie > 0 arbitrar. Deoarece toate funct
,
iile
f
x
j
(j = 1, . . . , n)
sunt continue n punctul a, pentru ecare j {1, . . . , n} exista un
j
> 0 as
,
a
ncat oricare ar x A cu x a <
j
sa avem
_
_
_
_
f
x
j
(x)
f
x
j
(a)
_
_
_
_
<

:=

2n
.
Notam := min{
1
, . . . ,
n
}. Atunci > 0 s
,
i
_
_
_
_
f
x
j
(x)
f
x
j
(a)
_
_
_
_
<

pentru orice x A cu x a < s


,
i orice j {1, . . . , n}. Vom dovedi ca
(1) df(x) df(a) < oricare ar x A cu x a < .
Fie x A un punct arbitrar cu proprietatea xa < si e h := (h
1
, . . . , h
n
)
un punct arbitrar din R
n
cu proprietatea h = 1. Avem
_
_
_
df(x) df(a)
_
(h)
_
_
=
_
_
_
_
_
_
n

j=1
h
j
f
x
j
(x)
n

j=1
h
j
f
x
j
(a)
_
_
_
_
_
_

j=1
|h
j
|
_
_
_
_
f
x
j
(x)
f
x
j
(a)
_
_
_
_

j=1
|h
j
| n

h =

2
< .
Intrucat h S
n1
a fost arbitrar, rezulta ca (1) are loc, deci funct
,
ia df este
continua n punctul a.
3.13.3 Denit
,
ie (funct
,
ii de clasa C
1
). Fie A R
n
o mult
,
ime deschisa nevida
s
,
i f : A R
m
o funct
,
ie. Daca f este diferent
,
iabila n ecare punct al lui A s
,
i
funct
,
ia df : A L(R
n
, R
m
) este continua pe A, atunci se spune ca f este de
clasa C
1
pe A.
104 3 Calcul diferent
,
ial n R
n
3.13.4 Teorema (caracterizarea funct
,
iilor de clasa C
1
). Fie A o submult
,
ime
deschisa nevida a lui R
n
. O funct
,
ie f : A R
m
este de clasa C
1
pe A daca
s
,
i numai daca ea este derivabila part
,
ial pe A s
,
i pentru ecare j {1, . . . , n}
funct
,
ia
f
x
j
este continua pe A.
Demonstrat
,
ie. Partea de necesitate rezulta din teoremele 3.6.4 s
,
i 3.13.2, iar
partea de sucient
,
a din teoremele 3.6.7 s
,
i 3.13.2.
3.14 Teorema difeomorsmului local
3.14.1 Denit
,
ie (difeomorsme). Fie A s
,
i B submult
,
imi deschise nevide ale
spat
,
iului R
n
. O funct
,
ie f : A B se numes
,
te difeomorsm daca ea este
bijectiva, diferent
,
iabila pe A s
,
i cu inversa diferent
,
iabila pe B. Funct
,
ia f se
numes
,
te difeomorsm de clasa C
1
daca ea este bijectiva, de clasa C
1
pe A s
,
i
cu inversa de clasa C
1
pe B.
3.14.2 Teorema (teorema difeomorsmului local, teorema de inversabilitate
locala). Fie A R
n
o mult
,
ime deschisa, a A s
,
i f : A R
n
o funct
,
ie
diferent
,
iabila pe A, cu diferent
,
iala df continua n a s
,
i cu proprietatea ca df(a)
este bijectiv a. Atunci exista o vecinatate deschisa U V(a) cu urmatoarele
proprietat
,
i:
(i) U A s
,
i mult
,
imea f(U) este deschisa;
(ii) funct
,
ia

f : U f(U), denita prin

f(x) := f(x), este un difeomor-
sm.
Demonstrat
,
ie. Facem demonstrat
,
ia n cazul particular cand df(a) = I, unde
I este aplicat
,
ia identica a lui R
n
, I(x) := x oricare ar x R
n
. Aceasta nu
constituie o restrangere a generalitat
,
ii deoarece, altfel, lucramn locul funct
,
iei
f cu funct
,
ia F : A R
n
, denita prin F :=
1
f, unde := df(a). Aceasta
satisface
dF(a) = d
1
(f(a)) df(a) =
1
= I.
Presupunem as
,
adar ca df(a) = I. Pentru ecare punct y R
n
consideram
funct
,
ia g
y
: A R
n
, denita prin g
y
(x) := y + x f(x). Evident, g
y
este
diferent
,
iabila pe A s
,
i
dg
y
(x) = I df(x) oricare ar x A.
Impart
,
im demonstrat
,
ia n trei etape.
3.14 Teorema difeomorsmului local 105
Etapa I. Localizarea unei mult
,
imi deschise U n asa fel ncat a U A
s
,
i f

U
este injectiva.
Deoarece funct
,
ia df este continua n punctul a s
,
i df(a) = I, exista un
r
0
> 0 as
,
a ncat notand U := B(a, r
0
) sa avem U A s
,
i
(1) df(x) I <
1
2
oricare ar x U.
Vom dovedi n continuare ca
(2) g
y
(x) g
y
(x

)
1
2
x x

pentru orice y R
n
s
,
i orice x, x

U.
Fie, n acest scop, y R
n
s
,
i x, x

U arbitrar alese. Avandn vedere ca U este


o mult
,
ime convexa, n baza teoremei 3.11.7 rezulta existent a unui (0, 1)
astfel ncat notand c := (1 )x +x

sa avem
g
y
(x) g
y
(x

) dg
y
(c) x x

.
Cum c U, din (1) deducem ca
g
y
(x) g
y
(x

) I df(c) x x


1
2
x x

,
deci (2) are loc.
Mai aratam ca f

U
este injectiva. Presupunand contrarul, ar exista punc-
tele x, x

U, x = x

, astfel ca f(x) = f(x

) =: y. Atunci
g
y
(x) = y +x f(x) = x s
,
i g
y
(x

) = y +x

f(x

) = x

.
T
,
inand seama de (2), rezulta ca
x x

= g
y
(x) g
y
(x

)
1
2
x x

,
deci x x

= 0, ceea ce este absurd. Contradict


,
ia obt inuta arata ca f

U
este injectiva.
Etapa a II-a. Mult
,
imea V := f(U) este deschisa.
Fie y
0
V arbitrar. Exista atunci un unic x
0
U astfel ca y
0
= f(x
0
).
Cum U este deschisa, exista un
0
> 0 n as
,
a fel ncat notand B
0
:=

B(x
0
,
0
)
sa avem B
0
U. Notam :=
0
/2 s
,
i demonstram ca B(y
0
, ) V .
106 3 Calcul diferent
,
ial n R
n
Fie, n acest scop, y un punct oarecare din B(y
0
, ). Observam ca pentru
ecare x B
0
avem
g
y
(x) x
0
g
y
(x) g
y
(x
0
) +g
y
(x
0
) x
0

1
2
x x
0
+y +x
0
f(x
0
) x
0
(conform lui (2))

1
2

0
+y y
0

1
2

0
+ =

0
2
+

0
2
=
0
.
As
,
adar, g
y
(x)

B(x
0
,
0
) = B
0
pentru orice x B
0
.
Intrucat B
0
U, din (2) urmeaza ca g
y
: B
0
B
0
este o
1
2
-contract
,
ie.
Mult imea B
0
indnchisa, n baza teoremei de punct x a lui Banach deducem
existent
,
a unui unic punct x B
0
astfel ncat g
y
( x) = x, adica
y + x f( x) = x,
de unde y = f( x) f(B
0
) f(U) = V . Cum y a fost un punct oarecare din
B(y
0
, ), rezulta ca B(y
0
, ) V , deci V este mult
,
ime deschisa.
Evident, funct
,
ia

f : U V = f(U), denita prin

f(x) := f(x), este
bijectiva s
,
i diferent
,
iabila pe U.
Etapa a III-a. Funct
,
ia

f
1
: V U este diferent
,
iabila pe V .
Aratam mai ntai ca pentru ecare x U, aplicat
,
ia liniara d

f(x) = df(x)
L(R
n
, R
n
) este bijectiva. Presupunand contrarul, ar rezulta existent
,
a unui
punct x U astfel ca df(x) sa nu e injectiva, deci ar exista un h R
n
\ {0
n
}
cu proprietatea ca df(x)(h) = 0
n
. Avem
h = I(h) df(x)(h) I df(x) h <
1
2
h,
ceea ce este absurd. Contradict
,
ia obt
,
inuta arata ca d

f(x) este bijectiva oricare


ar x U.
Arataam acum ca
(3)

f
1
(y)

f
1
(y

) 2y y

oricare ar y, y

V.
Fie y, y

V s
,
i e x :=

f
1
(y), x

:=

f
1
(y

). Atunci avem x, x

U s
,
i
f(x) = y, f(x

) = y

. Alegem un y R
n
arbitrar. Avem

f
1
(y)

f
1
(y

) = x x

2g
y
(x) g
y
(x

)
= 2y +x f(x) y x

+f(x

)
= 2
_
_
x x


_
f(x) f(x

)
__
_
= 2

f
1
(y)

f
1
(y

) (y y

)
2

f
1
(y)

f
1
(y

) 2y y

,
3.14 Teorema difeomorsmului local 107
de unde rezulta validitatea lui (3). Relat
,
ia (3) asigura ca

f
1
este continua
pe V .
In concluzie, funct
,
ia

f : U V se bucura de urmatoarele proprietat
,
i:


f este bijectiva s
,
i diferent
,
iabila pe U;
d

f(x) este bijectiva pentru ecare x U;


f
1
este continua pe V .
Aplicand teorema 3.10.4, deducem ca

f
1
este diferent
,
iabila pe V .
3.14.3 Teorema. Fie A s
,
i B submult
,
imi deschise nevide ale spat
,
iului R
n
s
,
i
f : A B o funct
,
ie bijectiva, de clasa C
1
pe A, avand inversa diferent
,
iabila
pe B. Atunci f
1
este de clasa C
1
pe B.
Demonstrat
,
ie. Fara demonstrat
,
ie.
3.14.4 Teorema. Fie A R
n
o mult
,
ime deschisa nevida, f : A R
n
o
funct
,
ie de clasa C
1
pe A s
,
i a A un punct cu proprietatea ca df(a) este bijec-
tiva. Atunci exista o vecin atate deschisa U V(a) cu urmatoarele proprietat
,
i:
(i) U A s
,
i mult
,
imea f(U) este deschisa;
(ii) funct
,
ia

f : U f(U), denita prin

f(x) := f(x), este un difeomor-
sm de clasa C
1
.
Demonstrat
,
ie. Se aplica teoremele 3.14.2 s
,
i 3.14.3.
3.14.5 Consecint
,
a. Fie A o submult
,
ime deschisa nevida a spat
,
iului R
n
, iar
f : A R
n
o funct
,
ie de clasa C
1
cu proprietatea ca df(x) este bijectiva pentru
ecare x A. Atunci urmatoarele armat
,
ii sunt adevarate:
1

Mult
,
imea f(A) este deschisa.
2

Daca, n plus, f este injectiva, atunci funct ia



f : A f(A), denita
prin

f(x) := f(x), este un difeomorsm de clasa C
1
.
3.14.6 Consecint
,
a. Fie A s
,
i B submult
,
imi deschise nevide ale spat
,
iului R
n
,
iar f : A B o funct
,
ie bijectiva, de clasa C
1
pe A s
,
i cu proprietatea ca df(x)
este bijectiva oricare ar x A. Atunci f este un difeomorsm de clasa C
1
.
108 3 Calcul diferent
,
ial n R
n
3.15 Funct
,
ii implicite
3.15.1 Denit
,
ie. Fie A R
n
s
,
i B R
m
mult
,
imi nevide, iar F : AB R
m
o funct
,
ie. Notam
S := { (x, y) AB | F(x, y) = 0
m
}
S
1
:= { x A | y B : F(x, y) = 0
m
}.
Fie apoi (a, b) S. Atunci a A, b B s
,
i F(a, b) = 0
m
. Se spune ca
ecuat
,
ia F(x, y) = 0
m
deneste implicit pe y ca funct
,
ie de x n jurul punctului
(a, b) daca exista U V(a) s
,
i V V(b) as
,
ancat pentru orice punct x US
1
,
mult
,
imea
(1) { y V B | F(x, y) = 0
m
}
cont
,
ine exact un element. In acest caz putem deni funct
,
ia f : U S
1
V ,
punand f(x) := unicul element al mult
,
imii (1). Avem atunci
F(x, f(x)) = 0
m
pentru orice x U S
1
.
Se mai spune ca f este o funct
,
ie implicita denita n jurul punctului (a, b) de
ecuat
,
ia F(x, y) = 0
m
.
3.15.2 Exemplu. Fie A = B = R s
,
i e F : R R R funct
,
ia denita prin
F(x, y) := x
2
+y
2
1. Atunci
S = { (x, y) R
2
| x
2
+y
2
= 1 } s
,
i S
1
= [1, 1].
Fie (a, b) S, adica a
2
+b
2
= 1. Daca a = 1, atunci ecuat
,
ia
F(x, y) = x
2
+y
2
1 = 0
denes
,
te implicit pe y ca funct
,
ie de x n jurul punctului (a, b). Mai precis,
y(x) =

1 x
2
daca b > 0 si respectiv y(x) =

1 x
2
daca b < 0.
Daca nsa a = 1, atunci b = 0. In acest caz, ecuat ia F(x, y) = 0 nu
denes
,
te implicit pe y ca funct
,
ie de x n jurul punctului (1, 0). Intr-adevar,
pentru orice vecinatate U V(1) s
,
i orice vecinatate V V(0) exista puncte
x U S
1
= U [1, 1] pentru care mult
,
imea {y V | x
2
+y
2
= 1} cont
,
ine
exact doua elemente.
In continuare, vom identica pe R
n
R
m
cu R
n+m
. In acest fel, daca
A R
n
s
,
i B R
m
, atunci AB poate privita ca submult ime a lui R
n+m
.
Putem vorbi as
,
adar despre diferent
,
iabilitatea unei funct
,
ii F : AB R
m
.
3.15 Funct
,
ii implicite 109
Fie A o submult
,
ime a lui R
n
, B o submult
,
ime a lui R
m
, a int A, b int B
s
,
i F = (F
1
, . . . , F
m
) : AB R
m
o funct
,
ie diferent iabila n punctul (a, b):
F(x
1
, . . . , x
n
, y
1
, . . . , y
m
) =
_
F
1
(x
1
, . . . , x
n
, y
1
, . . . , y
m
), . . . ,
F
m
(x
1
, . . . , x
n
, y
1
, . . . , y
m
)
_
pentru orice x = (x
1
, . . . , x
n
) A s
,
i orice y = (y
1
, . . . , y
m
) B. Notam
J
y
(F)(a, b) :=
_
_
_
F
1
y
1
(a, b)
F
1
y
m
(a, b)
.
.
.
.
.
.
F
m
y
1
(a, b)
F
m
y
m
(a, b)
_
_
_
.
3.15.3 Teorema (teorema funct
,
iei implicite). Fie A R
n
s
,
i B R
m
mult
,
imi
deschise nevide, F : A B R
m
o funct
,
ie de clasa C
1
pe A B, iar
a A s
,
i b B puncte cu proprietatea F(a, b) = 0
m
si det J
y
(F)(a, b) = 0.
Atunci exista o vecinatate deschisa U V(a) cu U A, o vecinatate deschisa
V V(b) cu V B s
,
i o funct
,
ie f : U V care se bucura de urmatoarele
proprietat
,
i:
(i) f este de clasa C
1
pe U;
(ii) f(a) = b;
(iii) x U : F(x, f(x)) = 0
m
.
Mai mult, vecinatat
,
ile U s
,
i V pot alese n as
,
a fel ncat sa existe o singura
funct
,
ie f : U V care sa satisfaca (iii).
Demonstrat
,
ie. Fara demonstrat
,
ie.
3.15.4 Observat
,
ie. Notand cu f
1
, . . . , f
m
componentele lui f, (iii) se scrie
x U : F(x
1
, . . . , x
n
, f
1
(x
1
, . . . , x
n
), . . . , f
m
(x
1
, . . . , x
n
)) = 0
m
,
relat
,
ie echivalenta cu sistemul
x U : F
i
(x
1
, . . . , x
n
, f
1
(x
1
, . . . , x
n
), . . . , f
m
(x
1
, . . . , x
n
)) = 0,
oricare ar i { 1, . . . , m}. Fixam un indice j {1, . . . , n}. Aplicand regula
de derivare a funct
,
iilor compuse, deducem ca
F
i
x
j
(x, f(x)) +
F
i
y
1
(x, f(x))
f
1
x
j
(x) +
F
i
y
2
(x, f(x))
f
2
x
j
(x)
+ +
F
i
y
m
(x, f(x))
f
m
x
j
(x) = 0
110 3 Calcul diferent
,
ial n R
n
pentru orice x U s
,
i orice i {1, . . . , m}. Punand x = a, rezulta
_

_
F
1
y
1
(a, b)
f
1
x
j
(a) + +
F
1
y
m
(a, b)
f
m
x
j
(a) =
F
1
x
j
(a, b)
F
2
y
1
(a, b)
f
1
x
j
(a) + +
F
2
y
m
(a, b)
f
m
x
j
(a) =
F
2
x
j
(a, b)
.
.
.
F
m
y
1
(a, b)
f
1
x
j
(a) + +
F
m
y
m
(a, b)
f
m
x
j
(a) =
F
m
x
j
(a, b).
Acesta este un sistem liniar de m ecuat
,
ii cu necunoscutele
f
1
x
j
(a),
f
2
x
j
(a), . . . ,
f
m
x
j
(a).
Determinantul sistemului este det J
y
(F)(a, b) = 0, deci sistemul este compa-
tibil determinat s
,
i poate rezolvat cu regula lui Cramer. Dand lui j suc-
cesiv valorile 1, . . . , n, se determina n acest mod toate derivatele part
,
iale
f
i
x
j
(a), i = 1, m, j = 1, n.
3.16 Probleme
1. Fie mult
,
imile denite prin A = { (x, y) R
2
| x > 0, y > 0 } s
,
i respectiv
B = { (u, v) R
2
| u + v > 0, u v > 0 }. Sa se demonstreze ca
funct
,
ia f : A B, denita prin f(x, y) = (x
2
+ y
2
, x
2
y
2
), este un
difeomorsm de clasa C
1
.
2. Sa se demonstreze ca funct
,
ia f : R
2
R
2
, denita prin
f(x, y) = (x
3
+x, y x
2
),
este un difeomorsm de clasa C
1
.
3. Sa se demonstreze ca funct
,
ia f : R
2
R
2
, denita prin
f(x, y) = (x
3
+ 3xe
y
, y x
2
),
este un difeomorsm de clasa C
1
.
3.16 Probleme 111
4. Se cauta funct
,
iile f : A R, de clasa C
1
s
,
i care satisfac
(1)
f
x
(x, y)
f
y
(x, y) + 3(x y)f(x, y) = 0
oricare ar (x, y) A, unde A = {(x, y) R
2
| x > y}.
a) Sa se demonstreze ca pentru orice (x, y) A avem (xy, x + y) B,
unde B = {(u, v) R
2
| v
2
> 4u}.
b) Funct
,
ia g : A B, denita prin g(x, y) = (xy, x + y), este un
difeomorsm de clasa C
1
.
c) Daca funct
,
ia f : A R este de clasa C
1
, atunci s
,
i funct
,
ia F : B R,
denita prin F = f g
1
, este de clasa C
1
. Mai mult, (1) are loc oricare
ar (x, y) A daca s
,
i numai daca avem
(u, v) B :
F
u
(u, v) 3F(u, v) = 0.
d) Sa se determine funct
,
iile f : A R, de clasa C
1
s
,
i care satisfac (1)
pentru orice (x, y) A.
5. Fie f : R
2
R
2
funct
,
ia denita prin
f(x, y) =
_ _
x,
1
x
arctg(xy)
_
daca x = 0
(0, y) daca x = 0.
a) Sa se arate ca f este o funct
,
ie injectiva de clasa C
1
.
b) Sa se demonstreze ca mult
,
imea B = f
_
R
2
_
este deschisa s
,
i ca funct ia

f : R
2
B, denita prin

f(x, y) = f(x, y), este un difeomorsm de
clasa C
1
.
6. Fie A = { (x, y) R
2
| < y < } s
,
i f : A R
2
funct
,
ia denita prin
f(x, y) = (e
x
cos y, e
x
sin y).
a) Sa se arate ca f este o funct
,
ie injectiva de clasa C
1
.
b) Sa se arate ca mult
,
imea f(A) este deschisa, iar funct
,
ia

f : A f(A),
denita prin

f(x, y) = f(x, y), este un difeomorsm de clasa C
1
. Sa se
determine J
_

f
1
_
_

2
,
e

2
_
.
112 3 Calcul diferent
,
ial n R
n
7. Fie f : R
3
R
3
funct
,
ia denita prin
f(x
1
, x
2
, x
3
) = (x
1
, x
1
x
2
, x
1
x
2
x
3
)
s
,
i mult
,
imea A = { (x
1
, x
2
, x
3
) R
3
| x
1
> 0, x
2
> 0 }.
a) Sa se arate ca mult
,
imea f(A) este deschisa.
b) Sa se studieze daca funct
,
ia

f : A f(A), denita prin

f(x) = f(x),
este un difeomorsm de clasa C
1
.
8. Fie A = { x R
n
| x < 1 }. Sa se arate ca funct
,
ia f : A R
n
, denita
prin
f(x) =
1

1 x
2
x,
este un difeomorsm de clasa C
1
.
9. Fie funct
,
ia f : R
2
R
2
, denita prin
f(x, y) =
_
2x + 2 cos
_

2
y
_
, 2y + 2 cos
_

2
x
__
.
Sa se demonstreze ca exista o vecinatate deschisa U a punctului
_
1
2
,
1
2
_
,
n as
,
a fel ncat mult
,
imea f(U) sa e deschisa s
,
i funct
,
ia

f : U f(U),
denita prin

f(x, y) = f(x, y), sa e un difeomorsm de clasa C
1
. Sa se
determine J
_

f
1
_
(

2 + 1,

2 1).
10. Fie g : R
2
R
2
funct
,
ia denita prin
g(x, y) = (y cos x, (x +y) sin y),
iar f : R
2
R
3
funct
,
ia denita prin
f(x, y) = (x
2
y, 3x 2y, 2xy +y
2
).
a) Sa se arate ca exista o vecinatate deschisa U a punctului
_
0,

2
_
astfel
ca mult
,
imea g(U) sa e deschisa, iar funct
,
ia g : U g(U), denita prin
g(x, y) = g(x, y), sa e un difeomorsm de clasa C
1
.
b) Sa se determine J
_
f g
1
_
_

2
,

2
_
.
11. Fie g : R R o funct
,
ie de clasa C
1
, iar f : R
2
R
2
funct
,
ia denita
prin f(x, y) = (x + g(y), y + g(x)). Sa se demonstreze ca daca exista
un (0, 1) as
,
a ncat |g

(t)| pentru orice t R, atunci f este un


difeomorsm de clasa C
1
.
3.16 Probleme 113
12. (generalizarea problemei precedente) Fie n 2, g : R
n1
R o funct
,
ie
de clasa C
1
, iar f : R
n
R
n
funct
,
ia denita prin
f(x
1
, . . . , x
n
) = (x
1
+g(x
2
, x
3
, . . . , x
n
), x
2
+g(x
1
, x
3
, . . . , x
n
),
. . . , x
n
+g(x
1
, x
2
, . . . , x
n1
)).
Sa se demonstreze ca daca exista un numar pozitiv <
1
n 1
as
,
a ncat
j { 1, . . . , n 1 }, x R
n1
:

g
x
j
(x)

,
atunci f este un difeomorsm de clasa C
1
.
13. Fie g : R R o funct
,
ie de clasa C
1
, f : R
2
R
2
funct
,
ia denita prin
f(x, y) = (g(x), y+xg(x)) s
,
i (x
0
, y
0
) un punct al lui R
2
cu proprietatea
g

(x
0
) = 0. Sa se demonstreze ca exista o vecinatate deschisa W a lui
(x
0
, y
0
) si o funct
,
ie h : R R, de clasa C
1
, asa ncat sa e ndeplinite
urmatoarele condit ii:
(i) mult
,
imea f(W) este deschisa;
(ii) funct
,
ia

f : W f(W), denita prin

f(x, y) = f(x, y), este un
difeomorsm de clasa C
1
;
(iii) (u, v) f(W) :

f
1
(u, v) = (h(u), v +uh(u)).
Berkeley 1984
14. Fie f : R
n
R
n
o funct
,
ie de clasa C
1
cu proprietatea ca exista > 0
astfel ca
x, y R
n
: f(x) f(y) x y.
a) Sa se arate ca x, h R
n
: df(x)(h) h.
b) Folosind eventual faptul ca R
n
este singura submult
,
ime nevida simul-
tan deschisa s
,
i nchisa a lui R
n
, sa se arate ca f este un difeomorsm de
clasa C
1
.
15. Sa se studieze daca exista o vecinatate deschisa U a punctului (1, 1) s
,
i o
funct
,
ie f : U R, de clasa C
1
s
,
i care ndeplineste urmatoarele condit
,
ii:
(i) f(1, 1) = 1;
(ii) pentru orice (x
1
, x
2
) U are loc egalitatea
e
x
1
+x
2
ln(x
1
+x
2
+f(x
1
, x
2
) 2) 2x
1
+x
2
+f(x
1
, x
2
) = 0.
114 3 Calcul diferent
,
ial n R
n
In caz armativ, sa se determine
f
x
1
(1, 1) s
,
i
f
x
2
(1, 1).
16. Fie F : R
2
R funct
,
ia denita prin
F(x, y) := (1 x
2
) cos y e
x
sin y ln(1 +x
2
+y
2
).
Sa se demonstreze ca exista o vecinatate deschisa U R a lui 1 s
,
i o
funct
,
ie f : U R, de clasa C
1
pe U, n as
,
a fel ncat
f(1) = 0 s
,
i F(x, f(x)) = 0 pentru orice x U.
Sa se determine f

(1).
17. Sa se arate ca ecuat
,
ia x
2
+ xy + 2y
2
+ 3z
4
z = 9 denes
,
te implicit
ntr-o vecinatate a punctului (1, 2) o funct
,
ie z = f(x, y), de clasa C
1
s
,
i cu proprietatea f(1, 2) = 1. Sa se determine derivatele part
,
iale de
ordinul ntai s
,
i diferent
,
iala lui f n punctul (1, 2).
18. Sa se studieze posibilitatea aplicarii teoremei funct
,
iei implicite pentru
funct
,
ia F : R
2
R
2
R
2
, denita prin
F(x, y) = (x
2
1
x
2
2
y
3
1
+y
2
2
+ 4, 2x
1
x
2
+x
2
2
2y
2
1
+ 3y
4
2
+ 8)
s
,
i punctul (a, b) R
2
R
2
, unde a = (2, 1), b = (2, 1). In caz arma-
tiv, sa se determine derivatele part
,
iale de ordinul ntai n punctul a ale
funct
,
iei denite implicit de ecuat
,
ia F(x, y) = 0
2
.
19. Sa se arate ca sistemul
_
x
2
y
2
z
3
t
8
= 9
2xy +y
2
+ 2z
2
+ 3t
4
= 14
denes
,
te ntr-o vecinatate a punctului (1, 1) o funct ie
f = (f
1
(x, y), f
2
(x, y)),
de clasa C
1
s
,
i cu proprietatea f(1, 1) = (2, 1). Sa se determine
df(1, 1).
20. Sa se demonstreze ca sistemul
_
_
_
x
2
y cos(uv) +z
2
= 0
x
2
+y
2
sin(uv) + 2z
2
= 2
xy cos ucos v +z = 1
3.17 Extreme condit ionate 115
denes
,
te implicit ntr-o vecinatate a punctului
_

2
, 0
_
o funct
,
ie
f = (f
1
(u, v), f
2
(u, v), f
3
(u, v)),
de clasa C
1
s
,
i cu proprietatea f
_

2
, 0
_
= (1, 1, 0). Sa se determine
df
_

2
, 0
_
.
21. Sa se demonstreze ca sistemul
_
x
2
+uy +e
v
= 0
2x +u
2
uv = 5
denes
,
te implicit ntr-o vecinatate a punctului (2, 5) o funct
,
ie
f = (f
1
(x, y), f
2
(x, y)),
de clasa C
1
s
,
i cu proprietatea f(2, 5) = (1, 0). Sa se determine df(2, 5).
3.17 Extreme condit
,
ionate
Fie A R
n
o mult
,
ime nevida, m N cu m < n, iar f : A R s
,
i F : A R
m
funct
,
ii date. Notam
C := {x A | F(x) = 0
m
}.
Un punct c C se numes
,
te punct de extrem local al lui f relativ la mult
,
imea C
(sau punct de extrem local condit
,
ionat, sau punct de extrem local cu leg aturi )
daca c este punct de extrem local pentru f

C
.
3.17.1 Teorema (regula multiplicatorilor lui Lagrange). Fie m s
,
i n numere
naturale astfel ncat m < n, A R
n
o mult
,
ime deschisa nevida, iar f : A R
s
,
i F = (F
1
, . . . , F
m
) : A R
m
funct
,
ii de clasa C
1
. Daca c C este un punct
de extrem local al lui f relativ la mult imea C, iar rang J(F)(c) = m, atunci
exista un punct
0
R
m
as
,
a ncat (c,
0
) A R
m
sa e punct critic al
funct
,
iei L : AR
m
R, denite prin
L(x, ) := f(x) +
1
F
1
(x) + +
m
F
m
(x).
Demonstrat
,
ie. Fara demonstrat
,
ie.
3.17.2 Observat
,
ie. Teorema precedenta da o condit
,
ie necesara ca c sa e
punct de extrem local condit
,
ionat. Numerele
01
, . . . ,
0m
, a caror existent
,
a
este garantata de teorema 3.17.1, se numesc multiplicatori Lagrange, iar func-
t
,
ia L se numes
,
te funct
,
ia lui Lagrange asociata funct iilor f s
,
i F.
116 3 Calcul diferent
,
ial n R
n
3.18 Derivate part
,
iale de ordinul doi
3.18.1 Denit
,
ie. Fie A o submult
,
ime a lui R
n
, a int A, f : A R o funct
,
ie
s
,
i e i, j {1, . . . , n}. Presupunem ca exista o vecinatate deschisa V V(a),
as
,
a ncat V A, care ndeplines
,
te urmatoarele condit
,
ii:
(i) f este derivabila part
,
ial n raport cu variabila x
i
pe V ;
(ii) funct
,
ia
(1) x V
f
x
i
(x) R
este derivabila part
,
ial n raport cu variabila x
j
n punctul a.
Atunci se spune ca f este de doua ori derivabila part
,
ial n raport cu variabilele
(x
i
, x
j
) n punctul a. Derivata part
,
iala a funct
,
iei (1) n raport cu x
j
n punctul
a se numes
,
te derivata part
,
iala de ordinul doi a funct iei f n raport cu vari-
abilele (x
i
, x
j
) n punctul a si se noteaza cu

2
f
x
j
x
i
(a) sau cu f

x
i
x
j
(a). Daca
i = j, atunci se foloses
,
te notat
,
ia

2
f
x
2
i
(a) sau f

x
2
i
(a).
3.18.2 Denit
,
ie (matricea hessiana). Funct
,
ia f poate avea n
2
derivate par-
t
,
iale de ordinul doi n punctul a. Daca exista toate aceste derivate part
,
iale,
atunci se poate forma matricea
H(f)(a) :=
_
_
_
_

2
f
x
2
1
(a)

2
f
x
2
x
1
(a)

2
f
x
n
x
1
(a)
.
.
.
.
.
.
.
.
.

2
f
x
1
x
n
(a)

2
f
x
2
x
n
(a)

2
f
x
2
n
(a)
_
_
_
_
.
Aceasta se numes
,
te matricea hessiana a lui f n punctul a.
In general, daca i = j, atunci se poate ca

2
f
x
j
x
i
(a) =

2
f
x
i
x
j
(a).
3.18.3 Teorema (criteriul lui Schwarz). Fie A R
n
o mult ime deschisa,
a A, i, j {1, . . . , n} cu i < j, iar f : A R o funct
,
ie de doua ori
derivabila part ial pe A, atat n raport cu variabilele (x
i
, x
j
) cat s
,
i n raport cu
3.18 Derivate part
,
iale de ordinul doi 117
variabilele (x
j
, x
i
). Daca funct
,
iile

2
f
x
j
x
i
,

2
f
x
i
x
j
: A R sunt continue n
punctul a, atunci are loc egalitatea

2
f
x
j
x
i
(a) =

2
f
x
i
x
j
(a).
Demonstrat
,
ie. Etapa I . Consideram mai ntai n = 2, i = 1, j = 2.
Fie > 0 arbitrar. Deoarece A este deschisa, a := (a
1
, a
2
) A, iar funct
,
iile

2
f
x
2
x
1
s
,
i

2
f
x
1
x
2
sunt continue n a, exista un r > 0 as
,
a ncat B(a, r) A
s
,
i

2
f
x
2
x
1
(x)

2
f
x
2
x
1
(a)

<

2
,

2
f
x
1
x
2
(x)

2
f
x
1
x
2
(a)

<

2
pentru orice x B(a, r).
Alegem un punct x := (x
1
, x
2
) B(a, r) cu x
1
> a
1
, x
2
> a
2
si notam
:= f(x
1
, x
2
) f(a
1
, x
2
) f(x
1
, a
2
) +f(a
1
, a
2
).
Aplicand teorema de medie a lui Lagrange funct
,
iei g : [a
1
, x
1
] R, denite
prin g(t) := f(t, x
2
) f(t, a
2
), deducem existent
,
a unui punct c
1
(a
1
, x
1
)
astfel ca
g(x
1
) g(a
1
) = (x
1
a
1
)g

(c
1
).
Dar g(x
1
) g(a
1
) = s
,
i
g

(t) =
f
x
1
(t, x
2
)
f
x
1
(t, a
2
).
Avem as
,
adar
(2) = (x
1
a
1
)
_
f
x
1
(c
1
, x
2
)
f
x
1
(c
1
, a
2
)
_
.
Aplicand acum teorema de medie a lui Lagrange funct
,
iei g
1
: [a
2
, x
2
] R,
denite prin g
1
(t) :=
f
x
1
(c
1
, t), deducem existent
,
a unui punct c
2
(a
2
, x
2
)
astfel ca
g
1
(x
2
) g
1
(a
2
) = (x
2
a
2
)g

1
(c
2
),
adica
f
x
1
(c
1
, x
2
)
f
x
1
(c
1
, a
2
) = (x
2
a
2
)

2
f
x
2
x
1
(c
1
, c
2
),
118 3 Calcul diferent
,
ial n R
n
deoarece
g

(c
2
) =

x
2
_
f
x
1
_
(c
1
, c
2
) =

2
f
x
2
x
1
(c
1
, c
2
).
T
,
inand seama de relat
,
ia (2), obt inem
(3) = (x
1
a
1
)(x
2
a
2
)

2
f
x
2
x
1
(c
1
, c
2
).
Mai avem
(c
1
a
1
)
2
+ (c
2
a
2
)
2
< (x
1
a
1
)
2
+ (x
2
a
2
)
2
< r
2
,
deci c := (c
1
, c
2
) B(a, r).
Analog, aplicand teorema lui Lagrange funct
,
iilor h : [a
2
, x
2
] R, h(t) :=
f(x
1
, t) f(a
1
, t) s
,
i respectiv h
1
: [a
1
, x
1
] R, h
1
(t) :=
f
x
2
(t, d
2
), se deduce
existent
,
a unui punct d := (d
1
, d
2
) B(a, r) astfel ca
(4) = (x
1
a
1
)(x
2
a
2
)

2
f
x
1
x
2
(d
1
, d
2
).
Din (3) s
,
i (4) rezulta

2
f
x
2
x
1
(c) =

2
f
x
1
x
2
(d), deci

2
f
x
2
x
1
(a)

2
f
x
1
x
2
(a)

2
f
x
2
x
1
(a)

2
f
x
2
x
1
(c) +

2
f
x
1
x
2
(d)

2
f
x
1
x
2
(a)

2
f
x
2
x
1
(a)

2
f
x
2
x
1
(c)

2
f
x
1
x
2
(d)

2
f
x
1
x
2
(a)

<

2
+

2
= .
Cum > 0 a fost arbitrar, rezulta ca

2
f
x
2
x
1
(a) =

2
f
x
1
x
2
(a).
Etapa a II-a. Consideram acum n > 2 arbitrar.
Fie a := (a
1
, . . . , a
n
). Deoarece a int A, exista un > 0 as
,
a ncat notand
A
0
:= (a
i
, a
i
+) (a
j
, a
j
+), sa avem
(a
1
, . . . , a
i1
, y
1
, a
i+1
, . . . , a
j1
, y
2
, a
j+1
, . . . , a
n
) A
3.19 Diferent
,
iala a doua 119
pentru orice (y
1
, y
2
) A
0
. Consideram funct
,
ia f
0
: A
0
R, denita prin
f
0
(y
1
, y
2
) := f(a
1
, . . . , a
i1
, y
1
, a
i+1
, . . . , a
j1
, y
2
, a
j+1
, . . . , a
n
).
In baza celor demonstrate la etapa precedenta, avem

2
f
0
y
2
y
1
(a
i
, a
j
) =

2
f
0
y
1
y
2
(a
i
, a
j
).
Dar

2
f
0
y
2
y
1
(a
i
, a
j
) =

2
f
x
j
x
i
(a) s
,
i

2
f
0
y
1
y
2
(a
i
, a
j
) =

2
f
x
i
x
j
(a).
Drept urmare s
,
i n acest caz egalitatea din enunt
,
are loc.
3.18.4 Teorema (criteriul lui Young). Fie A R
n
o mult
,
ime deschisa, e
a A, i, j {1, . . . , n} cu i < j, iar f : A R o funct
,
ie derivabila part
,
ial pe
A atat n raport cu variabila x
i
cat s
,
i n raport cu variabila x
j
. Daca funct
,
iile
f
x
i
,
f
x
j
: A R sunt diferent
,
iabile n punctul a, atunci are loc egalitatea

2
f
x
j
x
i
(a) =

2
f
x
i
x
j
(a).
Demonstrat
,
ie. Fara demonstrat
,
ie.
3.19 Diferent
,
iala a doua
3.19.1 Denit
,
ie. Fie A R
n
, a int A s
,
i f : A R o funct
,
ie. Daca exista
o vecinatate deschisa V V(a) cu V A astfel ncat
(i) f este diferent
,
iabila pe V ;
(ii) pentru orice i {1, . . . , n}, funct
,
ia x V
f
x
i
(x) R este
diferent
,
iabila n a,
atunci se spune ca f este de doua ori diferent iabila n punctul a.
3.19.2 Teorema. Fie A R
n
, a int A s
,
i f : A R o funct
,
ie de doua ori
diferent
,
iabila n punctul a. Atunci urmatoarele armat
,
ii sunt adevarate:
1

Pentru orice i, j {1, . . . , n}, f este de doua ori derivabila part


,
ial n
raport cu variabilele (x
i
, x
j
) n punctul a.
2

i, j {1, . . . , n} :

2
f
x
i
x
j
(a) =

2
f
x
j
x
i
(a).
120 3 Calcul diferent
,
ial n R
n
Demonstrat
,
ie. Armat
,
ia 1

rezulta din teorema 3.6.4 (o funct


,
ie diferent
,
iabila
ntr-un punct este derivabila part
,
ial n acel punct), iar armat
,
ia 2

rezulta din
teorema 3.18.4 (criteriul lui Young).
Fie A R
n
, a int A, iar f : A R o funct
,
ie de doua ori diferent
,
iabila
n a. Funct
,
ia d
2
f(a) : R
n
R, denita prin
d
2
f(a)(h) :=
n

i=1
n

j=1
h
i
h
j

2
f
x
j
x
i
(a) oricare ar h := (h
1
, . . . , h
n
) R
n
,
se numes
,
te diferent
,
iala a doua (sau diferent
,
iala de ordinul doi ) a lui f n
punctul a.
Fiind data o matrice C := (c
ij
) R
nn
, putem deni funct
,
ia : R
n
R
prin
(h) :=
n

i=1
n

j=1
c
ij
h
i
h
j
oricare ar h := (h
1
, . . . , h
n
) R
n
.
Funct
,
ia se numes
,
te forma patratica asociata matricei C.
Diferent
,
iala a doua d
2
f(a) este tocmai forma patratica asociata matricei
hessiene H(f)(a).
3.19.3 Teorema. Fie A o submult
,
ime a lui R
n
, e a int A s
,
i f : A R o
funct
,
ie de doua ori diferent iabila n a. Atunci are loc egalitatea
lim
xa
1
x a
2
_
f(x) f(a) df(a)(x a)
1
2
d
2
f(a)(x a)
_
= 0.
Demonstrat
,
ie. Deoarece f este de doua ori diferent
,
iabila n a, exista o vecina-
tate deschisa V V(a), astfel ncat V A s
,
i care ndeplines
,
te condit
,
iile (i) s
,
i
(ii) de la nceputul acestui paragraf. Pentru ecare i {1, . . . , n}, consideram
funct ia F
i
: V R, denita prin F
i
(x) :=
f
x
i
(x). Din (ii) rezulta ca F
i
este
diferent
,
iabila n a, deci
lim
xa
1
x a
_
F
i
(x) F
i
(a) dF
i
(a)(x a)

= 0,
oricare ar i {1, . . . , n}.
Fie > 0 arbitrar. Atunci pentru ecare i {1, . . . , n} exista un r
i
> 0 n
as
,
a fel ncat B(a, r
i
) V s
,
i

1
x a
_
F
i
(x) F
i
(a) dF
i
(a)(x a)

<

:=

2n
3.19 Diferent
,
iala a doua 121
pentru orice x B(a, r
i
) \ {a}. Notand r := min{r
1
, . . . , r
n
}, avem r > 0,
B(a, r) V s
,
i
(1)

F
i
(x) F
i
(a) dF
i
(a)(x a)

x a
pentru orice i {1, . . . , n} s
,
i orice x B(a, r).
Consideram funct
,
ia g : B(a, r) R, denita prin
g(x) := f(x) f(a) df(a)(x a)
1
2
d
2
f(a)(x a)
= f(x) f(a)
n

i=1
(x
i
a
i
)
f
x
i
(a)

1
2
n

i=1
n

j=1
(x
i
a
i
)(x
j
a
j
)

2
f
x
j
x
i
(a).
Deoarece f este diferent
,
iabila pe V , urmeaza ca g este diferent
,
iabila pe B(a, r).
Fie k {1, . . . , n}. Observam ca pentru orice x B(a, r) avem
g
x
k
(x) =
f
x
k
(x)
f
x
k
(a)
n

j=1
(x
j
a
j
)

2
f
x
j
x
k
(a)
= F
k
(x) F
k
(a)
n

j=1
(x
j
a
j
)
F
k
x
j
(a)
= F
k
(x) F
k
(a) dF
k
(a)(x a).
T
,
inand seama de relat
,
ia (1), deducem ca
(2)

g
x
k
(x)

x a pentru orice k {1, . . . , n} s


,
i orice x B(a, r).
Fie acum x B(a, r) \ {a} arbitrar ales. Aplicand teorema 3.11.6 funct
,
iei
g, rezulta ca exista un (0, 1) as
,
a ncat notand c := (1 )a +x, sa avem
g(x) g(a) = dg(c)(x a) =
n

k=1
(x
k
a
k
)
g
x
k
(c).
Cum g(a) = 0, c B(a, r) s
,
i c a x a, n baza lui (2) avem
|g(x)|
n

k=1
|x
k
a
k
|

g
x
k
(c)

x a
2
=

2
x a
2
.
122 3 Calcul diferent
,
ial n R
n
Drept urmare, pentru orice x B(a, r) \ {a} avem
1
x a
2
|g(x)| =
1
x a
2

f(x) f(a) df(a)(x a)


1
2
d
2
f(a)(x a)


2
< ,
ceea ce arata ca egalitatea din enunt
,
are loc.
3.20 Probleme
1. Fie f : R
2
R funct
,
ia denita prin
f(x, y) =
_
xy(x
2
y
2
)
x
2
+y
2
daca (x, y) = (0, 0)
0 daca (x, y) = (0, 0).
Sa se demonstreze ca f este de clasa C
1
s
,
i ca

2
f
yx
(0, 0) =

2
f
xy
(0, 0).
Sa se verice ca f nu satisface condit
,
iile din teoremele lui Schwarz s
,
i
Young.
2. Fie f : R
2
R funct
,
ia denita prin
f(x, y) =
_
xy
3
x
2
+y
2
daca (x, y) = (0, 0)
0 daca (x, y) = (0, 0).
Sa se demonstreze ca f este de clasa C
1
s
,
i ca

2
f
yx
(0, 0) =

2
f
xy
(0, 0).
3. Sa se demonstreze ca funct
,
ia f : R
2
R, denita prin
f(x, y) =
_
x
2
arctg
y
x
y
2
arctg
x
y
daca x = 0 s
,
i y = 0
0 daca x = 0 sau y = 0
este de clasa C
1
. Sa se determine

2
f
yx
(0, 0) si

2
f
xy
(0, 0).
3.20 Probleme 123
4. Fie f : R
2
R funct
,
ia denita prin
f(x, y) =
_
y
2
ln
_
x
2
+y
2
_
daca (x, y) = (0, 0)
0 daca (x, y) = (0, 0).
a) Sa se demonstreze ca f este derivabila part
,
ial pe R
2
.
b) Sa se demonstreze ca f este de doua ori derivabila part
,
ial pe R
2
atat
n raport cu variabilele (x, y) cat s
,
i n raport cu variabilele (y, x) s
,
i ca

2
f
yx
(x, y) =

2
f
xy
(x, y) oricare ar (x, y) R
2
.
c) Sa se verice ca funct
,
iile
f
x
s
,
i
f
y
nu sunt diferent
,
iabile n (0, 0), iar
funct
,
iile

2
f
yx
s
,
i

2
f
xy
nu sunt continue n (0, 0).
5. Fie A = { (x, y) R
2
| x
2
+y
2
< 1 }, iar f : A R funct
,
ia denita prin
f(x, y) =
_
xy

ln(x
2
+y
2
) daca (x, y) = (0, 0)
0 daca (x, y) = (0, 0).
Sa se demonstreze ca:
a) f este de doua ori derivabila part
,
ial atat n raport cu variabilele (x, x)
cat s
,
i n raport cu variabilele (y, y) pe A.
b) Derivatele part
,
iale

2
f
x
2
s
,
i

2
f
y
2
sunt continue pe A.
c) f nu este de doua ori derivabila part
,
ial nici n raport cu variabilele
(x, y) nici n raport cu variabilele (y, x) n punctul (0, 0).
6. Fie f : R
2
R o funct
,
ie de doua ori derivabila part
,
ial n raport cu
variabilele (x, y) pe R
2
s
,
i cu proprietatea

2
f
yx
(x, y) 0 pentru orice
(x, y) R
2
. Sa se demonstreze ca pentru orice numere reale a < b s
,
i
c < d are loc inegalitatea
f(b, d) f(b, c) f(a, d) +f(a, c) 0.
7. Fie f : R
3
R funct
,
ia denita prin f(x, y, z) =

x
2
+y
2
+z
2
. Sa se
demonstreze ca d
2
f(x, y, z) este o forma patratica pozitiv semidenita
pentru orice (x, y, z) = (0, 0, 0).
124 3 Calcul diferent
,
ial n R
n
8. Sa se determine numerele reale a s
,
i b as
,
a ncat pentru orice funct
,
ie de
doua ori diferent iabila f : R
2
R, cu proprietatea

2
f
xy
= 0, funct ia
F : R
2
R, denita prin F(x, y) = e
ax+by
f(x, y), sa satisfaca
(x, y) R
2
:

2
F
xy
(x, y)
F
x
(x, y)
F
y
(x, y) +F(x, y) = 0.
9. Fie f = f(x, y) : R
2
R o funct
,
ie de doua ori diferent
,
iabila pe R
2
, cu
proprietatea
(x, y) R
2
:

2
f
x
2
(x, y) +

2
f
y
2
(x, y) = 0,
iar F : R
2
R funct
,
ia denita prin F(, ) = f( cos , sin ). Sa se
demonstreze ca
(, ) R
2
:
2

2
F

2
(, ) +

2
F

2
(, ) +
F

(, ) = 0.
10. Fie f = f(u, v) : R
2
R o funct
,
ie de doua ori diferent
,
iabila pe R
2
, cu
proprietatea
(u, v) R
2
:
f
v
(u, v) =

2
f
u
2
(u, v),
iar F : R (0, ) R funct
,
ia denita prin
F(x, y) =
1

y
e

x
2
4y
f
_
x
y
,
1
y
_
.
Sa se demonstreze ca
(x, y) R (0, ) :
F
y
(x, y) =

2
F
x
2
(x, y).
11. Fie f = f(x, y) : R
2
R s
,
i F = F(u, v) : R
2
R funct
,
ii de doua ori
diferent
,
iabile pe R
2
, cu proprietatea f(x, y) = F
_
xy,
x
y
_
pentru orice
(x, y) R
2
cu y = 0. Sa se demonstreze ca daca
(x, y) R
2
: x
2

2
f
x
2
(x, y) y
2

2
f
y
2
(x, y) = 0,
3.20 Probleme 125
atunci
2xy

2
F
uv
_
xy,
x
y
_

F
v
_
xy,
x
y
_
= 0
pentru orice (x, y) R
2
cu x = 0 s
,
i y = 0.
12. Fie f = f(x, y) : R
2
R s
,
i F = F(u, v) : R
2
R funct
,
ii de doua ori
diferent
,
iabile pe R
2
, cu proprietatea
f(x, y) = xF
_
x +y,
y
x
_
pentru orice (x, y) R
2
cu x = 0. Sa se demonstreze ca daca
(x, y) R
2
:

2
f
x
2
(x, y) 2

2
f
xy
(x, y) +

2
f
y
2
(x, y) = 0,
atunci

2
F
v
2
_
x +y,
y
x
_
= 0
pentru orice (x, y) R
2
cu x = 0 s
,
i x +y = 0.
13. Fie f = f(x, y) : R
2
R s
,
i F = F(u, v) : R
2
R funct
,
ii de doua
ori diferent
,
iabile pe R
2
, cu proprietatea F(x
2
, y
2
) = xy +f(x, y) pentru
orice (x, y) R
2
. Sa se arate ca daca
1
x
2

2
f
x
2
(x, y) +
1
y
2

2
f
y
2
(x, y) =
1
x
3
f
x
(x, y) +
1
y
3
f
y
(x, y) +
y
x
3
+
x
y
3
pentru orice (x, y) R
2
cu x = 0 s
,
i y = 0, atunci

2
F
u
2
(x
2
, y
2
) +

2
F
v
2
(x
2
, y
2
) = 0
pentru orice (x, y) R
2
cu x = 0 s
,
i y = 0.
14. Fie A = [0, 1]
n
s
,
i f : R
n
R o funct
,
ie de doua ori diferent
,
iabila care
ndeplines
,
te urmatoarele condit
,
ii:
(i) x bd A : f(x) = 0;
(ii) min f(A) = 1.
Sa se demonstreze ca
j {1, . . . , n} : sup
_

2
f
x
2
j
(x)

x A
_
8
s
,
i ca, n inegalitatea de mai sus, 8 nu se poate nlocui, n general, cu o
constanta mai mare.
126 3 Calcul diferent
,
ial n R
n
15. Fie f : R
n
R o funct
,
ie de doua ori diferent
,
iabila care se bucura de
urmatoarele proprietat
,
i:
(i) exista s
,
i este nita limita lim
x
f(x);
(ii) toate derivatele part
,
iale de ordinul al doilea ale lui f sunt marginite
pe R
n
.
Sa se demonstreze ca lim
x
f(x) = 0.
3.21 Condit
,
ii necesare s
,
i suciente de extrem
3.21.1 Denit
,
ie. Fie : R
n
R o forma patratica. Se spune ca este:
a) indenita daca a, b R
n
: (a) < 0 < (b);
b) pozitiv semidenita daca h R
n
: (h) 0;
c) pozitiv denita daca h R
n
\ {0
n
} : (h) > 0;
d) negativ semidenita daca h R
n
: (h) 0;
e) negativ denita daca h R
n
\ {0
n
} : (h) < 0.
3.21.2 Teorema (caracterizarea formelor patratice pozitiv denite). O forma
patratica : R
n
R este pozitiv denita daca s
,
i numai daca exista un > 0
astfel ca (h) h
2
oricare ar h R
n
.
Demonstrat
,
ie. Necesitatea. Presupunem ca este o forma patratica pozitiv
denita, adica (x) > 0 pentru orice x R
n
\ {0
n
}. Fie
S
n1
:= {x R
n
| x = 1}
sfera cu centrul n origine s
,
i de raza 1 din R
n
. Deoarece S
n1
este o mult
,
ime
compacta s
,
i funct ia este continua, conform teoremei lui Weierstrass exista
un a S
n1
astfel ca (a) = min
xS
n1
(x). Notam := (a). Cum a = 0
n
,
avem > 0.
Daca h = 0
n
, atunci inegalitatea din enunt
,
are loc cu egalitate, deoarece
(0
n
) = 0. Daca h = 0
n
, atunci
1
h
h S
n1
, deci

_
1
h
h
_
=
1
h
2
(h),
3.21 Condit
,
ii necesare s
,
i suciente de extrem 127
de unde (h) h
2
.
Sucient
,
a. Este evidenta.
3.21.3 Teorema. Fie A R
n
, a int A, iar f : A R o funct
,
ie de doua
ori diferent
,
iabila n punctul a. Atunci urmatoarele armat
,
ii sunt adevarate:
1

Daca a este punct de minim local (respectiv de maxim local) al lui


f, atunci f(a) = 0
n
s
,
i d
2
f(a) este pozitiv semidenita (respectiv negativ
semidenita).
2

Daca f(a) = 0
n
s
,
i d
2
f(a) este pozitiv denita (respectiv negativ
denita), atunci a este punct de minim local (respectiv de maxim local) al
lui f.
Demonstrat
,
ie. 1

Presupunem, pentru xarea ideilor, ca a este punct de minim


local al lui f. Aplicand teorema 3.11.2 (a lui Fermat), rezulta ca f(a) = 0
n
.
Ramane sa mai dovedim ca d
2
f(a) este pozitiv semidenita. Fie n acest
scop h un punct oarecare al lui R
n
s
,
i e > 0 arbitrar. Conform teoremei
3.19.3, avem
lim
xa
1
x a
2
_
f(x) f(a) df(a)(x a)
1
2
d
2
f(a)(x a)
_
= lim
xa
1
x a
2
_
f(x) f(a)
1
2
d
2
f(a)(x a)
_
= 0.
Exista as
,
adar o vecinatate V V(a) astfel ncat pentru orice x V A\ {a}
sa avem

f(x) f(a)
1
2
d
2
f(a)(x a)

x a
2
< .
Deducem de aici ca
f(x) f(a)
1
2
d
2
f(a)(x a) +x a
2
pentru orice x V A.
Dar a ind punct de minim local al lui f, exista o vecinatate W V(a) astfel
ca f(a) f(x) oricare ar x W A. Drept urmare, avem
0
1
2
d
2
f(a)(x a) +x a
2
pentru orice x V W A.
Alegem un t > 0 sucient de mic, ncat a + th V W A. Inlocuindu-l n
inegalitatea precedenta pe x cu a +th, obt
,
inem
0
1
2
d
2
f(a)(th) +th
2
=
t
2
2
d
2
f(a)(h) +t
2
h
2
,
128 3 Calcul diferent
,
ial n R
n
de unde
0
1
2
d
2
f(a)(h) +h
2
.
Facand 0, deducem ca d
2
f(a)(h) 0. Cum h R
n
a fost arbitrar,
conchidem ca d
2
f(a) este o forma patratica pozitiv semidenita.
2

Presupunand ca f(a) = 0
n
s
,
i d
2
f(a) este pozitiv denita, sa aratam
ca a este punct de minim local al lui f. Aplicand teorema 3.21.2, deducem ca
exista un > 0 astfel ca
d
2
f(a)(h) h
2
pentru orice h R
n
.
Pe de alta parte, teorema 3.19.3 garanteaza ca
lim
xa
1
x a
2
_
f(x) f(a)
1
2
d
2
f(a)(x a)
_
= 0.
Exista deci V V(a) astfel ncat pentru orice x V A\ {a} sa avem

f(x) f(a)
1
2
d
2
f(a)(x a)

x a
2
<

4
.
Deducem de aici ca
f(x) f(a)
1
2
d
2
f(a)(x a)

4
x a
2
pentru orice x V A.
Drept urmare avem
f(x) f(a)
1
2
d
2
f(a)(x a)

4
x a
2

1
2
x a
2


4
x a
2
0,
adica f(x) f(a) pentru orice x V A. Aceasta arata ca a este punct de
minim local al lui f.
3.21.4 Observat
,
ii. 1

Daca f(a) = 0
n
s
,
i d
2
f(a) este o forma patratica
indenita, atunci a nu este punct de extrem local al lui f.
2

(Criteriul lui Sylvester). Fie C = (c


ij
) R
nn
o matrice patrata
simetrica s
,
i : R
n
R, (h) =
n

i=1
n

j=1
c
ij
h
i
h
j
, forma patratica asociata
matricei C. Pentru ecare k {1, . . . , n} notam
k
:= det (c
ij
)
1i,jk
. Atunci
urmatoarele armat
,
ii sunt adevarate:
3.21 Condit
,
ii necesare s
,
i suciente de extrem 129
1

este pozitiv denita daca s


,
i numai daca inegalitatea
k
> 0 are loc
pentru orice k {1, . . . , n}.
2

este negativ denita daca s


,
i numai daca inegalitatea (1)
k

k
> 0
are loc pentru orice k {1, . . . , n}.
Fie m, n N cu m < n, A R
n
o mult
,
ime deschisa nevida, iar f : A R
s
,
i F = (F
1
, . . . , F
m
) : A R
m
funct
,
ii cu urmatoarele proprietat
,
i:
(i) f este de doua ori diferent
,
iabila pe A;
(ii) F
1
, . . . , F
m
sunt de doua ori diferent
,
iabile pe A.
Notam C := { x A | F(x) = 0
m
}. Fie apoi L : A R
m
R funct
,
ia lui
Lagrange asociata funct
,
iilor f s
,
i F:
L(x, ) = f(x) +
1
F
1
(x) + +
m
F
m
(x).
3.21.5 Teorema. Fie c C un punct cu proprietatea rang J(F)(c) = m.
Atunci urmatoarele armat
,
ii sunt adevarate:
1

Daca c este punct de minim local (respectiv de maxim local) al lui f


relativ la C, atunci exista
0
R
m
astfel ca
dL(c,
0
) = 0
d
2
L
0
(c)(h) 0 (respectiv d
2
L
0
(c)(h) 0)
pentru orice h R
n
cu proprietatea dF(c)(h) = 0
m
, unde L
0
: A R este
funct
,
ia denita prin
(1) L
0
(x) = f(x) +
01
F
1
(x) + +
0m
F
m
(x).
2

Daca exista
0
R
m
as
,
a ncat
dL(c,
0
) = 0
d
2
L
0
(c)(h) > 0 (respectiv d
2
L
0
(c)(h) < 0)
pentru orice h R
n
\{0
n
} cu proprietatea dF(c)(h) = 0
m
, unde L
0
este funct
,
ia
denita prin (1), atunci c este punct de minim local (respectiv de maxim local)
al lui f relativ la mult
,
imea C.
Demonstrat
,
ie. Fara demonstrat
,
ie.
130 3 Calcul diferent
,
ial n R
n
3.22 Probleme
1. Sa se determine punctele critice ale urmatoarelor funct
,
ii s
,
i sa se precizeze
natura acestora:
a) f : R
2
R, f(x, y) = x
4
+y
4
4(x y)
2
;
b) f : (0, )
2
R, f(x, y) = x
3
+ 3xy
2
15x 12y;
c) f : (0, ) R R, f(x, y) = x(y
2
+ ln
2
x);
d) f : R
2
R, f(x, y) = 3x 3y 2x
3
xy
2
+ 2x
2
y +y
3
;
e) f : R
3
R, f(x, y, z) =
1
2
x
2
+xyz y +z;
f) f : R
3
R, f(x, y, z) = x
2
+y
2
+z
2
2xyz;
g) f : R
3
R, f(x, y, z) = 2x
2
xy + 2xz y +y
3
+z
2
;
h) f : R
3
R, f(x, y, z) = x
3
3x +y
2
+z
2
;
i) f : R
3
R, f(x, y, z) =
xye
y
1 +x
2
+z
2
+ 2z;
j) f : R
3
R, f(x, y, z) =
3x
4
6x
2
+ 1
1 +z
2
y
2
;
k) f : R
3
R, f(x, y, z) = z
2
(1 +xy) +xy;
l) f : R
3
R, f(x, y, z) = (x +y)e
x+yz
2
;
m) f : (0, )
3
R, f(x, y, z) = xyz(ln x)(ln y)(ln z);
n) f : R
3
R, f(x, y, z) = xye
z
+yze
x
+zxe
y
;
p) f : R
3
R, f(x, y, z) = xe
y
+ye
z
+ze
x
.
2. Sa se demonstreze ca funct
,
ia f : R
2
R, denita prin
f(x, y) = (1 +e
x
) cos y xe
x
,
are o innitate de maxime locale s
,
i nici un minim local.
3. Sa se demonstreze ca funct
,
ia f : R
2
R, denita prin
f(x, y) = x
2
+y
2
(1 x)
3
,
este diferent
,
iabila, are un singur punct critic care este punct de minim
local, dar inf f(R
2
) = . Exista funct
,
ii f : R R cu aceleas
,
i pro-
prietat
,
i?
3.22 Probleme 131
4. Fie a
1
, . . . , a
n
numere reale strict pozitive s
,
i f : (0, )
n
R funct
,
ia
denita prin
f(x
1
, . . . , x
n
) = x
1
ln
a
1
x
1
+ +x
n
ln
a
n
x
n
.
Sa se determine punctele de extrem local ale lui f s
,
i sa se precizeze
natura acestora.
5. Fie f : R
2
R o funct
,
ie de doua ori diferent
,
iabila pe R
2
, cu proprietatea
(x, y) R
2
:

2
f
x
2
(x, y) +

2
f
y
2
(x, y) > 0.
Sa se demonstreze ca f nu are nici un maxim local.
Berkeley 1988
6. Fie A = { (x, y) R
2
| x
2
+ y
2
1 } s
,
i f : A R o funct
,
ie care
ndeplines
,
te urmatoarele condit
,
ii:
(i) f este continua pe A;
(ii) f admite toate derivatele part
,
iale de ordinul doi pe int A s
,
i acestea
sunt continue pe int A;
(iii) f se anuleaza pe bd A;
(iv) f se anuleaza n cel put
,
in un punct interior lui A.
Sa se demonstreze ca exista un punct c int A astfel ca

2
f
x
2
(c) +

2
f
y
2
(c) = 0.
Ramane adevarata concluzia daca se renunt
,
a la ipoteza (iv)?
G. Polya
7. Fie funct
,
ia f : R
3
R, denita prin f(x, y, z) = x + y + z s
,
i mult
,
imea
C = { (x, y, z) R
3
| x
2
+y
2
+z
2
= 1, 2x+y+2z = 1 }. Sa se determine
min f(C) s
,
i max f(C).
8. Fie funct
,
ia f : R
3
R, denita prin f(x, y, z) = x
2
+y
2
z
2
s
,
i mult
,
imea
C = { (x, y, z) R
3
| x
2
+y
2
+z
2
= 9, x +y +z = 5 }. Sa se determine
min f(C) s
,
i max f(C).
132 3 Calcul diferent
,
ial n R
n
9. Fie funct
,
ia f : R
3
R, denita prin
f(x, y, z) = x
2
+y
2
+z
2
6x + 8y + 24z
s
,
i mult
,
imea C = { (x, y, z) R
3
| x
2
+ y
2
+ z
2
1 }. Sa se determine
min f(C) s
,
i max f(C).
10. Fie funct
,
ia f : R
3
R, denita prin
f(x, y, z) = (1 x)(1 y)(1 z)
s
,
i mult
,
imea C = { (x, y, z) R
3
| x
2
+ y
2
+ z
2
1 }. Sa se determine
max f(C).
11. Fie a, b, c > 0, C =
_
(x, y, z) R
3

x
2
a
2
+
y
2
b
2
+
z
2
c
2
= 1
_
s
,
i e funct
,
ia
f : R
3
R, denita prin f(x, y, z) = xy
2
z
6
. Sa se determine min f(C)
s
,
i max f(C).
12. Sa se determine punctele de extrem local ale funct
,
iei f : R
3
R, denite
prin f(x, y, z) = xy +yz +zx, relative la mult
,
imea
C = { (x, y, z) R
3
| xyz = 1 }
s
,
i sa se precizeze natura acestora.
13. Sa se determine punctele de extrem local ale funct
,
iei f : R
3
R, denite
prin f(x, y, z) = xyz, relative la mult
,
imea
C = { (x, y, z) R
3
| xy +yz +zx = 3 }
s
,
i sa se precizeze natura acestora. Sa se determine inf f(C) s
,
i sup f(C).
14. Sa se determine punctele de extrem local ale funct
,
iei f : R
3
R, denite
prin f(x, y, z) = xyz, relative la mult
,
imea
C = { (x, y, z) R
3
| x +y +z = 5, xy +yz +zx = 8 }
s
,
i sa se precizeze natura acestora.
15. Fie a > 0 s
,
i g : R
2
R funct
,
ia denita prin
g(x, y) = x
2
+y
2

a
2
_
x +

x
2
+y
2
_
.
a) Sa se demonstreze ca g este de clasa C
1
pe R
2
\ {(0, 0)}.
b) Fie C = {(x, y) R
2
| g(x, y) = 0}. Sa se determine punctele
(x, y) C \ {(0, 0)}, n care coordonata x poseda un extrem local.
3.22 Probleme 133
16. Cu ajutorul regulii multiplicatorilor lui Lagrange, sa se determine norma
aplicat
,
iei liniare : R
3
R
3
, s
,
tiind ca
[] =
_
_
1 0 1
0 1 1
1 0 1
_
_
.
17. Fie P
2
mult
,
imea tuturor funct
,
iilor polinomiale cu coecient
,
i reali, de
grad cel mult 2, J : P
2
R funct
,
ia denita prin
J(f) =

1
0
f
2
(x)dx,
iar Q = { f P
2
| f(1) = 1 }. Sa se demonstreze ca J are o valoare
minima pe Q s
,
i sa se determine elementele lui Qn care se atinge aceasta
valoare minima.
Berkeley 1980
18. Fie a
1
, . . . , a
n
numere reale strict pozitive, f : R
n
R funct
,
ia denita
prin f(x
1
, . . . , x
n
) = a
1
x
2
1
+ +a
n
x
2
n
, iar
C = { (x
1
, . . . , x
n
) R
n
| x
1
+ +x
n
= 1 }.
Sa se demonstreze ca
min f(C) =
1
1
a
1
+ +
1
a
n
.
19. Sa se determine maximul produsului (1x
1
) (1x
n
) daca variabilele
x
1
, . . . , x
n
[0, ) satisfac x
2
1
+ +x
2
n
= 1.
Hassan Ali Shah Ali, Amer. Math. Monthly [1998, 176]
20. Fie : R
n
R
m
o aplicat
,
ie liniara avand matricea [] R
mn
.
Cu ajutorul regulii multiplicatorilor lui Lagrange, sa se demonstreze
ca =

, unde

este cea mai mare valoare proprie a matricei


simetrice []
T
[].
21. Fie : R
n
R
n
o aplicat
,
ie liniara cu proprietatea
x, y R
n
: (x), y = x, (y).
(Aceasta este echivalent cu a spune ca matricea aplicat
,
iei este sime-
trica.) Fie apoi f : R
n
R funct
,
ia denita prin f(x) = (x), x. Sa
se demonstreze ca daca x S
n1
este un punct de extrem local al lui f
relativ la mult
,
imea S
n1
, atunci x este un vector propriu al lui .
134 3 Calcul diferent
,
ial n R
n
3.23 Derivate part
,
iale s
,
i diferent
,
iale de ordin
superior
3.23.1 Denit
,
ie (derivate part
,
iale de ordin superior). Fie A R
n
, a int A
s
,
i f : A R o funct
,
ie. Inductiv, se poate deni pentru f derivabilitatea
part
,
iala de orice ordin k N. Fie k 1 un numar natural pentru care au fost
denite derivatele part
,
iale de ordinul k ale lui f. Fie apoi i
1
, . . . , i
k
, i
k+1

{1, . . . , n}. Presupunem ca exista o vecinatate deschisa V V(a), as
,
a ncat
V A, care ndeplines
,
te urmatoarele condit ii:
(i) funct
,
ia f este de k ori derivabila part
,
ial pe V n raport cu variabilele
(x
i
1
, . . . , x
i
k
);
(ii) funct
,
ia
(1) x V

k
f
x
i
k
x
i
1
(x) R
este derivabila part
,
ial n raport cu variabila x
i
k+1
n punctul a.
Atunci se spune ca f este de k+1 ori derivabila part ial n raport cu variabilele
(x
i
1
, . . . , x
i
k
, x
i
k+1
) n punctul a. Derivata part
,
iala a funct
,
iei (1) n raport cu
variabila x
i
k+1
n punctul a se numes
,
te derivata part ial a de ordinul k + 1 a
funct
,
iei f n raport cu variabilele (x
i
1
, . . . , x
i
k
, x
i
k+1
) n a s
,
i se noteaza cu

k+1
f
x
i
k+1
x
i
k
x
i
1
(a) sau cu f
(k+1)
x
i
1
x
i
k
x
i
k+1
(a).
3.23.2 Denit
,
ie (diferent
,
iabilitate de ordin superior). Tot inductiv, se poate
deni conceptul de diferent
,
iabilitate de orice ordin k N pentru f. Fie k 1
un numar natural pentru care a fost deja denita diferent iabilitatea de ordinul
k. Mai presupunem ca s-a demonstrat urmatoarea armat
,
ie: daca funct
,
ia
f : A R este de k ori diferent
,
iabila ntr-un punct a int A atunci pentru
orice indici i
1
, . . . , i
k
{1, . . . , n}, f este de k ori derivabila part
,
ial n raport
cu variabilele (x
i
1
, . . . , x
i
k
) n punctul a.
Fie A R
n
, a int A s
,
i f : A R o funct
,
ie. Daca exista o vecinatate
deschisa V V(a) cu V A as
,
a nat
(i) funct
,
ia f este de k ori diferent
,
iabila pe V ;
(ii) pentru orice indici i
1
, . . . , i
k
{1, . . . , n}, funct
,
ia (1) este diferent
,
iabi-
la n a,
3.23 Derivate part
,
iale s
,
i diferent
,
iale de ordin superior 135
atunci se spune ca f este de k + 1 ori diferent
,
iabila n punctul a. Teorema
3.6.4 mpreuna cu (ii) garanteaza ca pentru orice i
1
, . . . , i
k
, i
k+1
{1, . . . , n},
f este de k+1 ori derivabila part
,
ial n raport cu variabilele (x
i
1
, . . . , x
i
k
, x
i
k+1
)
n punctul a.
3.23.3 Teorema. Fie A o submult
,
ime a spat
,
iului R
n
, a int A, k N s
,
i
f : A R o funct
,
ie de k ori diferent
,
iabila n punctul a. Atunci pentru orice
i
1
, . . . , i
k
{1, . . . , n} s
,
i pentru orice permutare a mult
,
imii {1, . . . , k}, are
loc egalitatea

k
f
x
i
k
x
i
1
(a) =

k
f
x
i
(k)
x
i
(1)
(a).
Demonstrat
,
ie. Fara demonstrat
,
ie.
3.23.4 Denit
,
ie (diferent
,
iala de ordinul k). Fie A R
n
, a int A, k N
s
,
i f : A R o funct
,
ie de k ori diferent
,
iabila n a. Funct
,
ia d
k
f(a) : R
n
R,
denita prin
d
k
f(a)(h) :=
n

i
1
=1

n

i
k
=1
h
i
1
h
i
k

k
f
x
i
1
x
i
k
(a)
oricare ar h = (h
1
, . . . , h
n
) R
n
, se numes
,
te diferent
,
iala de ordinul k a
funct
,
iei f n punctul a. Se observa ca d
k
f(a) este un polinom omogen de
gradul k n variabilele h
1
, . . . , h
n
.
3.23.5 Teorema. Fie A o submult
,
ime deschisa convexa nevida a lui R
n
,
e k {0, 1, 2, . . .} s
,
i e f : A R o funct
,
ie de k + 1 ori diferent
,
iabila
pe A. Atunci pentru orice a, b A exista un (0, 1) as
,
a ncat punctul
c := (1 )a +b sa satisfaca egalitatea
f(b) = f(a) +
k

j=1
1
j!
d
j
f(a)(b a) +
1
(k + 1)!
d
k+1
f(c)(b a).
Demonstrat
,
ie.
3.23.6 Observat
,
ie. Egalitatea din teorema 3.23.5 se numeste formula lui
Taylor cu restul lui Lagrange.
3.23.7 Teorema. Fie A R
n
, a int A, k N, iar f : A R o funct
,
ie de
k ori diferent
,
iabila n a. Atunci are loc egalitatea
lim
xa
1
x a
k
_
_
f(x) f(a)
k

j=1
1
j!
d
j
f(a)(x a)
_
_
= 0.
Demonstrat
,
ie. Fara demonstrat
,
ie.
136 3 Calcul diferent
,
ial n R
n
3.24 Probleme
1. Fie f : R
2
R funct
,
ia denita prin f(x, y) = (x
2
+ y
2
)e
x+y
. Sa se
determine

m+n
f
x
m
y
n
.
2. Fie f : R
2
R funct
,
ia denita prin f(x, y) = e
xy
. Sa se determine

m+n
f
x
m
y
n
.
3. Fie f : R
n
R o funct
,
ie de doua ori diferent
,
iabila pe R
n
, care ndepli-
nes
,
te urmatoarele condit
,
ii:
(i) x R
n
: f(x) 0;
(ii) x R
n
, h R
n
: d
2
f(x)(h) 0.
Sa se demonstreze ca f este constanta.
4. (Generalizarea problemei precedente) Fie k N s
,
i f : R
n
R o funct
,
ie
de 2k ori diferent iabila pe R
n
, care ndeplines
,
te urmatoarele condit ii:
(i) x R
n
: f(x) 0;
(ii) x R
n
, h R
n
: d
2k
f(x)(h) 0.
Sa se demonstreze ca f este o funct
,
ie polinomiala de n variabile avand
gradul cel mult 2k 2.
5. Fie r > 0, A = {x R
n
| x < r} s
,
i f : A [0, ) o funct
,
ie de doua
ori diferent
,
iabila pe A, care ndeplines
,
te urmatoarele condit
,
ii:
(i) exista un a A as
,
a ncat f(a) = 0;
(ii) x A :
n

i=1
n

j=1
_

2
f
x
i
x
j
(x)
_
2
1.
Sa se demonstreze ca f(x) < 2r
2
oricare ar x A.
6. a) Fie A R
n
convexa deschisa nevida s
,
i f : A R o funct
,
ie de doua
ori diferent iabila pe A. Sa se demonstreze ca daca d
2
f(x) este o forma
patratica pozitiv semidenita pentru orice x A, atunci ecare punct
critic al lui f este punct de minim global.
b) Fie A = {(x
1
, x
2
) R
2
| x
2
> 0} s
,
i f : A R funct
,
ia denita prin
f(x
1
, x
2
) =
x
2
1
x
2
+x
2
ln x
2
x
1
x
2
.
Folosind eventual armat
,
ia a), sa se determine inf f(A).
Bibliograe
[1] W. W. Breckner: Analiza matematica. Topologia spat
,
iului R
n
. Uni-
versitatea din Cluj-Napoca, 1985.
[2] S. Cobzas
,
: Analiza matematica (calculul diferent
,
ial). Presa Universitara
Clujeana, Cluj-Napoca, 1997.
[3] P. N. De Souza s
,
i J.-N. Silva: Berkeley Problems in Mathematics.
Springer, 1998.
[4] P.M. Fitzpatrick: Advanced Calculus: Second Edition. AMS, 2006
[5] K. S. Kedlaya, B. Poonen s
,
i R. Vakil: The William Lowell Put-
nam Mathematical Competition 1985 - 2000. Problems, Solutions, and
Commentary. The Mathematical Association of America, 2002.
[6] S. R adulescu s
,
i M. R adulescu: Teoreme s
,
i probleme de analiza
matematica. Editura Didactica s
,
i Pedagogica, Bucures
,
ti, 1982.
[7] T. Trif: Probleme de calcul diferent
,
ial s
,
i integral n R
n
. Universitatea
Babes
,
-Bolyai, Cluj-Napoca, 2003.
137
Index
s
,
ir n R
n
, 30
convergent, 30
fundamental, 31
aplicat
,
ie liniara, 49
norma unei aplicat
,
ii liniare, 52
baza canonica a lui R
n
, 23
criterii de convergent
,
a pentru integra-
le improprii
criteriul comparat
,
iei, 9
criteriul lui Abel, 13
criteriul lui Cauchy, 8
criteriul majorantei, 13
derivata part
,
iala, 63
de ordin superior, 118
de ordinul doi, 108
derivata dupa o direct
,
ie, 61
difeomorsm, 96
de clasa C
1
, 96
diferent
,
iala a doua, 112
diferent
,
iala Frechet, 58
distant
,
a, 25
distant
,
a euclidiana n R
n
, 24
funct
,
ie
continua, 44
de clasa C
1
, 95
de doua ori diferent
,
iabila, 111
derivabila dupa o direct
,
ie, 61
derivabila part
,
ial, 63
diferent
,
iabila Frechet, 58
local integrabila, 1
reala de variabila vecrtoriala, 41
vectoriala de variabila reala, 41
vectoriala de variabila vectoriala,
41
gradient, 64
integrala improprie
absolut convergenta, 12
convergenta, 1, 2
semiconvergenta, 13
matrice
a unei aplicat
,
ii liniare, 50
hessiana a unei funct
,
ii, 108
Jacobi a unei funct
,
ii, 64
metrica, 25
metrica euclidiana n R
n
, 24
mult
,
ime
nchisa, 27
compacta, 33
convexa, 90
deschisa, 27
marginita, 35
secvent
,
ial compacta, 35
norma, 24
norma euclidiana n R
n
, 24
produs scalar, 23
n R
n
, 23
138
139
punct
s
,
a, 89
aderent, 25
critic, 89
de acumulare, 26
de extrem, 88
de extrem condit
,
ionat, 107
exterior, 25
frontiera, 26
interior, 25
izolat, 26
spat
,
iu
liniar real, 22
metric, 25
normat real, 24
prehilbertian real, 23
topologic, 29
topologie, 29

S-ar putea să vă placă și